Top Banner
Question #: 1 Which of the following structures represents a site of compression of the median nerve at the elbow? 1- Ligament of Struthers 2- Intermuscular septum 3- Osborne’s ligament 4- Fascia of the flexor carpi ulnaris 5- Flexor-pronator aponeurosis in the forearm Preferred Response: 1 Recommended Reading(s): Elhassan B, Steinmann SP: Entrapment neuropathy of the ulnar nerve. J Am Acad Orthop Surg 2007;15:672-681. Bainbridge C: Cubital tunnel syndrome, in Berger RA, Weiss APC (eds): Hand Surgery. Philadelphia, PA, Lippincott Williams & Wilkins, 2004, pp 887-896. Question #: 2 In infants with congenital muscular torticollis, passive stretching exercises should include 1- lateral head tilt with no rotation. 2- lateral head tilt and chin rotation toward the affected side. 3- lateral head tilt and chin rotation away from the affected side. 4- lateral head tilt toward the affected side and chin rotation away from the affected side. 5- lateral head tilt away from the affected side and chin rotation toward the affected side. Preferred Response: 5 Recommended Reading(s): Coventry MB, Harris LE: Congenital muscular torticollis in infancy: Some observations regarding treatment. J Bone Joint Surg Am 1959;41:815-822. Loder RT: The cervical spine, in Morrissy RT, Weinstein SL (eds): Lovell and Winter’s Pediatric Orthopaedics, ed 6. Philadelphia, PA, Lippincott Williams & Wilkins, 2006, pp 871-919.
229
Welcome message from author
This document is posted to help you gain knowledge. Please leave a comment to let me know what you think about it! Share it to your friends and learn new things together.
Transcript
Page 1: 2009 oite review

Question #: 1 Which of the following structures represents a site of compression of the median nerve at the elbow? 1- Ligament of Struthers 2- Intermuscular septum 3- Osborne’s ligament 4- Fascia of the flexor carpi ulnaris 5- Flexor-pronator aponeurosis in the forearm Preferred Response: 1 Recommended Reading(s): Elhassan B, Steinmann SP: Entrapment neuropathy of the ulnar nerve. J Am Acad Orthop Surg 2007;15:672-681. Bainbridge C: Cubital tunnel syndrome, in Berger RA, Weiss APC (eds): Hand Surgery. Philadelphia, PA, Lippincott Williams & Wilkins, 2004, pp 887-896. Question #: 2 In infants with congenital muscular torticollis, passive stretching exercises should include 1- lateral head tilt with no rotation. 2- lateral head tilt and chin rotation toward the affected side. 3- lateral head tilt and chin rotation away from the affected side. 4- lateral head tilt toward the affected side and chin rotation away from the affected side. 5- lateral head tilt away from the affected side and chin rotation toward the affected side. Preferred Response: 5 Recommended Reading(s): Coventry MB, Harris LE: Congenital muscular torticollis in infancy: Some observations regarding treatment. J Bone Joint Surg Am 1959;41:815-822. Loder RT: The cervical spine, in Morrissy RT, Weinstein SL (eds): Lovell and Winter’s Pediatric Orthopaedics, ed 6. Philadelphia, PA, Lippincott Williams & Wilkins, 2006, pp 871-919.

Page 2: 2009 oite review

Question #: 3 What is the best way to prevent failure due to cut-out after fixation of an unstable four-part intertrochanteric fracture with a compression hip screw device? 1- Use of an extra large screw and deep thread design 2- Use of a supplemental anti-rotation screw 3- A Dimon-Hughston medialization to allow load sharing 4- Screw tip placement in the inferior hemisphere of the femoral head 5- Screw tip placement with a tip-apex distance of less than 25 mm Preferred Response: 5 Recommended Reading(s): Baumgaertner MR, Curtin SL, Lindskog DM, et al: The value of the tip-apex distance in predicting failure of fixation of peritrochanteric fractures of the hip. J Bone Joint Surg Am 1995;77:1058-1064. Kyle RF, Gustilo RB, Premer RF: Analysis of six hundred and twenty-two intertrochanteric hip fractures. J Bone Joint Surg Am 1979;61:216-221. Koval KJ, Zuckerman JD: Intertrochanteric fractures, in Bucholz RW, Heckman JD (eds): Rockwood and Green’s Fractures in Adults, ed 5. Philadelphia, PA, Lippincott Williams & Wilkins, 2001, pp 1635-1663. Question #: 4 Cuboidal cells that line along layers of immature osteoid to synthesize bone matrix and are not directly inhibited by bisphosphonates are best described as 1- osteoblasts. 2- osteocytes. 3- osteoclasts. 4- histiocytes. 5- megakarocytes. Preferred Response: 1 Recommended Reading(s): Fischgrund JS (ed): Orthopaedic Knowledge Update 9. Rosemont, IL, American Academy of Orthopaedic Surgeons, 2008, pp 189-196. Einhorn TA, O’Keefe RJ, Buckwalter JA (eds): Orthopaedic Basic Science: Foundations of Clinical Practice, ed 3. Rosemont, IL, American Academy of Orthopaedic Surgeons, 2007, pp 129-159.

Page 3: 2009 oite review

Figure 5a Figure 5b Question #: 5 Figures 5a and 5b show the radiographs of a 64-year-old man who underwent primary total hip arthroplasty 3 months ago for degenerative arthritis. He has now had four dislocations. How should his instability be managed? 1- Acetabular revision 2- Femoral head revision 3- Femoral head and stem revision 4- Abduction orthosis for 6 weeks 5- Placement of a constrained acetabular liner Preferred Response: 1 Recommended Reading(s): Dorr LD, Wan Z: Causes of and treatment protocol for instability of total hip replacement. Clin Orthop Relat Res 1998;355:144-151. Morrey BF: Instability after total hip arthroplasty. Orthop Clin North Am 1992;23:237-248.

Page 4: 2009 oite review

Figure 6 Question #: 6 A 44-year-old woman undergoes a CT myelogram for evaluation of persistent low back pain and bilateral lower extremity paresthesias. Axial imaging through the vertebral body of L4 is shown in Figure 6. Further management should include which of the following? 1- Nuclear bone scan 2- Serum immunoelectrophoresis 3- Biopsy of lytic lesions of the vertebral body 4- Continued observation of the patient’s clinical course 5- Serum thyroid studies Preferred Response: 4 Recommended Reading(s): Ross JS, Bell GR: Spine imaging, in Herkowitz HN, Garfin SR, Eismont FJ, et al (eds): Rothman-Simeone The Spine, ed 5. Philadelphia, PA, Saunders Elsevier, 2006, pp 187-217.

Page 5: 2009 oite review

Question #: 7 Which of the following physical examination findings is most helpful to diagnose a fixed posterior shoulder dislocation? 1- Apprehension sign 2- Sulcus sign 3- Jerk test 4- Jobe relocation test 5- Lack of external rotation Preferred Response: 5 Recommended Reading(s): Fischgrund JS (ed): Orthopaedic Knowledge Update 9. Rosemont, IL, American Academy of Orthopaedic Surgeons, 2008, pp 301-311. Ivkovic A, Boric I, Cicak N: One-stage operation for locked bilateral posterior dislocation of the shoulder. J Bone Joint Surg Br 2007;89:825-828.

Page 6: 2009 oite review

Figure 8 Question #: 8 Figure 8 shows the radiograph of a 14-year-old girl with sickle cell disease and mild left hip discomfort. What is the radiographic natural history of this disease process? 1- Initial loss of sphericity of the femoral head with later restoration of sphericity 2- Maintenance of sphericity of the femoral head 3- Coxa magna without loss of sphericity of the femoral head 4- Progressive loss of sphericity of the femoral head 5- Protrusio acetabuli Preferred Response: 4 Recommended Reading(s): Hernigou P, Habibi A, Bachir D, et al: The natural history of asymptomatic osteonecrosis of the femoral head in adults with sickle cell disease. J Bone Joint Surg Am 2006;88:2565-2572. Aguilar C, Vichinsky E, Neumayr L: Bone and joint disease in sickle cell disease. Hematol Oncol Clin North Am 2005;19:929-941.

Page 7: 2009 oite review

Question #: 9 In a reverse shoulder arthroplasty, placement of the glenoid component in a high position on the native glenoid places the patient at risk of 1- an acromial fracture. 2- scapular notching. 3- postoperative instability. 4- restricted external rotation. 5- early loss of glenoid fixation. Preferred Response: 2 Recommended Reading(s): Simovitch RW, Zumstein MA, Lohri E, et al: Predictors of scapular notching in patients managed with the Delta III reverse total shoulder replacement. J Bone Joint Surg Am 2007;89:588-600. Gutiérrez S, Levy JC, Lee WE III, et al: Center of rotation affects abduction range of motion of reverse shoulder arthroplasty. Clin Orthop Relat Res 2007;458:78-82. Question #: 10 Achondroplasia is associated with which of the following mutations? 1- Insulin-like growth factor-1 receptor 2- Insulin-like growth factor-1 3- Insulin-like growth factor-2 4- Fibroblast growth factor receptor-3 5- Fibroblast growth factor Preferred Response: 4 Recommended Reading(s): Einhorn TA, O’Keefe RJ, Buckwalter JA (eds): Orthopaedic Basic Science: Foundations of Clinical Practice, ed 3. Rosemont, IL, American Academy of Orthopaedic Surgeons, 2007, pp 3-23. Fischgrund JS (ed): Orthopaedic Knowledge Update 9. Rosemont, IL, American Academy of Orthopaedic Surgeons, 2008, pp 773-783.

Page 8: 2009 oite review

Figure 11a Figure 11b Question #: 11 An otherwise healthy 20-year-old dancer has a 1-year history of foot and ankle pain. She describes her great toe locking with active range of motion. Examination reveals triggering of the great toe but no tenderness at the level of the first metatarsal head. Palpable crepitus is present along the medial ankle with active range of motion of the great toe. Radiographs are unremarkable. MRI scans are shown in Figures 11a and 11b. What is the most likely diagnosis? 1- Tarsal tunnel syndrome 2- Medial sesamoid stress fracture 3- Flexor hallucis longus tendon tear at the metatarsophalangeal joint 4- Flexor hallucis longus tendon tear at the posteromedial ankle 5- Chronic deep space infection of the posterior ankle Preferred Response: 4 Recommended Reading(s): Hamilton WG, Hamilton LH: Foot and ankle injuries in dancers, in Coughlin MJ, Mann RA (eds): Surgery of the Foot and Ankle, ed 7. St Louis, MO, Mosby, 1999, pp 1225-1256. Sammarco GJ, Cooper PS: Flexor hallucis longus tendon injury in dancers and nondancers. Foot Ankle Int 1998;9:356-362.

Page 9: 2009 oite review

Question #: 12 A 15-year-old boy has diffuse back pain, particularly in the morning. Examination reveals an increased kyphosis in the thoracic spine. Spinal mobility in flexion and extension is limited. He has decreased chest excursion with inspiration. The FABER test is positive. Standing radiographs show absence of the anterior concavity of the vertebral body and a kyphosis of 62° from T-3 to T-12. What is the most likely diagnosis? 1- Pott disease 2- Reiter syndrome 3- Scheuermann’s kyphosis 4- Ankylosing spondylitis 5- Diffuse idiopathic skeletal hyperostosis Preferred Response: 4 Recommended Reading(s): Azouz EM, Dufy CM: Juvenile spondyloarthropathies: Clinical manifestations and medical imaging. Skeletal Radiol 1995;24:399-408. Gensler L, Davis JC Jr: Recognition and treatment of juvenile-onset spondyloarthritis. Curr Opin Rheumatol 2006;18:507-511. Kredich D, Patrone NA: Pediatric spondyloarthropathies. Clin Orthop Relat Res 1990;259:18-22. Question #: 13 Which of the following represents a contraindication to hyperbaric oxygen therapy? 1- Intubation 2- Osteomyelitis 3- Insulin pump 4- Thermal burns 5- Thoracostomy tube Preferred Response: 3 Recommended Reading(s): Greensmith JE: Hyperbaric oxygen therapy in extremity trauma. J Am Acad Orthop Surg 2004;12:376-384. Kindwall EP: Contraindications and side effects to hyperbaric oxygen treatment, in Kindwall EP, Whelan HT (eds): Hyperbaric medicine practice, ed 2. Flagstaff, AZ, Best Publishing Company, 1999, pp 83-98. Buettner MF, Wolkenhauer D: Hyperbaric oxygen therapy in the treatment of open fractures and crush injuries. Emerg Med Clin North Am 2007;25:177-188.

Page 10: 2009 oite review

Question #: 14 A 42-year-old woman injured her dominant middle finger in a car door. Examination reveals a partial nail avulsion with nail bed lacerations. No fractures are seen on radiographs. Without compromising functional and cosmetic results, what is the most expeditious treatment? 1- Placement of 6-0 chromic suture 2- Placement of 6-0 fast-absorbing gut suture 3- Application of a saline gauze dressing 4- Application of a non-adherent dressing 5- Application of 2-octylcyanoacrylate Preferred Response: 5 Recommended Reading(s): Strauss EJ, Weil WM, Jordan C, et al: A prospective, randomized, controlled trial of 2-octylcyanoacrylate versus suture repair for nail bed injuries. J Hand Surg Am 2008;33:250-253. Richards AM, Chrick A, Cole RP: A novel method of securing the nail following nail bed repair. Plast Reconstr Surg 1999;103:1983-1985.

Page 11: 2009 oite review

Figure 15a Figure 15b Question #: 15 Figures 15a and 15b show the bone scan and CT scan of a 20-year-old collegiate football player who has persistent low back pain without neurologic symptoms. What physical examination maneuver would you expect to be abnormal? 1- Babinski test 2- Femoral stretch test 3- Straight leg raise 4- Contralateral straight leg raise 5- Single-limb standing lumbar extension Preferred Response: 5 Recommended Reading(s): Hu SS, Tribus CB, Diab M, et al: Spondylolisthesis and spondylolysis. J Bone Joint Surg Am 2008;90:656-671. Garrick JG (ed): Orthopaedic Knowledge Update: Sports Medicine 3. Rosemont IL, American Academy of Orthopaedic Surgeons, 2004, pp 19-28.

Page 12: 2009 oite review

Question #: 16 During surgical reduction of a congenitally dislocated hip through an anteromedial approach, what structure is most at risk when performing the psoas tenotomy? 1- Femoral vein 2- Femoral artery 3- Lateral circumflex artery 4- Medial circumflex artery 5- Obturator artery Preferred Response: 4 Recommended Reading(s): Morrissy RT, Weinstein SL: Atlas of Pediatric Orthopaedic Surgery, ed 4. Philadelphia, PA, Lippincott Williams & Wilkins, 2006, pp 271-277. Weinstein SL, Ponseti IV: Congenital dislocation of the hip. J Bone Joint Surg Am 1979;61:119-124.

Page 13: 2009 oite review

Figure 17a Figure 17b

Figure 17c Figure 17d

Page 14: 2009 oite review

Question #: 17 The 20-year-old man sustains the fracture seen in Figures 17a through 17d. What technical aspect of the surgical treatment is most important for the patient’s outcome? 1- Restoring fibular length 2- Achieving fibular compression 3- Reduction of the posterior malleolus 4- Reduction of the anteromedial impacted fragments 5- Fixing the distal tibiofibular syndesmosis Preferred Response: 4 Recommended Reading(s): Marsh JL, Saltzman CL: Ankle fractures, in Rockwood and Green’s Fractures in Adults, ed 6. Philadelphia, PA, Lippincott Williams & Wilkins, 2006, pp 2147-2247. McConnell T, Tornetta P III: Marginal plafond impaction in association with supination-adduction ankle fractures: A report of eight cases. J Orthop Trauma 2001;15:447-449. Question #: 18 A unicompartment knee arthroplasty is contraindicated in which of the following clinical scenarios? 1- Inflammatory arthritis 2- Anteromedial osteoarthritis 3- Lateral compartment osteoarthritis 4- Medial femoral condyle osteonecrosis 5- Advanced age Preferred Response: 1 Recommended Reading(s): Marmor L: Unicompartment arthroplasty for osteonecrosis of the knee joint. Clin Orthop Relat Res 1993;294:247-253. Parrette S, Argenson JN, Dumas J, et al: Unicompartmental knee arthroplasty for avascular necrosis. Clin Orthop Relat Res 2007;464:37-42. Fischgrund JS (ed): Orthopaedic Knowledge Update 9. Rosemont, IL, American Academy of Orthopaedic Surgeons, 2008, pp 457-471.

Page 15: 2009 oite review

Question #: 19 Decreased stature and bone density due to lead poisoning is thought to be caused by interference in the signaling of which of the following growth factors? 1- BMP2 2- RANKL 3- PTHrP/TGF-beta 4- Fibroblast growth factor 3 (FGF3) 5- Insulin-like growth factor (IGF) Preferred Response: 3 Recommended Reading(s): Einhorn TA, O’Keefe RJ, Buckwalter JA (eds): Orthopaedic Basic Science: Foundations of Clinical Practice, ed 3. Rosemont, IL, American Academy of Orthopaedic Surgeons, 2007, pp 115-127. Campbell JR, Rosier RN, Novotny L, et al: The association between environmental lead exposure and bone density in children. Environ Health Perspect 2004;112:1200-1203. Question #: 20 Which of the following is considered the strongest predictor of disability in adult patients with scoliosis? 1- Concomitant diagnosis of osteoporosis 2- Presence of a vacuum disk sign at two or more lumbar levels 3- Sagittal plane imbalance and inability to obtain neutral upright stance 4- The number of spinal levels affected by disk degeneration in the lumbar spine 5- Coronal plane Cobb angle measurement of greater than 30° in the lumbar spine Preferred Response: 3 Recommended Reading(s): Schwab F, Farcy JP, Bridwell K, et al: A clinical impact classification of scoliosis in the adult. Spine 2006;31:2109-2114. Glassman SD, Bridwell K, Dimar JR, et al: The impact of positive sagittal balance in adult spinal deformity. Spine 2005;30:2024-2029. Kim YJ, Bridwell KH, Lenke LG, et al: Sagittal thoracic decompensation following long adult lumbar spinal instrumentation and fusion to L5 or S1: Causes, prevalence, and risk factor analysis. Spine 2006;31:2359-2366.

Page 16: 2009 oite review

Figure 21a Figure 21b Question #: 21 A 9-year-old boy is seen for bilateral thigh pain. He has a history of precocious puberty. Examination reveals multiple café-au-lait spots. AP pelvis and frog lateral hip radiographs are shown in Figures 21a and 21b. His condition is linked to an abnormality in 1- G protein function. 2- osteoclastic function. 3- vitamin D metabolism. 4- sulfate transporter gene. 5- type I collagen formation. Preferred Response: 1 Recommended Reading(s): DiCaprio MR, Enneking WF: Fibrous dysplasia: Pathophysiology, evaluation, and treatment. J Bone Joint Surg Am 2005;87:1848-1864. Parekh SG, Donthineni-Rao R, Ricchetti E, et al: Fibrous dysplasia. J Am Acad Orthop Surg 2004;12:305-313.

Page 17: 2009 oite review

Question #: 22 What is the incidence of full-thickness rotator cuff tears in patients undergoing arthroplasty for the treatment of primary glenohumeral osteoarthritis? 1- 1% to 2% 2- 5% to 10% 3- 10% to 15% 4- 15% to 20% 5- 20% to 25% Preferred Response: 2 Recommended Reading(s): Edwards TB, Boulahia A, Kempf JF, et al: The influence of rotator cuff disease on the results of shoulder arthroplasty for primary osteoarthritis: Results of a multicenter study. J Bone Joint Surg Am 2002;84:2240-2248. Norris TR, Iannotti JP: Functional outcome after shoulder arthroplasty for primary osteoarthritis: A multicenter study. J Shoulder Elbow Surg 2002;11:130-135. Question #: 23 In children with Morquio syndrome, atlantoaxial instability is most commonly the result of 1- hypoplasia of the odontoid. 2- failure of fusion of the base of the odontoid. 3- incompetence of the transverse ligament. 4- generalized joint hypermobility. 5- erosion of the odontoid base due to synovial hypertrophy. Preferred Response: 1 Recommended Reading(s): Copley LA, Dormans JP: Cervical spine disorders in infants and children. J Am Acad Orthop Surg 1998;6:204-214. Stevens JM, Kendall BE, Crockard HA: The odontoid process in Morquio-Brailsford’s disease: The effects of occipitocervical fusion. J Bone Joint Surg Br 1991;73:851-858.

Page 18: 2009 oite review

Question #: 24 A 51-year-old woman has had left hand numbness for the past 2 years. Examination reveals decreased sensation over the fifth metacarpal region and little finger, both palmar and dorsal. There is no loss of abduction/adduction strength. Brace treatment has failed to provide relief, and neurodiagnostic studies reveal ulnar nerve compression with increased latency. The next step in treatment is ulnar nerve decompression at the 1- wrist only. 2- wrist and the elbow without transposition. 3- wrist and the elbow with anterior submuscular transposition. 4- elbow only without transposition. 5- elbow only with anterior submuscular transposition. Preferred Response: 4 Recommended Reading(s): Zlowodzki M, Chan S, Bhandari M, et al: Anterior transposition compared with simple decompression for treatment of cubital tunnel syndrome: A meta-analysis of randomized, controlled trials. J Bone Joint Surg Am 2007;89:2591-2598. Bartels RH, Verhagen WI, van der Wilt GJ, et al: Prospective randomized controlled study comparing simple decompression versus anterior subcutaneous transposition for idiopathic neuropathy of the ulnar nerve at the elbow: Part 1. Neurosurgery 2005;56:522-530. Nabhan A, Ahlhelm F, Kelm J, et al: Simple decompression or subcutaneous anterior transposition of the ulnar nerve for cubital tunnel syndrome. J Hand Surg Br 2005;30:521-524. Question #: 25 During the Henry (volar) approach for a junction mid and upper third diaphyseal radial shaft fracture, what nerve is most at risk for injury? 1- Ulnar 2- Radial 3- Median 4- Anterior interosseous 5- Posterior interosseous Preferred Response: 5 Recommended Reading(s): Hoppenfeld S, deBoer P: Surgical Exposures in Orthopaedics. Philadelphia, PA, Lippincott Williams & Wilkins, 1984, pp 58-63. Martini FH, Timmons MJ, Tallitsch RB: Human Anatomy, ed 5. San Francisco, CA, Pearson/Benjamin Cummings, 2006, pp 197-199.

Page 19: 2009 oite review

Question #: 26 What is the most common reason for failure of anterior cruciate ligament reconstruction? 1- Malposition of the bone tunnels 2- Medial meniscus deficiency 3- Lateral meniscus deficiency 4- Improper graft selection 5- Articular surface damage Preferred Response: 1 Recommended Reading(s): Battaglia TC, Miller MD: Management of bony deficiency in revision anterior cruciate ligament reconstruction using allograft bone dowels: Surgical technique. Arthroscopy 2005;21:767. Grossman MG, ElAttrache NS, Shields CL, et al: Revision anterior cruciate ligament reconstruction: Three- to nine-year follow-up. Arthroscopy 2005;21:418-423.

Page 20: 2009 oite review

Figure 27a Figure 27b Question #: 27 A 75-year-old woman has a slow growing mass in her thigh. She reports difficulty walking because of the increasing size of the mass. Chest imaging shows no evidence of metastatic disease. A radiograph and a T1-weighted MRI scan are shown in Figures 27a and 27b. What is the next step in management? 1- Biopsy 2- Surgical excision 3- Preoperative radiation therapy 4- Preoperative chemotherapy 5- Treatment with pain medication, physical therapy, and follow-up in 12 months Preferred Response: 2 Recommended Reading(s): Schwartz HS (ed): Orthopaedic Knowledge Update: Musculoskeletal Tumors 2. Rosemont, IL, American Academy of Orthopaedic Surgeons, 2007, pp 277-287. Dalal KM, Antonescu CR, Singer S: Diagnosis and management of lipomatous tumors. J Surg Oncol 2008;97:298-313.

Page 21: 2009 oite review

Question #: 28 A patient who underwent intramedullary nailing of a femoral shaft fracture 2 weeks ago now reports hip pain that radiates to the knee. What is the next most appropriate step in management? 1- Reassure the patient that the pain will improve and order physical therapy. 2- Review the radiographic report from the time of injury. 3- Obtain an AP and lateral view of the hip. 4- Obtain an MRI of the lumbar spine. 5- Obtain lumbar spine radiographs. Preferred Response: 3 Recommended Reading(s): Tornetta P III, Kain MS, Creevy WR: Diagnosis of femoral neck fractures in patients with a femoral shaft fracture: Improvement with a standard protocol. J Bone Joint Surg Am 2007;89:39-43. Daffner RH, Riemer BL, Butterfield SL: Ipsilateral femoral neck and shaft fractures: An overlooked association. Skeletal Radiol 1991;20:251-254.

Page 22: 2009 oite review

Figure 29a Figure 29b

Figure 29c

Page 23: 2009 oite review

Question #: 29 A 2½-year-old boy is examined for flat feet. Examination reveals bilateral flat feet, with the left side affected more than the right. The arch on his right foot is restored when he stands on his toes or is sitting. The left foot remains flat when standing on his toes or sitting. AP, lateral, and plantar flexion lateral radiographs of the left foot are shown in Figures 29a through 29c. Treatment of the left foot should consist of 1- triple arthrodesis. 2- serial casting followed by application of a foot abduction orthosis. 3- a supramalleolar orthosis. 4- surgical correction of the midfoot and heel cord lengthening. 5- talectomy. Preferred Response: 4 Recommended Reading(s): Sullivan JA: Pediatric flatfoot: Evaluation and management. J Am Acad Orthop Surg 1999;7:44-53. Drennan JC: Congenital vertical talus. Instr Course Lect 1996;45:315-322.

Page 24: 2009 oite review

Figure 30 Question #: 30 What anatomic structure is at the tip of the arrow in Figure 30? 1- Anterior sacral artery 2- S1 nerve root 3- L5 nerve root 4- Iliac vein 5- Sympathetic chain Preferred Response: 3 Recommended Reading(s): Louis R: Fusion of the lumbar and sacral spine by internal fixation with screw plates. Clin Orthop Relat Res 1986;203:18-33. Aylwin A, Saifuddin A, Tucker S: L5 radiculopathy due to sacral stress fracture. Skeletal Radiol 2003;32:590-593.

Page 25: 2009 oite review

Figure 31a Figure 31b

Figure 31c

Page 26: 2009 oite review

Question #: 31 A 48-year-old woman fell onto her outstretched arm 8 weeks ago. Radiographs at the time of injury revealed a comminuted radial head fracture and no dislocation. She was initially treated with early mobilization. Examination reveals no medial-sided tenderness or bruising and no wrist pain. She has a persistent block to supination and extension. CT scans are shown in Figures 31a through 31c. Treatment at this time should consist of 1- radial head arthroplasty. 2- resection of the radial head. 3- open reduction and internal fixation of the radial head. 4- manipulation under anesthesia with an intra-articular steroid injection. 5- arthroscopic debridement of the elbow. Preferred Response: 2 Recommended Reading(s): Herbertsson P, Josefsson PO, Hasserius R, et al: Uncomplicated Mason type-II and III fractures of the radial head and neck in adults: A long-term follow-up study. J Bone Joint Surg Am 2004;86:569-574. Jackson JD, Steinmann SP: Radial head fractures. Hand Clin 2007;23:185-193.

Page 27: 2009 oite review

Figure 32a Figure 32b Question #: 32 A 45-year-old man reports a 6-month history of lateral foot pain. Radiographs are shown in Figures 32a and 32b. What is the most appropriate orthotic device for this patient? 1- Solid ankle cushioned heel 2- Three-quarter length rigid insole with medial hindfoot and forefoot posting 3- Full-length rigid insole with medial forefoot posting 4- Full-length semi-rigid insole with medial hindfoot and forefoot posting 5- Full-length semi-rigid insole with a depression for the first ray and a lateral wedge Preferred Response: 5 Recommended Reading(s): Manoli A II, Graham B: The subtle cavus foot, “the underpronator.” Foot Ankle Int 2005;26:256-263. Janisse DJ, Janisse E: Shoe modification and the use of orthoses in the treatment of foot and ankle pathology. J Am Acad Orthop Surg 2008;16:152-158.

Page 28: 2009 oite review

Figure 33 Question #: 33 Figure 33 shows the radiograph of an 84-year-old woman who is seen in the emergency department with new onset severe right groin pain and inability to bear weight. She does not recall a fall but uses a walker and frequently stumbles. Laboratory studies, including a CBC, erythrocyte sedimentation rate, and C-reactive protein, are all normal. What is the most likely diagnosis? 1- Septic total hip 2- Metastasis to pelvis 3- Insufficiency fracture of the ramus 4- Primary bony malignancy 5- Loose acetabular component Preferred Response: 3 Recommended Reading(s): Buckwalter JA, Einhorn TA, Simon SR (eds): Orthopaedic Basic Science: Biology and Biomechanics of the Musculoskeletal System, ed 2. Rosemont, IL, American Academy of Orthopaedic Surgeons, 2000, pp 289-306. Vanderschot P: Treatment options of pelvic and acetabular fractures in patients with osteoporotic bone. Injury 2007;38:497-508.

Page 29: 2009 oite review

Question #: 34 Randomized controlled trials are considered to provide the highest level of evidence, and concealed treatment allocation is essential to prevent bias in this study design. Concealment is best achieved by using 1- even-odd days. 2- patient birth date. 3- randomization by surgeon. 4- patient hospital identification number. 5- opaque envelopes containing the randomization number. Preferred Response: 5 Recommended Reading(s): Einhorn TA, O’Keefe RJ, Buckwalter JA (eds): Orthopaedic Basic Science: Foundations of Clinical Practice, ed 3. Rosemont, IL, American Academy of Orthopaedic Surgeons, 2007, pp 87-101. Poolman RW, Struijs PA, Krips R, et al: Reporting of outcomes in orthopaedic randomized trials: Does blinding of outcome assessors matter? J Bone Joint Surg Am 2007;89:550-558. Question #: 35 A 25-year-old competitive soccer player has chronic anterior knee pain and reports “sloppiness” since injuring it in a collision with another player 2 months ago. He missed several weeks of practice but has since attempted a return to play. Examination reveals no quadriceps atrophy, standing varus alignment of 8°, a posterior sag sign, 3+ posterior drawer, 2+ varus instability in extension, 3+ varus instability at 30°, and 20° increased prone external rotation at 30° and 90°. He walks with a varus thrust. What is the best treatment option? 1- High tibial osteotomy 2- Reconstruction of the posterior cruciate ligament (PCL) and repair of the posterolateral corner (PLC) 3- Reconstruction of the PCL 4- Reconstruction of the PCL and PLC 5- High tibial osteotomy and PCL/PLC reconstruction Preferred Response: 5 Recommended Reading(s): Garrick JG (ed): Orthopaedic Knowledge Update: Sports Medicine 3. Rosemont, IL, American Academy of Orthopaedic Surgeons, 2004, pp 183-197. Giffin JR, Vogrin TM, Zantop T, et al: Effects of increasing tibial slope on the biomechanics of the knee. Am J Sports Med 2004;32:376-382.

Page 30: 2009 oite review

Question #: 36 When assessing the location of a guide pin for minimal-open screw fixation of a transverse scaphoid waist fracture, which of the following locations provides the best biomechanical fixation? 1- Central axis of the proximal and distal fragments 2- Dorsal axis in the distal and proximal fragments 3- Volar axis in the distal and proximal fragments 4- Dorsal axis distally and volar axis in the proximal fragment 5- Volar axis distally and dorsal axis in the proximal fragment Preferred Response: 1 Recommended Reading(s): McCallister WV, Knight J, Kaliappan R, et al: Central placement of the screw in simulated fractures of the scaphoid waist: A biomechanical study. J Bone Joint Surg Am 2003;85:72-77. Dodds SD, Panjabi MM, Slade JF III: Screw fixation of scaphoid fractures: A biomechanical assessment of screw length and screw augmentation. J Hand Surg Am 2006;31:405-413. Question #: 37 What is the most frequent variant of the relationship between the sciatic nerve and the piriformis tendon as the nerve exits the sciatic notch? 1- Entire nerve courses anterior to the piriformis muscle 2- Entire nerve courses posterior to the piriformis muscle 3- Entire nerve pierces and divides the piriformis muscle 4- Nerve divides and courses around the piriformis muscle 5- Nerve divides, with one division dividing the piriformis muscle Preferred Response: 1 Recommended Reading(s): Pokorny D, Jahoda D, Veigl D, et al: Topographic variations of the relationship of the sciatic nerve and the piriformis muscle and its relevance to palsy after total hip arthroplasty. Surg Radiol Anat 2006;28:88-91. Tornetta P III: Hip dislocations and fractures of the femoral head, in Bucholz RW, Heckman JD (eds): Rockwood and Green’s Fractures in Adults, ed 5. Philadelphia, PA, Lippincott Williams & Wilkins, 2001, p 1556. Beaton LE, Anson BJ: The relation of the sciatic nerve and of its subdivisions to the piriformis muscle. Anat Rec 1937;70:1-5.

Page 31: 2009 oite review

Figure 38a Figure 38b

Figure 38c Figure 38d

Page 32: 2009 oite review

Question #: 38 A 36-year-old woman has right knee pain and swelling that first began 4 months ago. Figures 38a through 38d show radiographs, MRI scans, and H & E histology slides at 20X and 100X. These findings are most consistent with what diagnosis? 1- High-grade central osteosarcoma 2- Periosteal osteosarcoma 3- Juxta-articular chondroma 4- Osteochondroma 5- Bizarre parosteal osteochondromatous proliferation (Nora’s lesion) Preferred Response: 2 Recommended Reading(s): Campanacci M: Periosteal osteosarcoma, in Campanacci M (ed): Bone and soft tissue tumors, ed 2. New York, NY, Springer-Verlag Wien, 1999, pp 517-524. Weis L: Common malignant bone tumors, in Simon M, Springfield D (eds): Osteosarcoma: Surgery for Bone and Soft Tissue Tumors. Philadelphia, PA, Lippincott Williams & Wilkins, 1998, pp 265-274.

Page 33: 2009 oite review

Question #: 39 An 18-month-old boy is being evaluated for intoeing gait. Foot progression angles are -10°. Thigh-foot angles are -20° bilaterally. There is no metatarsus adductus. His height is in the 60th percentile for his age. What is the most appropriate treatment? 1- Observation 2- Denis Browne bar 3- Referral to physical therapy 4- Vitamin D, calcium, and phosphate levels 5- Bilateral knee-ankle-foot orthoses with a medial upright Preferred Response: 1 Recommended Reading(s): Lincoln TL, Suen PW: Common rotational variations in children. J Am Acad Orthop Surg 2003;11:312-320. Staheli LT: Rotational problems in children. Instr Course Lect 1994;43:199-209. Kling TF Jr, Hensinger RN: Angular and torsional deformities of the lower limbs in children. Clin Orthop Relat Res 1983;176:136-147. Question #: 40 Postoperative loss of reduction after iliosacral screw fixation of a posterior pelvic ring injury has been attributed to which of the following? 1- Use of a partially threaded screw 2- Vertical fracture pattern through the sacrum 3- Lateral compression fracture pattern 4- Open book fracture pattern 5- Initial non-anatomic reduction Preferred Response: 2 Recommended Reading(s): Fischgrund JS (ed): Orthopaedic Knowledge Update 9. Rosemont, IL, American Academy of Orthopaedic Surgeons, 2008, pp 389-397. Griffin DR, Starr AJ, Reinert CM, et al: Vertically unstable pelvic fractures fixed with percutaneous iliosacral screws: Does posterior injury pattern predict fixation failure? J Orthop Trauma 2006;20:S30-S36.

Page 34: 2009 oite review

Question #: 41 A 65-year-old man with a distal femoral low grade exostotic chondrosarcoma reports mild knee pain. Radiologic work-up reveals no metastases. The patient’s leg otherwise functions normally. Definitive treatment for this tumor consists of 1- wide excision only. 2- chemotherapy only. 3- chemotherapy followed by wide excision. 4- radiation therapy only. 5- radiation therapy followed by wide excision. Preferred Response: 1 Recommended Reading(s): Menendez LR (ed): Orthopaedic Knowledge Update: Musculoskeletal Tumors. Rosemont, IL, American Academy of Orthopaedic Surgeons, 2002, pp 187-202. Lee FY, Mankin HJ, Fondren G, et al: Chondrosarcoma of bone: An assessment of outcome. J Bone Joint Surg Am 1999;81:326-338. Question #: 42 Shoulder pathology in adult patients with juvenile idiopathic arthritis (formerly juvenile rheumatoid arthritis) is characterized by 1- above average bone density. 2- large bone size. 3- a superiorly migrated humeral head. 4- medialization of the humeral head due to glenoid bone loss. 5- significant posterior glenoid erosion. Preferred Response: 4 Recommended Reading(s): Thomas S, Price AJ, Sankey RA, et al: Shoulder hemiarthroplasty in patients with juvenile idiopathic arthritis. J Bone Joint Surg Br 2005;87:672-676. Jolles BM, Grosso P, Bogoch ER: Shoulder arthroplasty for patients with juvenile idiopathic arthritis. J Arthroplasty 2007;22:876-883.

Page 35: 2009 oite review

Question #: 43 An acutely injured quadriplegic patient undergoes an anterior diskectomy and fusion with plating at C6-7. Two days postoperatively the patient has sweating, a severe headache, flushing, and a blood pressure of 180/120 mm Hg. What is the most appropriate initial management? 1- IV administration of epinephrine 2- MRI of the cervical spine 3- Removal of the cervical plate 4- Irrigation and/or replacement of the indwelling urinary catheter 5- Placement of a lumbar subarachnoid drain Preferred Response: 4 Recommended Reading(s): Banovac K, Sherman AL: Spinal cord injury rehabilitation, in Herkowitz HN, Garfin SR, Eismont FJ, et al (eds): Rothman-Simeone The Spine, ed 5. Philadelphia, PA, Saunders Elsevier, 2006, pp 1220-1231. Furlan JC, Fehlings MG: Cardiovascular complications after acute spinal cord injury: Pathophysiology, diagnosis, and managment. Neurosurg Focus 2008;25:E13.

Page 36: 2009 oite review

Figure 44a Figure 44b Question #: 44 The clinical test shown in Figures 44a and 44b, in which the hip is first flexed, abducted, and externally rotated, and then is extended and allowed to fall into adduction and internal rotation, tests for contractures of what muscle? 1- Sartorius 2- Pectineus 3- Gluteus medius 4- Rectus femoris 5- Tensor fascia lata Preferred Response: 5 Recommended Reading(s): Herring JA: The orthopaedic examination: A comprehensive overview, in Herring JA (ed): Tachdjian’s Pediatric Orthopaedics, ed 4. Philadelphia, PA, WB Saunders, 2008, pp 27-53. Gross R: Physical examination, in Staheli LT (ed): Pediatric Orthopaedic Secrets, ed 2. Philadelphia, PA, Hanley Belfus, 2003, p 15.

Page 37: 2009 oite review

Question #: 45 In synovial fluid analysis before revision total knee arthroplasty, what is the minimum threshold for white blood cell count that is considered strongly indicative of infection? 1- 100 2- 500 3- 2,500 4- 10,000 5- 25,000 Preferred Response: 3 Recommended Reading(s): Mason JB, Fehring TK, Odum SM, et al: The value of white blood cell counts before revision total knee arthroplasty. J Arthroplasty 2003;18:1038-1043. Parvizi J, Ghanem E, Menashe S, et al: Periprosthetic infection: What are the diagnostic challenges? J Bone Joint Surg Am 2006;88:138-147. Schinsky MF, Della Valle CJ, Sporer SM, et al: Perioperative testing for joint infection in patients undergoing revision total hip arthroplasty. J Bone Joint Surg Am 2008;90:1869-1875. Question #: 46 A patient has a both bone forearm fracture. After open reduction and internal fixation with modern plating techniques, addition of a bone graft to the radius is indicated if there is which of the following findings? 1- Forty percent comminution of the circumference of the radius 2- Segmental fracture of the ulna 3- Segmental bone loss of the radius 4- Open radius and ulna fractures 5- Open radius fracture Preferred Response: 3 Recommended Reading(s): Wright RR, Schmeling GJ, Schwab JP: The necessity of acute bone grafting in diaphyseal forearm fractures: A retrospective review. J Orthop Trauma 1997;11:288-294. Koval KJ (ed): Orthopaedic Knowledge Update 7. Rosemont, IL, American Academy of Orthopaedic Surgeons, 2002, pp 307-316. Moed BR, Kellam JF, Foster RJ, et al: Immediate internal fixation of open fractures of the diaphysis of the forearm. J Bone Joint Surg Am 1986;68:1008-1017.

Page 38: 2009 oite review

Question #: 47 Which of the following modes better defines corrosion resulting from an electrochemical potential created between two metals in contact and immersed in a conductive medium? 1- Crevice 2- Galvanic 3- Fretting 4- Degradation 5- Delamination Preferred Response: 2 Recommended Reading(s): Einhorn TA, O’Keefe RJ, Buckwalter JA (eds): Orthopaedic Basic Science: Foundations of Clinical Practice, ed 3. Rosemont, IL, American Academy of Orthopaedic Surgeons, 2007, pp 65-85. Mazzocca AD, Caputo AE, Browner BD, et al: Principles of internal fixation, in Browner BD, Jupiter JB, Levine AM (eds): Skeletal Trauma, ed 3. Philadelphia, PA, WB Saunders, 2003, pp 195-249.

Page 39: 2009 oite review

Figure 48 Question #: 48 A 4-year-old boy injures his finger in a lawn mower. A clinical photograph of the palmar side of the little finger is shown in Figure 48. With exam under sedation, the bone is noted to be covered by the subcutaneous tissue. To preserve length, contour, and sensation, treatment after irrigation and debridement should consist of 1- a thenar flap. 2- a V-Y advancement flap. 3- direct closure. 4- microvascular reattachment. 5- application of antibiotic ointment and a sterile dressing. Preferred Response: 5 Recommended Reading(s): Lamon RP, Cicero JJ, Frascone RJ, et al: Open treatment of fingertip amputations. Ann Emerg Med 1983;12:358-360. Söderberg T, Nyström A, Hallmans G, et al: Treatment of fingertip amputations with bone exposure: A comparative study between surgical and conservative treatment methods. Scand J Plast Reconstr Surg 1983;17:147-152. Farrell RG, Disher WA, Nesland RS, et al: Conservative management of fingertip amputations. JACEP 1977;6:243-246.

Page 40: 2009 oite review

Figure 49 Question #: 49 A 2-month-old boy is evaluated for short stature and severe clubfeet. Examination reveals swelling of the pinnae of the ears. The appearance of the hands and feet are shown in Figure 49. What is the most likely diagnosis? 1- Achondroplasia 2- Apert syndrome 3- Congenital syphilis 4- Diastrophic dysplasia 5- Cleidocranial dysplasia Preferred Response: 4 Recommended Reading(s): Sponseller PD: The skeletal dysplasias, in Morrissy RT, Weinstein SL (eds): Lovell and Winter’s Pediatric Orthopaedics, ed 5. Philadelphia, PA, Lippincott Williams & Wilkins, 2001, pp 243-285. Ryoppy S, Poussa M, Morikanto J, et al: Foot deformities in diastrophic dysplasia: An analysis of 102 patients. J Bone Joint Surg Br 1992;74:441-444.

Page 41: 2009 oite review

Figure 50a Figure 50b

Figure 50c Figure 50d

Page 42: 2009 oite review

Question #: 50 A 34-year-old woman reports pain in the tibia. Radiographs, a CT scan, and a biopsy specimen are shown in Figures 50a through 50d. What is the most appropriate treatment option? 1- Observation 2- External beam radiation 3- Radiofrequency ablation 4- Curettage with power burr and packing with cement 5- Wide resection and osteoarticular allograft reconstruction Preferred Response: 4 Recommended Reading(s): Turcotte RE: Giant cell tumor of bone. Orthop Clin North Am 2006;37:35-51. Bini SA, Gill K, Johnston JO, et al: Giant cell tumor of bone: Curettage and cement reconstruction. Clin Orthop Relat Res 1995;321:245-250. Question #: 51 A 63-year-old man with a history of renal cell carcinoma has had severe thoracic pain for the past month. Examination shows mild lower extremity weakness with positive Babinski. Radiographs show a lytic lesion within T9 and a localized gibbus deformity of 40° from T8-T10. What is the most appropriate management prior to surgery? 1- Radiation therapy 2- Chemotherapy 3- Chemotherapy and radiation therapy 4- Epidural corticosteroid placement 5- Intra-arterial embolization Preferred Response: 5 Recommended Reading(s): Prabhu VC, Bilsky MH, Jambhekar K, et al: Results of preoperative embolization for metastatic spinal neoplasms. J Neurosurg 2003;98:156-164. Heary RF, Bono CM: Metastatic spinal tumors. Neurosurg Focus 2001;11:e1.

Page 43: 2009 oite review

Question #: 52 In which of the following fracture patterns is the optimal orientation of the screws used to repair the medial malleolar fragment, parallel to the ankle joint? 1- Supination external rotation 2- Supination internal rotation 3- Pronation external rotation 4- Supination adduction 5- Pronation abduction Preferred Response: 4 Recommended Reading(s): Michelson JD: Ankle fractures resulting from rotational injuries. J Am Acad Orthop Surg 2003;11:403-412. Hak DJ, Lee MA: Ankle fractures: Open reduction internal fixation, in Wiss DA (ed): Master Techniques in Orthopaedic Surgery: Fractures, ed 2. Philadelphia, PA, Lippincott Williams & Wilkins, 2006, pp 551-567. Question #: 53 A 9-year-old boy with an L-4 level myelomeningocele is scheduled to undergo soft-tissue surgery for equinocavovarus feet. In addition to shunt clearance, what safety precautions should be taken in the perioperative period? 1- Latex-free environment 2- Avoidance of fiberglass casting material 3- Avoidance of intravenous narcotics 4- Hyperthermia prophylaxis 5- Preoperative echocardiogram Preferred Response: 1 Recommended Reading(s): Drennan JC: Current concepts in myelomeningocoele. Instr Course Lect 1999;48:543-550. Herring JA: Disorders of the spinal cord, in Herring JA (ed): Tachdjian’s Pediatric Orthopaedics, ed 4. Philadelphia, PA, WB Saunders, 2008, pp 1405-1482.

Page 44: 2009 oite review

Figure 54a Figure 54b Question #: 54 A 7-month-old infant has the findings shown in Figures 54a and 54b. Which of the following approaches to the surgical reconstruction is most appropriate? 1- The intrinsic tendon and collateral ligament insertions from the radial thumb should be preserved. 2- The ulnar thumb should be deleted with transfer of the extensor tendons. 3- The radial half of one thumb should be combined with the ulnar half of the other to create one thumb (Bilhaut-Cloquet procedure). 4- The radial digital nerves should be transferred into the ulnar thumb. 5- Brunner incisions across the palmar surfaces should be avoided. Preferred Response: 1 Recommended Reading(s): Ogino T, Ishii S, Takahata S, et al: Long-term results of surgical treatment of thumb polydactyly. J Hand Surg Am 1996;21:478-486. Baek GH, Gong HS, Chung MS, et al: Modified Bilhaut-Cloquet procedure for Wassel type-II and III polydactyly of the thumb. J Bone Joint Surg Am 2007;89:534-541.

Page 45: 2009 oite review

Question #: 55 Somitization of the axial skeleton has been shown to be highly dependent on what mechanism? 1- Sequential activation of homeobox genes 2- Notochord activation of sonic hedgehog 3- Formation of collagen 1 matrix in the ribosome 4- Dorsal sclerotome activation of MSX1 and MSX2 5- Alternation of activation of noggin and bone morphogenic proteins Preferred Response: 1 Recommended Reading(s): Iimura T, Pourquie PO: Hox genes in time and space during vertebrate body formation. Dev Growth Differ 2007;49:265-275. Turnpenny PD, Alman B, Cornier AS, et al: Abnormal vertebral segmentation and the notch signaling pathway in man. Dev Dyn 2007;236:1456-1474. Kappen C: Early and late functions of Hox genes in the development of the axial skeleton, in Buckwalter JA, Ehrlich MG, Sandell LJ, et al (eds): Skeletal growth and development: Clinical issues and basic science advances. Rosemont, IL, American Academy of Orthopaedic Surgeons, 1997, pp 147-162. Question #: 56 A 26-year-old man has a scaphoid waist fracture, and a decision is made for screw fixation through the dorsal approach (antegrade screw placement). Which of the following is considered the most reliable method to achieve proper screw seating below the subchondral bone? 1- Direct palpation 2- Direct visualization 3- Guide wire measurement 4- Static fluoroscopic imaging 5- Dynamic (live) fluoroscopic imaging Preferred Response: 2 Recommended Reading(s): Adamany DC, Mikola EA, Fraser BJ: Percutaneous fixation of the scaphoid through a dorsal approach: An anatomic study. J Hand Surg Am 2008;33:327-331. Tumilty JA, Squire DS: Unrecognized chondral penetration by a Herbert screw in the scaphoid. J Hand Surg Am 1996;21:66-68.

Page 46: 2009 oite review

Question #: 57 While trialing a cruciate-retaining total knee arthroplasty, the knee lacks 10° of extension. When flexed beyond 90°, the tibiofemoral contact point translates posterior and there is no lift-off of the tray trial with deep flexion. What is the next most appropriate step? 1- Increase the posterior slope of the tibia 2- Resect more tibia 3- Resect more distal femur 4- Recess the posterior cruciate ligament 5- Downsize the femoral compartment Preferred Response: 3 Recommended Reading(s): Fischgrund JS (ed): Orthopaedic Knowledge Update 9. Rosemont, IL, American Academy of Orthopaedic Surgeons, 2008, pp 457-471. Question #: 58 With the development of glenohumeral internal rotation deficit in a pitcher, the humeral head is shifted in what direction during the cocking phase of throwing? 1- Anteroinferior 2- Anterosuperior 3- Posterosuperior 4- Posteroinferior 5- No change in position occurs Preferred Response: 3 Recommended Reading(s): Grossman MG, Tibone JE, McGarry MH, et al: A cadaveric model of the throwing shoulder: A possible etiology of superior labrum anterior-to-posterior lesions. J Bone Joint Surg Am 2005;87:824-831. Lintner D, Mayol M, Uzodinma O, et al: Glenohumeral internal rotation deficits in professional pitchers enrolled in an internal rotation stretching program. Am J Sports Med 2007;35:617-621.

Page 47: 2009 oite review

Question #: 59 Paget’s disease of bone is associated with abnormal function of which of the following cell types? 1- Osteoblasts 2- Osteoclasts 3- Osteocytes 4- Histiocytes 5- Megakarocytes Preferred Response: 2 Recommended Reading(s): Klein GR, Parvizi J: Surgical manifestations of Paget’s disease. J Am Acad Orthop Surg 2006;14:577-586. Robey PG, Bianco P: The role of osteogenic cells in the pathophysiology of Paget’s disease. J Bone Miner Res 1999;14:9-16.

Page 48: 2009 oite review

Figure 60a Figure 60b

Figure 60c

Page 49: 2009 oite review

Question #: 60 A 55-year-old electrician who is 5’ 10” tall and weighs 250 lbs developed severe medial compartment degenerative arthritis 6 months ago and underwent a medial unicondylar knee arthroplasty. He did extremely well initially and returned to work after 5 weeks. He reports new onset pain made worse by prolonged weight bearing and stair climbing. He has no fevers, chills, or pain at rest. Radiographs are shown in Figures 60a and 60b. A delayed image from a bone scan is shown in Figure 60c. What is most likely diagnosis? 1- Infection 2- Anterior cruciate ligament disruption 3- Osteolysis 4- Stress fracture 5- Dislodgement of polyethylene Preferred Response: 4 Recommended Reading(s): Brumby SA, Carrington R, Zayontz S, et al: Tibial plateau stress fracture: A complication of unicompartmental knee arthroplasty using 4 guide pinholes. J Arthroplasty 2003;18:809-812. Pandit H, Murray DW, Dodd CA, et al: Medial tibial plateau fracture and the Oxford unicompartmental knee. Orthopedics 2007;30:28-31. Hamilton WG, Collier MB, Tarabee E, et al: Incidence and reasons for reoperation after minimally invasive unicompartmental knee arthroplasty. J Arthroplasty 2006;21:98-107.

Page 50: 2009 oite review

Figure 61a Figure 61b

Figure 61c

Page 51: 2009 oite review

Question #: 61 A 46-year-old female runner without foot deformity received a steroid injection into the second metatarsophalangeal (MTP) joint for forefoot pain 3 weeks prior to running a marathon. During the race, pain was noted beneath the second MTP joint. A clinical photograph and radiographs of her foot following the race are shown in Figures 61a through 61c. What is the most likely diagnosis? 1- Plantar fascia rupture 2- Second metatarsal stress fracture 3- Hammertoe deformity 4- Plantar plate rupture with subluxation of the second MTP joint 5- Rheumatoid arthritis Preferred Response: 4 Recommended Reading(s): McGarvey WC: Second metatarsophalangeal instability, in Nunley JA, Pfeffer GB, Sanders RW, et al (eds): Advanced Reconstruction of the Foot and Ankle. Rosemont, IL, American Academy of Orthopaedic Surgeons, 2004, pp 69-75. Mizel MS, Yodlowski ML: Disorders of the lesser metatarsophalangeal joints. J Am Acad Orthop Surg 1995;3:166-173. Question #: 62 The parents of a 5-year-old boy report that he had a right clubfoot corrected using the Ponseti method shortly after birth. They now note that he has been walking on the outside of his foot. Examination reveals the forefoot and hindfoot are well corrected. The ankle can be dorsiflexed 15°. When he walks, the foot supinates during swing phase and comes down on the lateral border during stance phase. What is the preferred management of this patient? 1- Reverse last shoes 2- Split posterior tibial tendon transfer 3- Full-time use of the Denis-Browne bar for 3 months 4- Anterior tibial tendon transfer to the lateral cuneiform 5- Percutaneous heel cord tenotomy and plantar fasciotomy Preferred Response: 4 Recommended Reading(s): Ponseti IV: Relapses, in Congenital Clubfoot: Fundamentals of Treatment. New York, NY, Oxford University Press, 1996, pp 98-106. Morcuende JA, Weinstein SL, Dietz FR, et al: Plaster cast treatment of clubfoot: The Ponseti method of manipulation and casting. J Pediatr Orthop Part B 1994;3:161-167.

Page 52: 2009 oite review

Question #: 63 The mechanism of action of bisphosphonates is largely dependent on 1- metabolites. 2- chemical structure. 3- route of administration. 4- synthetic characteristics. 5- bone binding properties. Preferred Response: 2 Recommended Reading(s): Einhorn TA, O’Keefe RJ, Buckwalter JA (eds): Orthopaedic Basic Science: Foundations of Clinical Practice, ed 3. Rosemont, IL, American Academy of Orthopaedic Surgeons, 2007, pp 315-330. Morris CD, Einhorn TA: Bisphosphonates in orthopaedic surgery. J Bone Joint Surg Am 2005;87:1609-1618. Question #: 64 What is the primary biomechanical role of the rotator cuff? 1- It provides the major force to move the arm forward. 2- It provides the majority of humeral rotation power. 3- It initiates arm elevation before the trapezius, serratus anterior, and deltoid are activated. 4- It compresses the humeral head against the glenoid. 5- It is the major dynamic stabilizer in the extremes of motion. Preferred Response: 4 Recommended Reading(s): Garrick JG (ed): Orthopaedic Knowledge Update: Sports Medicine 3. Rosemont, IL, American Academy of Orthopaedic Surgeons, 2004, pp 79-88. Hirashima M, Kadota H, Sakurai S, et al: Sequential muscle activity and its functional role in the upper extremity and trunk during overarm throwing. J Sports Sci 2002;20:301-310.

Page 53: 2009 oite review

Question #: 65 Two-flap Z-plasty with 60° limbs provides how much lengthening of scar contracture? 1- 10% 2- 25% 3- 50% 4- 75% 5- 100% Preferred Response: 4 Recommended Reading(s): Hove CR, Williams EF III, Rodgers BJ: Z-plasty: A concise review. Facial Plast Surg 2001;17:289-294. Browne EZ, Pederson WC: Skin grafts and skin flaps, in Green DP, Hotchkiss RN, Pederson WC, et al (eds): Green’s Operative Hand Surgery, ed 5. Philadelphia, PA, Elsevier, 2005, pp 1651-1692.

Page 54: 2009 oite review

Figure 66a Figure 66b

Figure 66c Figure 66d

Page 55: 2009 oite review

Question #: 66 Figures 66a through 66d show the radiographs and MRI scans of a 68-year-old woman who reports a 1-year history of worsening low back and left leg pain. Currently she is unable to walk more than two blocks without resting. She is able to complete her grocery shopping by leaning on the cart while in the store. She has a normal neurologic exam and no significant medical comorbidities. She has failed to respond to epidural steroids and physical therapy. Which of the following treatment options offers the most likely chance to relieve her pain symptoms? 1- Anterior diskectomy and interbody fusion at L4/5 2- Left-sided laminotomy and diskectomy at L4/5 3- Complete laminectomy at L4 with partial facetectomies and foraminotomies at L4/5 4- A series of three transforaminal epidural steroid injections at L4/5 5- Laminectomy, partial facetectomy, and foraminotomy at L4/5 with posterior spinal fusion Preferred Response: 5 Recommended Reading(s): Weinstein JN, Lurie JD, Tosteson TD, et al: Surgical versus nonsurgical treatment for lumbar degenerative spondylolisthesis. N Engl J Med 2007;356:2257-2270. Knaub MA, Won DS, McGuire R, et al: Lumbar spinal stenosis: Indications for arthrodesis and spinal instrumentation. Instr Course Lect 2005;54:313-319. Herkowitz HN, Kurz LT: Degenerative lumbar spondylolisthesis with spinal stenosis: A prospective study comparing decompression with decompression and intertransverse process arthrodesis. J Bone Joint Surg Am 1991;73:802-808.

Page 56: 2009 oite review

Question #: 67 A 7-year-old girl has a mass on the volar radial aspect of her wrist. The mass is 1 cm in diameter, firm, and not fixed to the skin. A decision is made for excision. Before doing this, what two clinical tests should be done? 1- Adams test and Steinberg sign 2- Stagnara and Froment tests 3- Transillumination and Allen test 4- Diadochokinesia and Phalen tests 5- Digital capillary refill and blood pressure Preferred Response: 3 Recommended Reading(s): Herring JA: Kyphosis, in Herring JA (ed): Tachdjian’s Pediatric Orthopaedics, ed 4. Philadelphia, PA, WB Saunders, 2008, pp 413-441. Wang AA, Hutchinson DT: Longitudinal observation of pediatric hand and wrist ganglia. J Hand Surg Am 2001;26:599-602. Question #: 68 Osteogenesis imperfecta results from a mutation in which of the following genes? 1- Type I collagen 2- Type II collagen 3- Type IV collagen 4- Type VI collagen 5- Type X collagen Preferred Response: 1 Recommended Reading(s): Einhorn TA, O’Keefe RJ, Buckwalter JA (eds): Orthopaedic Basic Science: Foundations of Clinical Practice, ed 3. Rosemont, IL, American Academy of Orthopaedic Surgeons, 2007, pp 25-47. Fischgrund JS (ed): Orthopaedic Knowledge Update 9. Rosemont, IL, American Academy of Orthopaedic Surgeons, 2008, pp 773-783.

Page 57: 2009 oite review

Figure 69a Figure 69b Question #: 69 A 56-year-old man has a thigh mass. He has no significant history of trauma or subcutaneous ecchymosis and is not taking anticoagulation medication. The MRI scans shown in Figures 69a and 69b confirm that this is a deep blood-filled cyst-like structure with enhancement of the cyst wall. Aspiration is consistent with old hematoma but it recurs after aspiration. What is the next most appropriate treatment option? 1- Ice, compression, and elevation 2- Open biopsy 3- Percutaneous drain placement, multiple if necessary 4- Irrigation, debridement, and placement of several drains 5- Resection of the entire mass including the cyst wall Preferred Response: 2 Recommended Reading(s): Ward WG Sr, Rougraff B, Quinn R, et al: Tumors masquerading as hematomas. Clin Orthop Relat Res 2007;465:232-240. Schwartz HS (ed): Orthopaedic Knowledge Update: Musculoskeletal Tumors 2. Rosemont, IL, American Academy of Orthopaedic Surgeons, 2007, pp 59-72.

Page 58: 2009 oite review

Question #: 70 Following open reduction and internal fixation of an ankle fracture, what is the time frame for patients to return to normal automobile breaking time? 1- 3 weeks 2- 6 weeks 3- 9 weeks 4- 12 weeks 5- 24 weeks Preferred Response: 3 Recommended Reading(s): Egol KA, Sheikhazadeh A, Mogatederi S, et al: Lower-extremity function for driving an automobile after operative treatment of ankle fracture. J Bone Joint Surg Am 2003;85:1185-1189. Egol KA, Sheikhazadeh A, Koval KJ: Braking function after complex lower extremity trauma. J Trauma 2008;65:1435-1438.

Page 59: 2009 oite review

Figure 71a Figure 71b

Figure 71c

Page 60: 2009 oite review

Question #: 71 A 20-year-old man has had progressive right buttock pain. AP pelvis and lateral hip radiographs, and an MRI scan are shown in Figures 71a through 71c. He reports that he has had several “bone spurs” removed in the past in his wrist and ankles. Genotype analysis demonstrates EXT 1 expression, which implies 1- no clinical significance. 2- less severe involvement than EXT 2. 3- less likelihood of subsequent familial transmission. 4- a high likelihood of local recurrence after removal. 5- a higher risk for sarcoma development than EXT 2. Preferred Response: 5 Recommended Reading(s): Porter DE, Lonie L, Fraser M, et al: Severity of disease and risk of malignant change in hereditary multiple exostoses: A genotype-phenotype study. J Bone Joint Surg Br 2004;86:1041-1046. Alvarez C, Tredwell S, De Vera M, et al: The genotype-phenotype correlation of hereditary multiple exostoses. Clin Genet 2006;70:122-130. Question #: 72 A baseball pitcher reports posterolateral shoulder pain and lateral shoulder paresthesias when in the cocking position of throwing. What is the most likely diagnosis? 1- SLAP lesion 2- Anterior instability 3- Internal impingement 4- Posterior capsule tightness 5- Quadrilateral space syndrome Preferred Response: 5 Recommended Reading(s): Fischgrund JS (ed): Orthopaedic Knowledge Update 9. Rosemont, IL, American Academy of Orthopaedic Surgeons, 2008, pp 273-285. DeLee JC, Drez D Jr, Miller MD (eds): Orthopaedic Sports Medicine, ed 2. Philadelphia, PA, WB Saunders, 2002, p 1247.

Page 61: 2009 oite review

Figure 73 Question #: 73 A 35-year-old man is seen in the emergency department with the injury shown in Figure 73. Reduction of this fracture should be performed urgently to prevent which of the following complications? 1- Subtalar arthrosis 2- Necrosis of the posterior skin 3- Malunion 4- Nonunion 5- Achilles tendon contracture Preferred Response: 2 Recommended Reading(s): Fitzgibbons TC, McMullen ST: Fractures and dislocations of the calcaneus in Heckman J, Bucholz RW (eds): Rockwood and Green’s Fractures in Adults, ed 5. New York, NY, Lippincott Williams & Wilkins, 2001, pp 2133-2174. Sanders R: Fractures and fracture-dislocations of the calcaneus, in Coughlin MJ, Mann RA (eds): Surgery of the Foot and Ankle, ed 7. St Louis, MO, Mosby, 1999, pp 1422-1464.

Page 62: 2009 oite review

Question #: 74 McCune-Albright syndrome (polyostotic fibrous dysplasia, café-au-lait spots, precocious puberty) is caused by a mutation in which of the following genes? 1- MAP kinase 2- Gs G-protein 3- Adenylyl cyclase 4- c-fos proto-oncogene 5- Parathyroid hormone receptor Preferred Response: 2 Recommended Reading(s): Einhorn TA, O’Keefe RJ, Buckwalter JA (eds): Orthopaedic Basic Science: Foundations of Clinical Practice, ed 3. Rosemont, IL, American Academy of Orthopaedic Surgeons, 2007, pp 25-47. Weinstein LS: G(s)alpha mutations in fibrous dysplasia and McCune-Albright syndrome. J Bone Miner Res 2006;21:P120-P124. Question #: 75 The Emergency Medical Treatment and Active Labor Act (EMTALA) requires that patients admitted to an emergency room cannot be transferred unless evaluated by a responsible health care provider and 1- they are screened for appropriate insurance coverage. 2- blood transfusions and other medication infusions are completed. 3- all emergent surgery is performed. 4- the benefits of transfer outweigh the risks. 5- transportation (air or ground) is less than 60 minutes. Preferred Response: 4 Recommended Reading(s): Fishgrund JS (ed): Orthopaedic Knowledge Update 9. Rosemont, IL, American Academy of Orthopaedic Surgeons, 2008, pp 3-12. Koval KJ, Tingey CW, Spratt KF: Are patients being transfered to level-1 trauma centers for reasons other than medical necessity? J Bone Joint Surg Am 2006;88:2124-2132.

Page 63: 2009 oite review

Question #: 76 Which of the following structures blocks successful closed reduction of a lateral subtalar dislocation? 1- Spring ligament 2- Deltoid ligament 3- Extensor digitorum brevis 4- Posterior tibial tendon 5- Extensor hallucis longus tendon Preferred Response: 4 Recommended Reading(s): Fischgrund JS (ed): Orthopaedic Knowledge Update 9. Rosemont, IL, American Academy of Orthopaedic Surgeons, 2008, pp 493-509. Bibbo C, Anderson RB, Davis WH: Injury characteristics and the clinical outcome of subtalar dislocations: A clinical and radiographic analysis of 25 cases. Foot Ankle Int 2003;24:158-163.

Page 64: 2009 oite review

Figure 77a Figure 77b Question #: 77 A 12-year-old boy sustains an injury to his knee while playing football. He has immediate swelling and is unable to walk on the limb. Examination reveals he is unable to do a straight leg raise and has marked swelling anteriorly. A lateral radiograph and MRI scan are seen in Figures 77a and 77b. What is the most appropriate treatment? 1- Reconstruction of the patellar tendon 2- Surgical reduction and internal fixation 3- Immobilization in a straight cast for 6 weeks 4- Open biopsy of the mass 5- Application of a range-of-motion brace limiting flexion to 45° for 6 weeks Preferred Response: 2 Recommended Reading(s): Zionts LE: Fractures around the knee in children. J Am Acad Orthop Surg 2002;10:345-355. Sponseller PD, Stanitski CL: Fractures and dislocations about the knee, in Beaty JH, Kasser JR (eds): Rockwood and Wilkins’ Fractures in Children, ed 5. Philadelphia, PA, Lippincott Williams & Wilkins, 2001, pp 1029-1033.

Page 65: 2009 oite review

Question #: 78 Most studies of cemented Charnley all-polyethylene acetabular shells showed a less than 5% failure rate at 10 years. What is the failure rate (revision rate) of these cups at 20 years? 1- Less than 10% 2- 15% to 20% 3- 30% to 40% 4- 60% to 70% 5- Greater than 90% Preferred Response: 2 Recommended Reading(s): Della Valle CJ, Kaplan K, Jazrawi A, et al: Primary total hip arthroplasty with a flanged cemented all-polyethylene acetabular component: Evaluation at a minimum of 20 years. J Arthroplasty 2004;19:23-26. Kavanagh BF, Wallrichs S, Dewitz M, et al: Charnley low-friction arthroplasty of the hip: Twenty-year results with cement. J Arthroplasty 1994;9:229-234. Callaghan JJ, Templeton JE, Liu SS, et al: Results of Charnley total hip arthroplasty at a minimum of thirty years: A concise follow-up of a previous report. J Bone Joint Surg Am 2004;86:690-695. Question #: 79 Which of the following individuals is considered the most ideal patient for total hip arthroplasty following a femoral neck fracture? 1- Healthy 31-year-old man with a displaced vertically oriented fracture 2- 61-year-old woman with a nondisplaced stress fracture 3- 70-year-old avid golfer 4- 81-year-old man with Parkinson’s disease 5- 93-year-old woman with dementia Preferred Response: 3 Recommended Reading(s): Blomfeldt R, Tornkvist H, Ponzer S, et al: Displaced femoral neck fracture: Comparison of primary total hip replacement with secondary replacement after failed internal fixation: A 2-year follow-up of 84 patients. Acta Orthop 2006;77:638-643. Blomfeldt R, Tornkvist H, Eriksson K, et al: A randomized controlled trial comparing bipolar hemiarthroplasty with total hip replacement for displaced intracapsular fractures of the femoral neck in elderly patients. J Bone Joint Surg Br 2007;89:160-165.

Page 66: 2009 oite review

Question #: 80 A 25-year-old woman has a Boutonniere deformity of the ring finger. Which of the following mechanisms is the most likely cause of this deformity? 1- Chronic mallet finger 2- Old avulsion fracture of the central slip insertion 3- Rupture of the flexor digitorum superficialis 4- Sagittal band rupture at the metacarpophalangeal joint 5- Loss of continuity of the volar plate of the proximal interphalangeal joint Preferred Response: 2 Recommended Reading(s): Tuttle HG, Olvey SP, Stern PJ: Tendon avulsion injuries of the distal phalanx. Clin Orthop Relat Res 2006;445:157-168. Lilly SI, Messer TM: Complications after treatment of flexor tendon injuries. J Am Acad Orthop Surg 2006;14:387-396. Imatami J, Hashizume H, Wake H, et al: The central slip attachment fracture. J Hand Surg Br 1997;22:107-109. Burton RI, Melchihor JA: Extensor tendons - late reconstruction, in Green DP, Hotchkiss RN, Pederson WC (eds): Green’s Operative Hand Surgery, ed 4. New York, NY, Churchill Livingstone, 1999, pp 1988-2019.

Page 67: 2009 oite review

Figure 81a Figure 81b Question #: 81 A 5-year-old boy fell in the park and sustained a closed injury to the elbow. He was initially treated with closed reduction and casting. Postreduction AP and lateral radiographs are shown in Figures 81a and 81b. At the 1-week follow-up visit, the radial head is noted to be dislocated. What is the next most appropriate step in treatment? 1- Closed reduction of the radial head and closed reduction of the ulna, with pin fixation of the ulna 2- Closed reduction of the ulna with open reduction of the radiocapitellar joint and reconstruction of the annular ligament 3- Closed reduction of the ulna and closed reduction of the radial head with pin fixation of the radiocapitellar joint 4- Open reduction of the ulna and radiocapitellar joint, with reconstruction of the annular ligament 5- Application of a joint spanning external fixator on the elbow to maintain reduction Preferred Response: 1 Recommended Reading(s): Ring D, Jupiter JB, Waters PM: Monteggia fractures in children and adults. J Am Acad Orthop Surg 1998;6:215-224. Wilkins KE: Changes in the management of Monteggia fractures. J Pediatr Orthop 2002;22:548-554.

Page 68: 2009 oite review

Question #: 82 What is the central concept in rehabilitating an athlete during nonsurgical care of an isolated grade II posterior cruciate ligament tear? 1- The knee should be splinted in full extension for 6 weeks. 2- Early range of motion and quadriceps strengthening should start as soon as pain permits. 3- Open chain hamstring strengthening exercises should be initiated early. 4- Functional braces have a high rate of success in patients with persistent symptoms. 5- Patients should not return to sport until hamstring strength equals the contralateral side. Preferred Response: 2 Recommended Reading(s): Garrick JG (ed): Orthopaedic Knowledge Update: Sports Medicine 3. Rosemont, IL, American Academy of Orthopaedic Surgeons, 2004, pp 155-168. Margheritini F, Rihn J, Musahl V, et al: Posterior cruciate ligament injuries in the athlete: An anatomical, biomechanical and clinical review. Sports Med 2002;32:393-408.

Page 69: 2009 oite review

Figure 83a Figure 83b Question #: 83 Figures 83a and 83b show the MRI scans of a 52-year-old man with a history of metastatic lung adenocarcinoma. He reports weakness involving the right leg over the last week to the extent that he is no longer able to ambulate independently. His oncologist estimates that his life expectancy is between 6 months to 1 year. His pulmonary function is stable without a need for supplemental oxygen. What treatment option offers the best potential to restore and maintain the patient’s ambulatory status? 1- Directed radiation therapy alone 2- Surgical laminectomy followed by radiation therapy 3- Combined focused radiation therapy and chemotherapy 4- Receptor-directed chemotherapy based on an open biopsy 5- Circumferential neurologic decompression and surgical stabilization with postoperative radiation therapy Preferred Response: 5 Recommended Reading(s): Schmidt MH, Klimo P Jr, Vrionis FD: Metastatic spinal cord compression. J Natl Compr Canc Netw 2005;3:711-719. Klimo P Jr, Kestle JR, Schmidt MH: Clinical trials and evidence-based medicine for metastatic spine disease. Neurosurg Clin N Am 2004;15:549-564. Patchell RA, Tibbs PA, Regine WF, et al: Direct decompressive surgical resection in the treatment of spinal cord compression caused by metastatic cancer: A randomized trial. Lancet 2005;366:643-648.

Page 70: 2009 oite review

Question #: 84 Implantable direct current bone stimulators decrease osteoclast activity and increase osteoblast activity by which of the following mechanisms? 1- Reduces oxygen concentration and increases local tissue pH 2- Activates calmodulin 3- Activates fibroblast growth factor receptor-3 4- Activates transmembrane calcium translocation via voltage-gated channels 5- Creates nano motion and mechanical stimulation at the fracture site Preferred Response: 1 Recommended Reading(s): Einhorn TA, O’Keefe RJ, Buckwalter JA (eds): Orthopaedic Basic Science: Foundations of Clinical Practice, ed 3. Rosemont, IL, American Academy of Orthopaedic Surgeons, 2007, pp 331-348. Otter MW, McLeod KJ, Rubin CT: Effects of electromagnetic fields in experimental fracture repair. Clin Orthop Relat Res 1998;355S:S90-S104. Question #: 85 A 62-year-old man has advanced osteoarthritis of the shoulder. Examination reveals no atrophy of the infraspinatus and good external rotation strength. A CT scan shows the glenoid version to be -10°, and there is mild posterior subluxation of the humeral head. What is the most appropriate treatment? 1- Reverse total shoulder arthroplasty 2- Hemiarthroplasty 3- Arthroscopic capsular release 4- Total shoulder arthroplasty 5- Glenoid bone block Preferred Response: 4 Recommended Reading(s): Bryant D, Litchfield R, Sandow M, et al: A comparison of pain, strength, range of motion, and functional outcomes after hemiarthroplasty and total shoulder arthroplasty in patients with osteoarthritis of the shoulder: A systematic review and meta-analysis. J Bone Joint Surg Am 2005;87:1947-1956. Gartsman GM, Roddey TS, Hammerman SM: Shoulder arthroplasty with or without resurfacing of the glenoid in patients who have osteoarthritis. J Bone Joint Surg Am 2000;82:26-34.

Page 71: 2009 oite review

Question #: 86 Which of the following factors correlates most with a satisfactory clinical outcome when managing an intra-articular fracture of the proximal tibia? 1- Age 2- Type of graft substitute 3- Condylar widening 4- Maintenance of mechanical axis 5- Degree of residual articular incongruency Preferred Response: 4 Recommended Reading(s): Rademakers MV, Kerkhoffs GM, Sierevelt IN, et al: Operative treatment of 109 tibial plateau fractures: Five- to 27-year follow-up results. J Orthop Trauma 2007;21:5-10. Weigel DP, Marsh JL: High-energy fractures of the tibial plateau: Knee function after longer follow-up. J Bone Joint Surg Am 2002;84:1541-1551. Stevens DG, Beharry R, McKee MD, et al: The long-term functional outcome of operatively treated tibial plateau fractures. J Orthop Trauma 2001;15:312-320. Honkonen SE: Degenerative arthritis after tibial plateau fractures. J Orthop Trauma 1995;9:273-277. Honkonen SE: Indications for surgical treatment of tibial condyle fractures. Clin Orthop Relat Res 1994;302:199-205.

Page 72: 2009 oite review

Figure 87a Figure 87b Question #: 87 A 5-month-old girl with a dislocated left hip has been treated in a Pavlik harness full-time for the last 4 weeks. An ultrasound obtained with the patient in the harness is shown in Figures 87a and 87b. Based on these image findings, what is the next step in treatment? 1- Arthrogram, closed reduction, and hip spica casting 2- MRI arthrogram to further assess obstacles to reduction 3- Continued use of the Pavlik harness for 4 more weeks 4- Change to an Ilfeld splint for 3 more months of abduction splinting 5- Open reduction and innominate osteotomy Preferred Response: 1 Recommended Reading(s): Guille JT, Pizzutillo PD, MacEwen GD: Development dysplasia of the hip from birth to six months. J Am Acad Orthop Surg 2000;8:232-242. Mubarak S, Garfin S, Vance R, et al: Pitfalls in the use of the Pavlik harness for treatment of congenital dysplasia, subluxation, and dislocation of the hip. J Bone Joint Surg Am 1981;63:1239-1248. Weinstein SL, Mubarak SJ, Wenger DR: Developmental hip dysplasia and dislocation: Part II. Instr Course Lect 2004;53:531-542.

Page 73: 2009 oite review

Question #: 88 Resident work hour guidelines should increase patient safety by decreasing resident fatigue. However, there is concern that safety benefits may be offset by the need for changes in systems of patient care necessitated by these guidelines. Which of these changes has caused the most concern for patient safety? 1- Night float rotations 2- Urgent surgery done the next day instead of on call 3- Home call 4- Decreased continuity of care 5- Use of physician extenders instead of residents Preferred Response: 4 Recommended Reading(s): Friedlander GE: The 80-hour duty week: Rationale, early attitudes, and future questions. Clin Orthop Relat Res 2006;449:138-142. Horwitz LI, Moin T, Krumholz HM, et al: Consequences of inadequate sign-out for patient care. Arch Intern Med 2008;168:1755-1760. Okie S: An elusive balance: Residents’ work hours and the continuity of care. N Engl J Med 2007;356:2665-2667. Question #: 89 Following total knee arthroplasty with resurfacing of the patella, a patient has lateral subluxation of the patella. What issue with the components is a cause of this complication? 1- Lateral placement of the tibial tray 2- Reduced composite thickness of the patella 3- External rotation of the femoral component 4- Internal rotation of the tibial component 5- Posterior translation of the femoral component Preferred Response: 4 Recommended Reading(s): Malo M, Vince KG: The unstable patella after total knee arthroplasty: Etiology, prevention, and management. J Am Acad Orthop Surg 2003;11:364-371.

Page 74: 2009 oite review

Question #: 90 What anatomic structure inserts most anteriorly on the proximal fibula? 1- Sartorius 2- Iliotibial band 3- Biceps femoris 4- Popliteofibular ligament 5- Lateral collateral ligament Preferred Response: 5 Recommended Reading(s): LaPrade RF, Ly TV, Wentorf FA, et al: The posterolateral attachments of the knee: A qualitative and quantitative morphologic analysis of the fibular collateral ligament, popliteus tendon, popliteofibular ligament, and lateral gastrocnemius tendon. Am J Sports Med 2003;31:854-860. Stannard JP, Brown SL, Farris RC, et al: The posterolateral corner of the knee: Repair versus reconstruction. Am J Sports Med 2005;33:881-888.

Page 75: 2009 oite review

Figure 91 Question #: 91 What nerve is most at risk when placing the intramedullary device shown in Figure 91? 1- Lateral plantar 2- Medial plantar 3- Deep peroneal 4- Saphenous 5- Medial calcaneal Preferred Response: 1 Recommended Reading(s): Flock TJ, Ishikawa S, Hecht PJ, et al: Heel anatomy for retrograde tibiotalocalcaneal roddings: A roentgenographic and anatomic analysis. Foot Ankle Int 1997;8:233-235. Pochatko DJ, Smith JW, Phillips RA, et al: Anatomic structures at risk: Combined subtalar and ankle arthrodesis with a retrograde intramedullary rod. Foot Ankle Int 1995;16:542-547.

Page 76: 2009 oite review

Figure 92a Figure 92b

Figure 92c Figure 92d

Page 77: 2009 oite review

Question #: 92 A 67-year-old woman reports knee and calf pain after mild exercise, and states that the pain is relieved by rest. She notes that radiographs were obtained 5 years ago and again recently. Figure 92a is from 5 years prior. Figure 92b is current. Current CT scans of the distal femur are seen in Figures 92c and 92d. What is the next most appropriate step in treatment? 1- Vascular surgery consultation 2- Biopsy and radiofrequency ablation 3- Biopsy, neoadjuvant chemotherapy, and resection 4- Biopsy, curettage, and filling the void with methylmethacrylate 5- Distal femoral resection and metal endoprosthetic reconstruction Preferred Response: 1 Recommended Reading(s): Skeletal Lesions Interobserver Correlation Among Expert Diagnosticians (SLICED) Study group: Reliability of histopathologic and radiologic grading of cartilaginous neoplasms in long bones. J Bone Joint Surg Am 2007;89:2113-2123. Ryzewicz M, Manaster BJ, Naar E, et al: Low-grade cartilage tumors: Diagnosis and treatment. Orthopaedics 2007;30:35-46.

Page 78: 2009 oite review

Figure 93 Question #: 93 The injury shown in Figure 93 is associated with which of the following? 1- Child abuse 2- Folic acid deficiency 3- Osteogenesis imperfecta 4- Hypophosphatemic rickets 5- Generalized joint hypermobility Preferred Response: 1 Recommended Reading(s): DeLee JC, Wilkins KE, Rogers LF, et al: Fracture-separation of the distal humeral epiphysis. J Bone Joint Surg Am 1980;62:46-51. Akbarnia BA, Silberstein MJ, Rende RJ, et al: Arthrography in the diagnosis of fractures of the distal end of the humerus in infants. J Bone Joint Surg Am 1986;68:599-602.

Page 79: 2009 oite review

Question #: 94 What is the most common cause of hematogenous osteomyelitis in sickle cell disease? 1- Salmonella 2- Propionibacterium 3- Enterobacteriaceae 4- Bartonella henselae 5- Pseudomonas aeruginosa Preferred Response: 1 Recommended Reading(s): Fischgrund JS (ed): Orthopaedic Knowledge Update 9. Rosemont, IL, American Academy of Orthopaedic Surgeons, 2008, pp 301-318. Cornwall R, Dormans JP: Diseases of the hematopoietic system, in Morrissy RT, Weinstein SL (eds): Lovell and Winter’s Pediatric Orthopaedics. Philadelphia, PA, Lippincott Williams & Wilkins, 2006, pp 357-404. Question #: 95 Patients with multiple hereditary osteochondromas frequently have loss of forearm rotation and ulnar shortening. What procedure is most likely to improve forearm rotation in these patients? 1- Tendon transfer 2- Ulnar lengthening 3- Radial head excision 4- Distal ulnar resection 5- Simple excision of the osteochondroma Preferred Response: 5 Recommended Reading(s): Shin EK, Jones NF, Lawrence JF: Treatment of multiple hereditary osteochondromas of the forearm in children: A study of surgical procedures. J Bone Joint Surg Br 2006;88:255-260. Akita S, Murase T, Yonenobu K, et al: Long-term results of surgery for forearm deformities in patients with multiple cartilaginous exostoses. J Bone Joint Surg Am 2007;89:1993-1999.

Page 80: 2009 oite review

Question #: 96 Partial distal biceps tendon tears occur primarily on which side of the tuberosity footprint? 1- Radial 2- Ulnar 3- Proximal 4- Distal 5- Central Preferred Response: 1 Recommended Reading(s): Kelly EW, Steinmann S, O’Driscoll SW: Surgical treatment of partial distal biceps tendon ruptures through a single posterior incision. J Shoulder Elbow Surg 2003;12:456-461. Davis WM, Yassine Z: An etiological factor in tear of the distal tendon of the biceps brachii: Report of two cases. J Bone Joint Surg Am 1956;39:1365-1368. Question #: 97 What is the preferred treatment for a 50% laceration of the flexor tendons with triggering? 1- Core 4/0 suture repair 2- Core 4/0 suture repair with 6/0 peritendinous suture 3- Epitendinous repair with 4/0 suture 4- Epitendinous repair with 6/0 suture 5- Trimming of frayed edges Preferred Response: 5 Recommended Reading(s): Bishop AT, Cooney WP III, Wood MB: Treatment of partial flexor tendon lacerations: The effect of tenorrhaphy and early protected mobilization. J Trauma 1986;26:301-312. McGeorge DD, Stillwell JH: Partial flexor tendon injuries: To repair or not. J Hand Surg Br 1992;17:176-177.

Page 81: 2009 oite review

Figure 98a Figure 98b

Figure 98c Figure 98d

Page 82: 2009 oite review

Question #: 98 A 52-year-old man was involved in a motor vehicle accident and now reports severe low back pain and left anterior leg and foot paresthesias. Radiographs and CT scans are shown in Figures 98a through 98d. He has no lower extremity weakness. He is otherwise hemodynamically stable and neurologically intact. What is the most appropriate management? 1- Thoracolumbosacral orthosis and mobilization 2- Anterior vertebrectomy, vertebral body reconstruction, and plating 3- Posterior L5 laminectomy 4- Posterior L5 laminectomy and L4-S1 posterolateral fusion 5- Transpedicular reduction of intracanal fragments Preferred Response: 1 Recommended Reading(s): Butler JS, Fitzpatrick P, Ni Mhaolain AM, et al: The management and functional outcome of isolated burst fractures of the fifth lumbar vertebra. Spine 2007;32:443-447. Seybold EA, Sweeney, CA, Fredrickson BE, et al: Functional outcome of low lumbar burst fractures: A multicenter review of operative and nonoperative treatment of L3-L5. Spine 1999;24:2154-2161.

Page 83: 2009 oite review

Question #: 99 Which of the following arteries connect the obturator to the external iliac? 1- Corona mortis 2- Recurrent obturator 3- Circumflex magnus 4- Hypogastric branches of the deep internal iliac 5- Ascending pudendal communicating branches Preferred Response: 1 Recommended Reading(s): Tornetta P III, Hochwald N, Levine R: Corona mortis: Incidence and location. Clin Orthop Relat Res 1996;329:97-101. Letournel E, Judet R: Fractures of the Acetabulum, ed 2. New York, NY, Springer-Verlag, 1993, pp 375-381. Question #: 100 The most common cause of a malpractice suit against an orthopaedic surgeon involves treatment of which of the following conditions? 1- Femoral fracture 2- Humeral fractures with nerve palsy 3- Knee arthroplasty 4- Meniscal tear 5- Spinal stenosis Preferred Response: 1 Recommended Reading(s): Femur fracture care frequent cause of lawsuit. Managing Orthopaedic Malpractice Risk, ed 2. Rosemont, IL, American Academy of Orthopaedic Surgeons, 2001 Bulletin. http://www2.aaos.org/aaos/archives/bulletin/feb01/fline4.htm. Accessed on July 24, 2009.

Page 84: 2009 oite review

Figure 101a

Figure 101b Figure 101c

Page 85: 2009 oite review

Question #: 101 A 12-year-old boy has had pain in the right knee for the past 6 weeks. He has had two episodes of giving way but no locking. He denies any history of injury. Examination reveals no effusion, ligaments are stable, range of motion is full, and there is no localized tenderness. An AP radiograph and MRI scans are shown in Figures 101a through 101c. What is the most appropriate management? 1- Excision of the lesion 2- Retrograde drilling of the lesion 3- Antegrade drilling of the lesion 4- Arthroscopic reduction and fixation 5- Observation and limitation of activities Preferred Response: 5 Recommended Reading(s): Schenck RC Jr, Goodnight JM: Osteochondritis dissecans. J Bone Joint Surg Am 1996;78:439-456. Herring JA: Disorders of the knee, in Herring JA (ed): Tachdjian’s Pediatric Orthopaedics, ed 4. Philadelphia, PA, WB Saunders, 2008, pp 919-971. Question #: 102 What muscle groups are weakest after antegrade intramedullary nailing of a midshaft femoral fracture? 1- Hamstrings and abductors 2- Hamstrings and adductors 3- Quadriceps and abductors 4- Quadriceps and external rotators 5- Abductors and external rotators Preferred Response: 3 Recommended Reading(s): Kapp W, Lindsey RW, Noble PC, et al: Long-term residual musculoskeletal deficits after femoral shaft fractures treated with intramedullary nailing. J Trauma 2000;49:446-449. Archdeacon M, Ford KR, Wyrick J, et al: A prospective functional outcome and motion analysis evaluation of the hip abductors after femur fracture and antegrade nailing. J Orthop Trauma 2008;22:3-9.

Page 86: 2009 oite review

Question #: 103 Which of the following surgical approaches to the hip uses an internervous plane? 1- Posterolateral 2- Posterior lateral (Moore) 3- Anterior (Smith-Peterson) 4- Anterolateral (Watson-Jones) 5- Direct lateral (Hardinge) Preferred Response: 3 Recommended Reading(s): Barrack RL, Booth RE Jr, Lonner JH, et al (eds): Orthopaedic Knowledge Update: Hip and Knee Reconstruction 3. Rosemont, IL, American Academy of Orthopaedic Surgeons, 2006, pp 311-321. Hoppenfeld S, deBoer P: Surgical Exposures in Orthopaedics: The Anatomic Approach, ed 2. Philadelphia, PA, Lippincott Williams & Wilkins, 1994, pp 325-335. Question #: 104 Which of the following methods of antibiotic treatment has been associated with the highest success rate in diabetic patients treated nonsurgically for osteomyelitis of the foot? 1- Vancomycin IV antibiotic 2- Dual drug oral antibiotics 3- IV antibiotics based on a bone biopsy 4- IV antibiotics based on a swab of the open wound 5- IV antibiotics based on the hospital profile of the most common organisms Preferred Response: 3 Recommended Reading(s): Senneville E, Lombart A, Beltrand E, et al: Outcome of diabetic foot osteomyelitis treated nonsurgically: A retrospective cohort study. Diabetes Care 2008;31:637-642. Lipsky BA: Infectious problems of the foot in diabetic patients, in Bowker JH, Pfeifer MA (eds): Levin and O’Neal’s The Diabetic Foot, ed 6. St Louis, MO, Mosby, 2001, pp 467-480.

Page 87: 2009 oite review

Question #: 105 Which of the following is considered the most important portion of the kinetic chain in protecting the thrower’s elbow from valgus loads? 1- Forearm pronation 2- Scapular retraction 3- Scapular protraction 4- Glenohumeral internal rotation 5- Glenohumeral external rotation Preferred Response: 4 Recommended Reading(s): Garrick JG (ed): Orthopaedic Knowledge Update: Sports Medicine 3. Rosemont, IL, American Academy of Orthopaedic Surgeons, 2002, pp 101-111. Marshall RN, Elliott BC: Long-axis rotation: The missing link in proximal-to-distal sequencing. J Sports Sci 2000;18:247-254.

Page 88: 2009 oite review

Figure 106 Question #: 106 Figure 106 shows the MRI arthrogram of an 18-year-old football player who injured his shoulder. What physical examination finding is most likely to demonstrate his pathology? 1- Jerk test 2- Gerber lift-off test 3- O’Brien test 4- Sulcus sign 5- Apprehension sign Preferred Response: 1 Recommended Reading(s): Garrick JG (ed): Orthopaedic Knowledge Update: Sports Medicine 3. Rosemont, IL, American Academy of Orthopaedic Surgeons, 2004, pp 53-77. Millett PJ, Clavert P, Hatch GF III, et al: Recurrent posterior shoulder instability. J Am Acad Orthop Surg 2006;14:464-467.

Page 89: 2009 oite review

Question #: 107 Which of the following findings on microscopy best characterizes the pathologic features of lateral epicondylitis? 1- Giant cell infiltration 2- Cystic degeneration with Rice bodies and fatty infiltration 3- Localized hemorrhage with proliferation of neutrophils 4- Fibroblast hypertrophy, disorganized collagen, and vascular hyperplasia 5- Hypertrophic collagen fibers, mature fibroblasts, and frequent macrophages Preferred Response: 4 Recommended Reading(s): Kraushaar BS, Nirschl RP: Tendinosis of the elbow (tennis elbow): Clinical features and findings of histological, immunohistochemical, and electron microscopy studies. J Bone Joint Surg Am 1999;81:259-278. Trumble TE (ed): Hand Surgery Update 3: Hand, Elbow & Shoulder. Rosemont, IL, American Society for Surgery of the Hand, 2003, pp 271-284. Nirschl RP, Pettrone FA: Tennis elbow: The surgical treatment of lateral epicondylitis. J Bone Joint Surg Am 1979;61:832-839. Question #: 108 What method of terminal polyethylene sterilization results in the greatest number of remaining free radicals within the polyethylene? 1- Gas plasma 2- Ethylene oxide 3- Gamma irradiation 4- Gamma irradiation and annealing 5- Gamma irradiation and remelting Preferred Response: 3 Recommended Reading(s): Gordon AC, D’Lima DD, Colwell CW Jr: Highly cross-linked polyethylene in total hip arthroplasty. J Am Acad Orthop Surg 2006;14:511-523. McKellop H, Shen FW, Lu B, et al: Effect of sterilization method and other modifications on the wear resistance of acetabular cups made of ultra-high molecular weight polyethylene: A hip-simulator study. J Bone Joint Surg Am 2000;82:1708-1725.

Page 90: 2009 oite review

Question #: 109 Which of the following conditions has little or no published data to support the use of locked plating techniques? 1- Proximal intra-articular bicondylar tibia fracture 2- Proximal humeral fracture 3- Bicondylar distal femur fracture 4- A geriatric periprosthetic fracture 5- Hypertrophic humeral nonunion in a young male Preferred Response: 5 Recommended Reading(s): Haidukewych GJ, Ricci W: Locked plating in orthopaedic trauma: A clinical update. J Am Acad Orthop Surg 2008;16:347-355. Question #: 110 The halo vest is least effective at controlling which of the following spinal motions? 1- Lateral bend 2- Flexion 3- Extension 4- Axial rotation 5- Axial distraction Preferred Response: 5 Recommended Reading(s): Ivancic PC, Beauchman NN, Tweardy: Effect of halo-vest components on stabilizing the injured cervical spine. Spine 2009;34:167-175. Johnson RM, Hart DL, Simmons EF, et al: Cervical orthoses: A study comparing their effectiveness in restricting cervical motion in normal subjects. J Bone Joint Surg Am 1977;59:332-339.

Page 91: 2009 oite review

Figure 111a

Figure 111b Figure 111c

Page 92: 2009 oite review

Question #: 111 An 11-year-old girl has had neck pain for the past 6 months. She has had no neurologic symptoms and is neurologically normal on physical examination. Cervical spine range of motion is normal. An open-mouth view and flexion and extension radiographs are seen in Figures 111a through 111c. What is the most appropriate treatment? 1- Cessation of contact sports 2- Decompressive laminectomy 3- Anterior cervical spinal fusion 4- Posterior cervical spinal fusion 5- Anterior and posterior cervical spinal fusion Preferred Response: 4 Recommended Reading(s): Reilly CW, Choit RL: Transarticular screws in the management of C1-C2 instability in children. J Pediatr Orthop 2006;26:582-588. Sankar WN, Wills BP, Dormans JP, et al: Os odontoideum revisited: The case for a multifactorial etiology. Spine 2006;31:979-984. Question #: 112 A review of a patient’s AP pelvis and oblique (Judet) radiographs reveals that the iliopectineal line is intact, the ilioischial line is disrupted, and there is a fracture of the inferior pubic ramus. Based on these findings, what is the most likely acetabular fracture pattern? 1- Transverse 2- Transverse and posterior wall 3- Posterior wall 4- Posterior column 5- T-type Preferred Response: 4 Recommended Reading(s): Letournel E: Acetabulum fractures: Classification and management. Clin Orthop Relat Res 1980;151:81-106. Patel V, Day A, Dinah F, et al: The value of specific radiological features in the classification of acetabular fractures. J Bone Joint Surg Br 2007;89:72-76.

Page 93: 2009 oite review

Question #: 113 The most effective method for nonsurgical management of a chronic (symptoms lasting greater than 3 months) flexible acquired adult flatfoot deformity is with which of the following devices? 1- UCBL orthosis 2- Ankle-foot orthosis 3- Patellar tendon-bearing orthosis 4- Indepth shoe with total contact foot orthosis 5- Indepth shoe with extended medial counter and medial flare Preferred Response: 2 Recommended Reading(s): Alvarez, RG, Marini A, Schmitt C, et al: Stage I and II posterior tibial tendon dysfunction treated by a structured nonoperative management protocol: An orthosis and exercise program. Foot Ankle Int 2006;27:2-8. Augustin JF, Lin SS, Berberian WS, et al: Nonoperative treatment of the adult acquired flat foot with the Arizona brace. Foot Ankle Clin 2003;8:491-502. Chao W, Wapner KL, Lee TH, et al: Nonoperative management of posterior tibial tendon dysfunction. Foot Ankle Int 1996;17:736-741. Question #: 114 When performing a lateral column lengthening procedure with a posterior tibial tendon reconstruction for correction of an acquired adult flatfoot deformity, what procedure is also commonly required? 1- Talar neck osteotomy 2- Lengthening of the gastrocsoleus complex 3- First metatarsophalangeal joint fusion 4- Lateral ankle ligament reconstruction 5- Peroneus longus to brevis tendon transfer Preferred Response: 2 Recommended Reading(s): Myerson MS: Adult acquired flatfoot deformity: Treatment of dysfunction of the posterior tibial tendon. Instr Course Lect 1997;46:393-405. Pinney SJ, Lin SS: Current concepts review: Acquired adult flatfoot deformity. Foot Ankle Int 2006;27:66-75.

Page 94: 2009 oite review

Question #: 115 The amount of energy per volume that a material can absorb prior to failure defines which of the following terms? 1- Toughness 2- Failure point 3- Viscoelasticity 4- Fatigue strength 5- Young’s modulus Preferred Response: 1 Recommended Reading(s): Einhorn TA, O’Keefe RJ, Buckwalter JA (eds): Orthopaedic Basic Science: Foundations of Clinical Practice, ed 3. Rosemont, IL, American Academy of Orthopaedic Surgeons, 2007, pp 49-64. Chao EYS, Aro HT: Biomechanics of fracture fixation, in Mow VC, Hayes WC (eds): Basic Orthopaedic Biomechanics. New York, NY, Raven Press, 1991, pp 293-336. Question #: 116 Which of the following postoperative modalities to prevent deep venous thrombosis is associated with the highest risk of hematoma? 1- Aspirin 2- Dipyridamole 3- Compression device 4- Clopidogrel bisulfate 5- Low-molecular-weight heparin Preferred Response: 5 Recommended Reading(s): Dorr LD, Gendelman V, Maheshwari AV, et al: Multimodal thromboprophylaxis for total hip and knee arthroplasty based on risk assessment. J Bone Joint Surg Am 2007;89:2648-2657. Lee MC, Nickisch F, Limbird RS: Massive retroperitoneal hematoma during enoxaperan treatment of pulmonary embolism after primary total hip arthroplasty: Case-reports and review of the literature. J Arthoroplasty 2006;21:1209-1214.

Page 95: 2009 oite review

Figure 117a Figure 117b Figure 117c Question #: 117 A 4-year-old girl has bowleg deformities. Radiographs of her lower and upper extremities are shown in Figures 117a through 117c. She is below the 25th percentile for height. Which of the following laboratory findings are most likely to be present? 1- Normal serum Ca; low serum phosphate; elevated alkaline phosphatase 2- Low serum Ca; normal serum phosphate; elevated alkaline phosphatase 3- Low serum Ca; elevated serum phosphate; low alkaline phosphatase 4- Elevated serum Ca; low serum phosphate; low alkaline phosphatase 5- Elevated serum Ca; elevated serum phosphate; elevated alkaline phosphatase Preferred Response: 1 Recommended Reading(s): Loeffler RD Jr, Sherman FC: The effect of treatment on growth and deformity in hypophosphatemic vitamin D-resistant rickets. Clin Orthop Relat Res 1982;162:4-10. Ferris B, Walker C, Jackson A, et al: The orthopaedic management of hypophosphatemic rickets. J Pediatr Orthop 1991;11:367-373.

Page 96: 2009 oite review

Figure 118a Figure 118b Question #: 118 What plating strategy provides the most optimal fixation for the fracture seen in Figures 118a and 118b? 1- Lateral locking plate 2- Posteromedial plate 3- Lateral buttress plate 4- Anteromedial plate 5- Medial and lateral plates Preferred Response: 2 Recommended Reading(s): Georgiadis GM: Combined anterior and posterior approaches for complex tibial plateau fractures. J Bone Joint Surg Br 1994;76:285-289. Bhattacharyya T, McCarty LP III, Harris MB, et al: The posterior shearing tibial plateau fracture: Treatment and results via a posterior approach. J Orthop Trauma 2005;19:305-310.

Page 97: 2009 oite review

Question #: 119 In lateral epicondylitis, the area of tendinopathy is classically located at the origin of the 1- anconeus. 2- extensor digitorum. 3- extensor carpi ulnaris. 4- extensor carpi radialis longus. 5- extensor carpi radialis brevis. Preferred Response: 5 Recommended Reading(s): Garrick JG (ed): Orthopaedic Knowledge Update: Sports Medicine 3. Rosemont, IL, American Academy of Orthopaedic Surgeons, 2002, pp 91-99. Nirschl RP, Pettrone FA: Tennis elbow: The surgical treatment of lateral epicondylitis. J Bone Joint Surg Am 1979;61:832-839. Question #: 120 Warfarin is commonly used to prevent deep venous thrombosis after total hip arthroplasty. What is its mechanism of action? 1- Forms complexes with antithrombin III 2- Inactivates active thrombin and active factor Xa 3- Prevents conversion of fibrinogen to fibrin 4- Prevents gamma carboxylation in factor X and prothrombin 5- Prevents thromboxane A2 formation, interfering with platelet aggregation Preferred Response: 4 Recommended Reading(s): Einhorn TA, O’Keefe RJ, Buckwalter JA (eds): Orthopaedic Basic Science: Foundations of Clinical Practice, ed 3. Rosemont, IL, American Academy of Orthopaedic Surgeons, 2007, pp 395-413. Lieberman JR, Hsu WK: Prevention of venous thromboembolic disease after total hip and knee arthroplasty. J Bone Joint Surg Am 2005;87:2097-2112.

Page 98: 2009 oite review

Figure 121a Figure 121b Question #: 121 Figures 121a and 121b show the radiographs of an otherwise healthy 42-year-old man who sustained a left ankle injury in a fall on stairs. The ankle is maintained in a position of external rotation and proves irreducible with closed efforts under general anesthesia. What anatomic structure is serving as a hindrance to reduction? 1- Peroneus longus 2- Tibialis posterior 3- Tibialis anterior 4- Ruptured deltoid ligament 5- Posterolateral tibial ridge Preferred Response: 5 Recommended Reading(s): Beekman R, Watson JT: Bosworth fracture-dislocation and resultant compartment syndrome. J Bone Joint Surg Am 2003;85:2211-2214. Hoblitzell RM, Ebraheim NA, Merritt T, et al: Bosworth fracture-dislocation of the ankle: A case report and review of the literature. Clin Orthop Relat Res 1990;255:257-262. Mayer PJ, Evarts CM: Fracture-dislocation of the ankle with posterior entrapment of the fibula behind the tibia. J Bone Joint Surg Am 1978;60:320-324. Perry CR, Rice S, Rao A, et al: Posterior fracture-dislocation of the distal part of the fibula: Mechanism and staging of injury. J Bone Joint Surg Am 1983;65:1149-1157. Szalay MD, Roberts JB: Compartment syndrome after Bosworth fracture-dislocation of the ankle: A case report. J Orthop Trauma 2001;15:301-303. White SP, Pallister I: Fracture-dislocation of the ankle with fixed displacement of the fibula behind the tibia-a rare variant. Injury 2002;33:292-294.

Page 99: 2009 oite review

Figure 122 Question #: 122 Figure 122 shows the radiograph of a 66-year-old woman who reports right thumb pain. Examination reveals a positive grind test. The patient requests an injection, and she has heard of off-label administration of hylan. You advise her that, for the relief of pain and improvement in function, hylan offers which of the following outcomes? 1- Better than placebo and better than corticosteroids 2- Better than placebo but worse than corticosteroids 3- Worse than placebo but better than corticosteroids 4- Worse than placebo and worse than corticosteroids 5- No better than placebo and no better than corticosteroids Preferred Response: 5 Recommended Reading(s): Heyworth BE, Lee JH, Kim PD, et al: Hylan versus corticosteroid versus placebo for treatment of basal joint arthritis: A prospective, randomized, double-blinded clinical trial. J Hand Surg Am 2008;33:40-48. Stahl S, Karsh-Zafrir I, Ratzon N, et al: Comparison of intraarticular injection of depot corticosteroid and hyaluronic acid for treatment of degenerative trapeziometacarpal joints. J Clin Rheumatol 2005;11:299-302. Henderson EB, Smith EC, Pegley F, et al: Intra-articular injections of 750 kD hyaluronan in the treatment of osteoarthritis: A randomized single centre double-blinded placebo-controlled trial of 91 patients demonstrating lack of efficacy. Ann Rheum Dis 1994;53:529-534.

Page 100: 2009 oite review

Figure 123a Figure 123b Question #: 123 Figures 123a and 123b show the radiographs of a 57-year-old man who injured his foot in a 5-foot fall into a ditch. The radiographs indicate what type of injury? 1- Medial subtalar dislocation 2- Lateral subtalar dislocation 3- Calcaneocuboid joint dislocation 4- Lisfranc subluxation 5- Spring ligament rupture Preferred Response: 1 Recommended Reading(s): Fischgrund JS (ed): Orthopaedic Knowledge Update 9. Rosemont, IL, American Academy of Orthopaedic Surgeons, 2008, pp 493-509. Bibbo C, Anderson RB, Davis WH: Injury characteristics and the clinical outcome of subtalar dislocations: A clinical and radiographic analysis of 25 cases. Foot Ankle Int 2003;24:158-163.

Page 101: 2009 oite review

Figure 124 Question #: 124 Figure 124 shows the lateral radiograph of a 24-year-old man who was involved in a motor vehicle accident. He reports axial back pain on arrival in the emergency department. He has no neurologic deficit nor complaints of paresthesia or pain in the lower extremities. He is otherwise healthy and uninjured. His injury is best treated by which of the following? 1- Open reduction and internal fixation 2- Thoracolumbosacral orthosis with a thigh cuff 3- Posterior spinal fusion with instrumentation 4- Bed rest for 4 weeks followed by mobilization in a plaster body jacket 5- Mobilization as tolerated with further treatment based on symptoms Preferred Response: 5 Recommended Reading(s): O’Brien MF: Low-grade isthmic/lytic spondylolisthesis in adults. Instr Course Lect 2003;52:511-524. Virta L, Ronnemaa T: The association of mild-moderate isthmic lumbar spondylolisthesis and low back pain in middle-aged patients is weak and it only occurs in women. Spine 1993;18:1496-1503.

Page 102: 2009 oite review

Question #: 125 Type II (beta) errors in clinical studies are minimized by performing which of the following? 1- Power analysis 2- Bonferroni correction 3- Mann-Whitney U test 4- Logistical regression analysis 5- Increase sample size by 20% Preferred Response: 1 Recommended Reading(s): Einhorn TA, O’Keefe RJ, Buckwalter JA (eds): Orthopaedic Basic Science: Foundations of Clinical Practice, ed 3. Rosemont, IL, American Academy of Orthopaedic Surgeons, 2007, pp 87-101. Lochner HV, Bhandari M, Tornetta P III: Type-II error rates (beta errors) of randomized trials in orthopaedic trauma. J Bone Joint Surg Am 2001;83:1650-1655. Question #: 126 What is the most common complication following a hip revision with isolated polyethylene exchange for osteolysis? 1- Infection 2- Recurrent osteolysis 3- Dislocation 4- Heterotopic ossification 5- Loosening of the acetabular component Preferred Response: 3 Recommended Reading(s): Boucher HR, Lynch C, Young AM, et al: Dislocation after polyethylene liner exchange in total hip arthroplasty. J Arthroplasty 2003;18:654-657. Restrepo C, Ghanem E, Houssock C, et al: Isolated polyethylene exchange versus acetabular revision for polyethylene wear. Clin Orthop Relat Res 2009;467:194-198.

Page 103: 2009 oite review

Question #: 127 What is the first cell to appear following acute muscle injury? 1- Neutrophils 2- Monocytes 3- Macrophages 4- Fibroblasts 5- Myoblasts Preferred Response: 1 Recommended Reading(s): Garrick JG (ed): Orthopaedic Knowledge Update: Sports Medicine 3. Rosemont, IL, American Academy of Orthopaedic Surgeons, 2004, pp 285-290. Tidball JG: Inflammatory cell response to acute muscle injury. Med Sci Sports Exerc 1995;27:1022-1032.

Page 104: 2009 oite review

Figure 128a Figure 128b Question #: 128 A 50-year-old man sustains the injury shown in Figures 128a and 128b. The best treatment of this fracture to prevent articular reduction loss is with a lateral plate, rafting screws, and 1- allograft. 2- autograft. 3- BMP-2. 4- calcium phosphate cement. 5- methylmethacrylate. Preferred Response: 4 Recommended Reading(s): Baumgartner MR, Tornetta P III (eds): Orthopaedic Knowledge Update: Trauma 3. Rosemont, IL, American Academy of Orthopaedic Surgeons, 2005, pp 419-429. Russell TA, Leighton RK: Comparison of autogenous bone graft and endothermic calcium phosphate cement for defect augmentation in tibial plateau fractures: A multicenter, prospective, randomized study. J Bone Joint Surg Am 2008;90:2057-2061. Lobenhoffer P, Gerich T, Witte F, et al: Use of an injectable calcium phosphate bone cement in the treatment of tibial plateau fractures: A prospective study of twenty-six cases with twenty-month mean follow-up. J Orthop Trauma 2002;16:143-149.

Page 105: 2009 oite review

Figure 129a Figure 129b

Figure 129c

Page 106: 2009 oite review

Question #: 129 A 12-year-old right-hand dominant boy has medial right elbow pain that is made worse by overhand throwing. There are no mechanical symptoms. AP and lateral elbow radiographs and a T2-weighted MRI scan are shown in Figures 129a through 129c. Treatment should consist of 1- corrective humeral osteotomy. 2- screw fixation of the medial epicondyle. 3- rest followed by a gradual return to activities. 4- elbow arthroscopy and removal of the loose body. 5- arthroscopic drilling of the osteochondral lesion. Preferred Response: 3 Recommended Reading(s): Chen FS, Diaz VA, Loebenberg M, et al: Shoulder and elbow injuries in the skeletally immature athlete. J Am Acad Orthop Surg 2005;13:172-185. Limpisvasti O, ElAttrache NS, Jobe FW: Understanding shoulder and elbow injuries in baseball. J Am Acad Orthop Surg 2007;15:139-147.

Page 107: 2009 oite review

Figure 130 Question #: 130 Figure 130 shows the radiograph of a 68-year-old woman with type 2 diabetes mellitus who reports a 2-week history of a plantar midfoot ulcer with exposed subcutaneous tissue. Pedal pulses are palpable, there is no exposed bone or tendon, and her blood glucose levels have been stable. Which of the following is the most appropriate treatment? 1- Wound swab for culture and administration of IV vancomycin 2- Plantar midfoot exostectomy 3- Application of a small wire multiplanar external fixator 4- Application of a total contact cast 5- Debridement and application of a vacuum-assisted wound closure device Preferred Response: 4 Recommended Reading(s): Wukich DK, Motko J: Safety of total contact casting in high-risk patients with neuropathic foot ulcers. Foot Ankle Int 2004;25:556-560. Bowker JH, Pfeifer MA: Total contact casting in the treatment of neuropathic ulcers in Bowker JH, Weir J, Pfeifer MA (eds): Levin and O’Neals’s The Diabetic Foot, ed 7. St Louis, MO, Mosby, 2007.

Page 108: 2009 oite review

Question #: 131 In the case of a mother (who is a carrier for Duchenne’s muscular dystrophy gene) and a father (who does not have the Duchenne’s muscular dystrophy gene), what is the chance that the son will be affected by Duchenne’s muscular dystrophy? 1- 25% 2- 50% 3- 75% 4- 100% 5- Unable to determine since the father may be a carrier. Preferred Response: 2 Recommended Reading(s): Thomson WH: The biochemical identification of the carrier state in X-linked recessive (Duchenne) muscular dystrophy. Clin Chim Acta 1969;26:207-221. Question #: 132 The immune response to metallic orthopaedic implants is typically what type? 1- Type I (Ig-E mediated) 2- Type II (antibody mediated) 3- Type III (immune complex mediated) 4- Type IV (delayed-type hypersensitivity reaction) 5- Type V (Ig-M mediated) Preferred Response: 4 Recommended Reading(s): Buckwalter JA, Einhorn TA, Simon SR (eds): Orthopaedic Basic Science: Biology and Biomechanics of the Musculoskeletal System, ed 2. Rosemont, IL, American Academy of Orthopaedic Surgeons, 2000, pp 401-426. Hallab NJ, Mikecz K, Jacobs JJ: A triple assay technique for the evaluation of metal-induced, delayed-type hypersensitivity responses in patients with or receiving total joint arthroplasty. J Biomed Mater Res 2000;53:480-489.

Page 109: 2009 oite review

Question #: 133 A 32-year-old man reports shoulder stiffness following hospitalization for 8 weeks following a polytrauma. Current radiographs show an anterior glenohumeral dislocation. A CT scan reveals 30% attritional bone loss from the anterior inferior glenoid. What is the most appropriate treatment? 1- Closed reduction under general anesthesia 2- Closed reduction followed by arthroscopic Bankart repair 3- Total shoulder arthroplasty 4- Open reduction with Bankart repair 5- Open reduction with bone augmentation of the glenoid Preferred Response: 5 Recommended Reading(s): Shahajpal DT, Zuckerman JD: Chronic glenohumeral dislocation. J Am Acad Orthop Surg 2008;16:385-398. Churchill S, Moskal M, Lippitt S, et al: Extrascapular anatomically contoured anterior glenoid bone grafting for complex glenohumeral instability. Techniques in Shoulder & Elbow Surgery 2001;2:210-218.

Page 110: 2009 oite review

Figure 134 Question #: 134 Figure 134 shows the radiograph of a 54-year-old man who underwent bilateral hip resurfacing 2 years ago and had been doing well until recently. He now describes right groin pain with ambulation and has 4/5 hip flexor strength on this side. What is the next most appropriate step in management? 1- MRI scan of the lumbar spine 2- Revision of the acetabular component 3- Non-weight-bearing for 6 weeks with crutches 4- Conversion to a conventional total hip arthroplasty 5- Physical therapy to improve range of motion and strength Preferred Response: 4 Recommended Reading(s): Anglin C, Masri BA, Tonetti J, et al: Hip resurfacing femoral neck fracture influenced by valgus placement. Clin Orthop Relat Res 2007;465:71-79. Hing C, Back D, Shimmin A: Hip resurfacing: Indications, results, and conclusions. Instr Course Lect 2007;56:171-178.

Page 111: 2009 oite review

Figure 135 Question #: 135 A 13-year-old boy was involved in a motor vehicle collision and sustained severe closed head trauma as well as a minimally displaced pelvic fracture. The latter was treated nonsurgically. Two years later, the patient has improved greatly from his comatose condition but his hip is fixed in a position of abduction and 40° of flexion, preventing him from sitting in a wheelchair. A radiograph is shown in Figure 135. If surgery is contemplated to restore hip range of motion, what is the best way to prevent recurrent ankylosis? 1- Perioperative pamidronate 2- Radiation of the surgical field immediately following surgery 3- Meticulous washing of any bony debris after excision of the bar 4- Complete excision of the proximal femur, including femoral head 5- Intraoperative and postoperative prophylaxis with ketoralac Preferred Response: 2 Recommended Reading(s): Board TN, Karva A, Board RE, et al: The prophylaxis and treatment of heterotopic ossification following lower joint arthroplasty. J Bone Joint Surg Br 2007;89:434-440. Ayers DC, Pelegrini VD Jr, Evarts CM: Prevention of heterotopic ossification in high-risk patients by radiation therapy. Clin Orthop Relat Res 1991;263:87-93.

Page 112: 2009 oite review

Figure 136 Question #: 136 The injury shown in Figure 136 is most likely caused by what mechanism? 1- Repetitive overhead throwing 2- Forced external rotation with the arm at the side 3- Fall onto an abducted, externally rotated shoulder 4- Fall onto a forward flexed, internally rotated shoulder 5- Impingement of the posterior rotator cuff on the posterosuperior labrum Preferred Response: 3 Recommended Reading(s): Fischgrund JS (ed): Orthopaedic Knowledge Update 9. Rosemont, IL, American Academy of Orthopaedic Surgeons, 2008, pp 301-311. Sanders TG, Morrison WB, Miller MD: Imaging techniques for the evaluation of glenohumeral instability. Am J Sports Med 2000;28:414-434.

Page 113: 2009 oite review

Figure 137 Question #: 137 The structure identified by the arrow in Figure 137 represents the 1- spur of the anterior column in its displaced position. 2- spur of the posterior column in its displaced position. 3- portion of the posterior wall in its displaced position. 4- portion of the posterior ilium in its undisplaced position. 5- portion of the posterior ilium in its displaced position. Preferred Response: 4 Recommended Reading(s): Letournel E, Judet R: Fractures of the Acetabulum, ed 2. Berlin, Heidelberg, Germany, Springer-Verlag, 1981. Vrahas MS, Tile M: Fractures of the acetabulum, in Bucholz RW, Heckman JD (eds): Rockwood and Green’s Fractures in Adults, ed 5. Philadelphia, PA, Lippincott Williams & Wilkins, 2001, pp 1513-1545.

Page 114: 2009 oite review

Question #: 138 A two-part olecranon fracture is fixed using Kirschner wires and a tension band. Postoperative radiographs show that the wires have penetrated the anterior cortex of the ulna by 7 mm. Postoperative clinical examination reveals an inability to flex the thumb interphalangeal joint and weak distal interphalangeal flexion of the index finger, but no sensory changes. The preoperative neurovascular examination was normal. The most likely cause of the postoperative examination findings is injury to which of the following nerves? 1- Ulnar 2- Radial 3- Median 4- Posterior interosseous 5- Anterior interosseous Preferred Response: 5 Recommended Reading(s): Mekhail AO, Ebrahaim NA, Jackson WT, et al: Anatomic considerations for the anterior exposure of the proximal portion of the radius. J Hand Surg Am 1996;21:794-801. Parker JR, Conroy J, Campbell DA: Anterior interosseous nerve injury following tension band wiring of the olecranon. Injury 2005;36:1252-1253. Question #: 139 Which of the following bone morphogenetic proteins (BMP) exhibits no osteogenic activity? 1- BMP2 2- BMP3 3- BMP4 4- BMP6 5- BMP7 Preferred Response: 2 Recommended Reading(s): Bahamonde ME, Lyons KM: BMP3: To be or not to be a BMP. J Bone Joint Surg Am 2001;83:S56-S62. Daluiski A, Engstrand T, Bahamonde ME, et al: Bone morphogenetic protein-3 is a negative regulator of bone density. Nat Genet 2001;27:84-88. Cheng H, Jiang W, Phillips FM, et al: Osteogenic activity of the fourteen types of human bone morphogenetic proteins (BMPs). J Bone Joint Surg Am 2003;85:1544-1552.

Page 115: 2009 oite review

Figure 140 Question #: 140 Figure 140 shows the radiograph of a 72-year-old woman who fell 4 weeks after undergoing an elective primary total hip arthroplasty. She was pain free until her fall. Treatment should now consist of 1- femoral revision. 2- acetabular and femoral revision. 3- non-weight-bearing in an abduction orthosis. 4- open reduction and internal fixation with a locking plate. 5- open reduction and internal fixation with cerclage cables. Preferred Response: 1 Recommended Reading(s): O’Shea K, Quinlan JF, Kutty S, et al: The use of uncemented extensively porous-coated femoral components in the management of Vancouver B2 and B3 periprosthetic femoral fractures. J Bone Joint Surg Br 2005;87:1617-1621. Sledge JB III, Abiri A: An algorithm for the treatment of Vancouver type B2 periprosthetic proximal femoral fractures. J Arthroplasty 2002;17:887-892.

Page 116: 2009 oite review

Figure 141 Question #: 141 A 3-year-old girl is brought to the emergency department because she is unable to walk. Her mother indicates that she may have fallen while playing the day before and has cried and not walked since then. She is hard to examine but there is little visible swelling and she seems to react more to pressure over the tibia. Knee motion appears normal. Crying ensues with dorsiflexion of the ankle. Radiographs are shown in Figure 141. What is the most appropriate management? 1- Immobilization in a long leg cast 2- Intramedullary nailing 3- MRI of the lower extremity 4- Laboratory studies including CBC count, erythrocyte sedimentation rate, CRP, and blood cultures, followed by antibiotic treatment 5- Notification of child protective services and initiation of a work-up for suspected child abuse Preferred Response: 1 Recommended Reading(s): Mashru RP, Herman MJ, Pizzutillo PD: Tibial shaft fractures in children and adolescents. J Am Acad Orthop Surg 2005;13:345-352. Heinreich SD: Fractures of the shaft of the tibia and fibula, in Beaty JH, Kasser JR (eds): Rockwood and Wilkins’ Fractures in Children, ed 5. Philadelphia, PA, Lippincott Williams & Wilkins, 2001, pp 1077-1111. Mellick LB, Reesor K, Demers D, et al: Tibial fractures of young children. Pediatr Emerg Care 1988;4:97-101.

Page 117: 2009 oite review

Figure 142 Question #: 142 A 21-year-old collegiate baseball pitcher reports a 6-month history of posterior shoulder pain with ball release. Examination reveals external rotation of 110° and internal rotation of 35° with the shoulder at 90° of abduction. The area of pathology is demonstrated in the CT scan shown in Figure 142. What is the most likely cause of his symptoms? 1- Glenohumeral osteoarthritis 2- Superior labral tear 3- Bankart lesion 4- Bennett lesion 5- Hill-Sachs lesion Preferred Response: 4 Recommended Reading(s): Galatz LM (ed): Orthopaedic Knowledge Update: Shoulder and Elbow 3. Rosemont, IL, American Academy of Orthopaedic Surgeons, 2008, pp 125-133. Wright RW, Paletta GA Jr: Prevalence of the Bennett lesion of the shoulder in major league pitchers. Am J Sports Med 2004;32:121-124.

Page 118: 2009 oite review

Figure 143 Question #: 143 The initial injury responsible for the shoulder pathology shown in Figure 143 is typically best diagnosed on what radiographic view? 1- Axillary 2- Serendipity 3- Supraspinatus outlet 4- AP of the acromioclavicular joint 5- AP of the shoulder Preferred Response: 1 Recommended Reading(s): Sanders TG, Morrison WB, Miller MD: Imaging techniques for the evaluation of glenohumeral instability. Am J Sports Med 2000;28:414-434. Sanders TG, Miller MD: A systematic approach to magnetic resonance imaging interpretation of sports medicine injuries of the shoulder. Am J Sports Med 2005;33:1088-1105.

Page 119: 2009 oite review

Question #: 144 Marginal impaction associated with a posterior wall acetabular fracture is best appreciated on which of the following imaging studies? 1- AP pelvis radiograph 2- Obturator oblique radiograph 3- Iliac oblique radiograph 4- CT 5- False profile view of the hip Preferred Response: 4 Recommended Reading(s): Kellam JF, Messer A: Evaluation of the role of coronal and sagittal axial CT scan reconstructions for the imaging of acetabular fractures. Clin Orthop Relat Res 1994;305:152-159. Letournel E, Judet R: Fractures of the Acetabulum, ed 2. Berlin, Springer-Verlag, 1993, pp 29-61. Question #: 145 A physician recently completed his/her residency and began practice in a three-member group. The physician has been having difficulty attracting new patients and began advertising in the local paper and personal web site as a surgeon that can perform “pain-free” minimally invasive knee arthroplasty on all patients. Are these actions appropriate? 1- No, physicians should not market or promote their skills. 2- No, the Federal Trade Commission prohibits false advertising. 3- No, the physician did not receive specific fellowship training in adult reconstruction. 4- Yes, these actions are appropriate provided the physician has had a patient that had minimal pain after surgery. 5- Yes, these actions are appropriate because the physician trained at an institution that performed minimally invasive knee surgery. Preferred Response: 2 Recommended Reading(s): AAOS Opinion Statement: Opinions on Ethics and Professionalism. Rosemont, IL, American Academy of Orthopaedic Surgeons, 2007. http://www.aaos.org/about/papers/ethics/1205eth.asp. Accessed on July 24, 2009. Capozzi JD: Physician advertising: Evaluation of a sample advertisement. J Bone Joint Surg Am 2007;89:2089-2091.

Page 120: 2009 oite review

Question #: 146 Which of the following conditions is most commonly associated with postaxial polydactyly of the hand? 1- Fanconi’s anemia 2- Radial deficiency 3- Ulnar deficiency 4- Thrombocytopenia 5- Positive family history of polydactyly Preferred Response: 5 Recommended Reading(s): Kozin SH: Upper-extremity congenital anomalies. J Bone Joint Surg Am 2003;85:1564-1576.

Page 121: 2009 oite review

Figure 147 Question #: 147 Without a history of a significant reinjury, a 22-year-old student reports recurrent instability 1 year after undergoing autologous patellar tendon anterior cruciate ligament reconstruction. A radiograph is seen in Figure 147. What is the most likely cause of the instability? 1- Varus alignment 2- Tunnel malposition 3- Failure of biological graft ingrowth 4- Failure of bone healing in the tunnels 5- Unrecognized posterolateral corner injury Preferred Response: 2 Recommended Reading(s): Garrick JG (ed): Orthopaedic Knowledge Update: Sports Medicine 3. Rosemont, IL, American Academy of Orthopaedic Surgeons, 2004, pp 169-181. Pinczewski LA, Salmon LJ, Jackson WF, et al: Radiological landmarks for placement of tunnels in single-bundle reconstruction of the anterior cruciate ligament. J Bone Joint Surg Br 2008;90:172-179.

Page 122: 2009 oite review

Figure 148a Figure 148b Figure 148c

Figure 148d Figure 148e

Page 123: 2009 oite review

Question #: 148 A 76-year-old man reports progressive bilateral lower extremity pain over the past 6 months despite physical therapy. He has a sense of fatigue of the lower extremities with ambulation. He has replaced his prior workout regime of treadmill ambulation with stationary bicycling. Examination reveals normal motor strength in both lower extremities with 2+ pedal pulses. Radiographs and MRI scans are shown in Figures 148a through 148e. What intervention is most likely to reduce his pain level and improve his functional status? 1- A daily program of home lumbar traction 2- A series of transforaminal epidural steroid injections 3- Laminotomy and microdiskectomy at L4/5 bilaterally 4- Laminectomy and partial facetectomies with foraminotomies of L4/5 5- Laminectomy, partial facetectomies, and foraminotomies with posterior spinal fusion from L4/L5 Preferred Response: 4 Recommended Reading(s): Weinstein JN, Tosteson TD, Lurie JD, et al: Surgical versus nonsurgical therapy for lumbar spinal stenosis. New Eng J Med 2008;358:794-810. Malmivaara A, Slatis P, Heliovaara M, et al: Surgical or nonoperative treatment for lumbar spinal stenosis? A randomized controlled trial. Spine 2007;32:1-8. Atlas SJ, Keller RB, Wu YA, et al: Long-term outcomes of surgical and nonsurgical management of lumbar spinal stenosis: 8 to 10 year results from the main lumbar spine study. Spine 2005;30:936-943.

Page 124: 2009 oite review

Question #: 149 Marfan syndrome demonstrates what type of inheritance pattern? 1- Autosomal dominant 2- Autosomal recessive 3- X-linked dominant 4- X-linked recessive 5- Germline mutation Preferred Response: 1 Recommended Reading(s): Einhorn TA, O’Keefe RJ, Buckwalter JA (eds): Orthopaedic Basic Science: Foundations of Clinical Practice, ed 3. Rosemont, IL, American Academy of Orthopaedic Surgeons, 2007, pp 25-47. McBride A, Gargan M: Marfan syndrome. Current Orthopaedics 2006;20:418-423. Question #: 150 A 17-year-old volleyball player has pain and weakness in her dominant shoulder with overhead use during her most recent season. She has been able to voluntarily subluxate the shoulder since the age of 10 and has no history of direct trauma to the shoulder. Appropriate management at this time should include 1- open capsular shift. 2- arthroscopic rotator interval closure. 3- thermal capsulorrhaphy. 4- shoulder stabilization bracing. 5- initiation of a dynamic stabilization therapy program. Preferred Response: 5 Recommended Reading(s): Lee SB, Kim KJ, O’Driscoll SW, et al: Dynamic glenohumeral stability provided by the rotator cuff muscles in the mid-range and end-range of motion: A study of cadavera. J Bone Joint Surg Am 2000;82:849-857. Schenk TJ, Brems JJ: Multidirectional instability of the shoulder: Pathophysiology, diagnosis, and management. J Am Acad Orthop Surg 1998;6:65-72.

Page 125: 2009 oite review

Figure 151 Question #: 151 A 10-month-old boy has been irritable with diaper changes and not been moving his right lower extremity for the past 3 days. His parents report that he had a fever 3 days ago, for which antibiotics were prescribed by his pediatrician. Examination reveals the patient cries on any attempted movement of the right lower extremity. A radiograph of the pelvis is shown in Figure 151. Management of this patient should include 1- aspiration of the hip. 2- observation. 3- application of a Pavlik harness. 4- open reduction and proximal femoral osteotomy. 5- admission to the hospital for bed rest, traction, and anti-inflammatory medication. Preferred Response: 1 Recommended Reading(s): Stans AA: Osteomyelitis and septic arthritis, in Morrissy RT, Weinstein SL (eds): Lovell and Winter’s Pediatric Orthopaedics, ed 6. Philadelphia, PA, Lippincott Williams & Wilkins, 2006, pp 39-491. Sucato DJ, Schwend RM, Gillespie R: Septic arthritis of the hip in children. J Am Acad Orthop Surg 1997;5:249-260.

Page 126: 2009 oite review

Figure 152 Question #: 152 A newborn infant is brought to the office with the features shown in Figure 152. Many children with this condition have multiple systemic anomalies. Which of the following body sites is most likely to have associated abnormalities? 1- Vertebral complex 2- Renal system 3- Cardiac system 4- Hematopoetic system 5- Central nervous system Preferred Response: 4 Recommended Reading(s): Maschke SD, Seitz W, Lawton J: Radial longitudinal deficiency. J Am Acad Orthop Surg 2007;15:41-52. Goldfarb CA, Wustrack R, Pratt JA, et al: Thumb function and appearance in thrombocytopenia: Absent radius syndrome. J Hand Surg Am 2007;32:157-161. Goldfarb CA, Wall L, Manske PR: Radial longitudinal deficiency: The incidence of associated medical and musculoskeletal conditions. J Hand Surg Am 2006;31:1176-1182.

Page 127: 2009 oite review

Question #: 153 What surgical technique will improve a flexion-extension mismatch in a revision total knee arthroplasty when the knee is stable in extension and loose in flexion? 1- Remove additional tibial bone 2- Insert a full block tibial augment 3- Use distal femoral augments 4- Downsize the femoral component 5- Translate the femoral component posteriorly Preferred Response: 5 Recommended Reading(s): Peters CL: Soft-tissue balancing in primary total knee arthroplasty. Instr Course Lect 2006;55:413-417. Question #: 154 The mechanism of action of functional electrical stimulation in the rehabilitation of patients with a spinal cord injury is via direct stimulation of the 1- spinal cord. 2- nerve roots. 3- motor cortex. 4- cauda equina. 5- skeletal muscles. Preferred Response: 5 Recommended Reading(s): Mulcahey MJ, Betz RR, Smith BT, et al: Implanted functional electrical stimulation hand system in adolescents with spinal injuries: An evaluation. Arch Phys Med Rehabil 1997;78:597-607. Davis SE, Mulcahey MJ, Smith BT, et al: Self-reported use of an implanted FES hand system by adolescents with tetraplegia. J Spinal Cord Med 1998;21:220-226.

Page 128: 2009 oite review

Figure 155a Figure 155b Question #: 155 The radiograph of a 46-year-old woman with hip dysplasia is seen in Figure 155a. She undergoes total hip arthroplasty and a postoperative radiograph is shown in Figure 155b. On postoperative day two she reports the inability to dorsiflex her foot when she is sitting in a chair after physical therapy. What is the most appropriate management? 1- IV methylprednisolone 30 mg/kg 2- Remain seated and place the operative leg on a stool 3- MRI/CT to evaluate for a hematoma 4- Transfer to bed with the head of the bed elevated to at least 60° 5- Transfer to bed with the bed level and the knee on the operative side flexed Preferred Response: 5 Recommended Reading(s): Eggli S, Hankemayer S, Muller ME: Nerve palsy after leg lengthening in total replacement arthroplasty for developmental dysplasia of the hip. J Bone Joint Surg Br 1999;81:843-845. Farrell CM, Springer BD, Haidukewych GJ, et al: Motor nerve palsy following primary total hip arthroplasty. J Bone Joint Surg Am 2005;87:2619-2625.

Page 129: 2009 oite review

Figure 156 Question #: 156 A 59-year-old woman reports the insidious onset of left forefoot pain and swelling over the past 4 weeks. A radiograph is shown in Figure 156. What is the next most appropriate step in management? 1- MRI 2- Reassurance that this will resolve with time 3- Walker boot application and evaluation for metabolic bone disease 4- Walker boot application and administration of calcium and vitamin D supplements 5- Noninvasive bone stimulator with pulsed electromagnetic field Preferred Response: 3 Recommended Reading(s): Fischgrund JS (ed): Orthopaedic Knowledge Update 9. Rosemont, IL, American Academy of Orthopaedic Surgeons, 2008, pp 189-196. Bogoch ER, Elliot-Gibson V, Beaton DE, et al: Effective initiation of osteoporosis diagnosis and treatment for patients with a fragility fracture in an orthopaedic environment. J Bone Joint Surg Am 2006;88:25-34.

Page 130: 2009 oite review

Figure 157 Question #: 157 Figure 157 shows the radiograph of a 30-year-old individual who sustained a noncontact soccer injury 2 days ago. What physical examination test is most likely to be abnormal? 1- McMurray 2- Lachman 3- Posterior drawer 4- Varus laxity at 30° 5- Patellar apprehension Preferred Response: 2 Recommended Reading(s): Bathala EA, Bancroft LW, Ortiguera CJ, et al: Radiologic case study: Segond fracture. Orthopedics 2007;30:689, & 797-798. DeLee JC, Drez D Jr, Miller MD (eds): Orthopaedic Sports Medicine, ed 2. Philadelphia, PA, WB Saunders, 2002, p 1643.

Page 131: 2009 oite review

Question #: 158 A 13-month-old boy has had right hip irritability for the last 36 hours. He is febrile and has pain with motion of the right hip. He holds the hip flexed, abducted, and externally rotated. Aspiration of the hip reveals a WBC count of 70,000 cells per mL of fluid. What is the next step in treatment? 1- Arthroscopic debridement 2- Emergent arthrotomy 3- Arthrotomy after MRI is obtained 4- Repeat aspiration in 12 hours while awaiting definitive culture results 5- Administration of IV antibiotics with reevaluation in 24 hours Preferred Response: 2 Recommended Reading(s): Sucato DJ, Schwend RM, Gillespie R: Septic arthritis of the hip in children. J Am Acad Orthop Surg 1997;5:249-260. Caird MS, Flynn JM, Leung YL, et al: Factors distinguishing septic arthritis from transient synovitis of the hip in children: A prospective study. J Bone Joint Surg Am 2006;88:1251-1257.

Page 132: 2009 oite review

Question #: 159 A patient undergoes surgical fixation of an articular fracture of the distal tibia. A clearly inappropriate approach and improper implant selection results in an infected nonunion and implant failure. The patient pursues a second opinion and questions if the standard of care has been breached. The second physician has what type of obligation when answering the patient’s question? 1- Legal but not ethical 2- Ethical but not legal 3- Neither legal nor ethical 4- Both legal and ethical 5- No clear guidelines exist regarding appropriate ethical and legal response Preferred Response: 2 Recommended Reading(s): Bhattacharyya T, Yeon HL: “Doctor, was this surgery done wrong?” Ethical issues in providing second opinions. J Bone Joint Surg Am 2005;87:223-225. Hébert PC, Levin AV, Robertson G: Bioethics for clinicians: 23: Disclosure of medical error. CMAJ 2001;164:509-513. Riddick FA Jr: American Medical Association Council on Ethical and Judicial Affairs. Code of medical ethics: Current opinions with annotations. Chicago, IL, American Medical Association, The Ochsner Journal, 2003, Vol 5, Issue 2. Riddick FA Jr: American Medical Association Counsel: Reporting impaired, incompetent, or unethical colleagues. Chicago, IL, The Ochsner Journal, Spring 2003, Vol 5, Issue 2, p 247.

Page 133: 2009 oite review

Figure 160 Question #: 160 A patient sustains a fracture of the acetabulum. An axial CT scan is shown in Figure 160. What structure takes origin from the region depicted by the arrow? 1- Pectineus 2- Sartorius 3- Iliopectineal fascia 4- Direct (straight) head of the rectus femoris 5- Indirect (reflected) head of the rectus femoris Preferred Response: 4 Recommended Reading(s): Pan H, Kawanabe K, Akiyama H, et al: Operative treatment of hip impingement caused by hypertrophy of the anterior inferior iliac spine. J Bone Joint Surg Br 2008;90:677-679. Hoppenfeld S, deBoer P: The hip and acetabulum, in Hoppenfeld S, deBoer P (eds): Surgical Exposures in Orthopaedics, ed 3. Philadelphia, PA, Lippincott Williams & Wilkins, 2003, pp 365-453.

Page 134: 2009 oite review

Figure 161a Figure 161b

Figure 161c

Page 135: 2009 oite review

Question #: 161 A 12-year-old boy reports left foot pain for the past 2 months that he attributes to jumping off a bunk bed. He denies any penetrating injury to his heel and the skin over the plantar surface is intact. He has pain with walking and has no fever. Examination reveals swelling of the hindfoot and tenderness to direct palpation of the left heel. Laboratory studies show a WBC of 11,000/mm3 (normal 3,500-10,500/mm3), and an erythrocyte sedimentation rate of 45 mm/h (normal up to 20 mm/h). Radiographs and a MRI are shown in Figures 161a through 161c. A culture of material obtained following a needle aspiration of the calcaneus is most likely to yield which of the following organisms? 1- Kingella kingae 2- Staphylococcus aureus 3- Streptococcus pyogenes 4- Pseudomonas aeruginosa 5- Mycobacterium tuberculosis Preferred Response: 2 Recommended Reading(s): Blyth MJ, Kincaid R, Craigne MA, et al: The changing epidemiology of acute and subacute hematogenous osteomyelitis in children. J Bone Joint Surg Br 2001;83:99-102. Karwowska A, Davies HD, Jadavji T: Epidemiology and outcome of osteomyelitis in the era of sequential intravenous-oral therapy. Pediatr Infect Dis J 1998;17:1021-1026. Puffinbarger WR, Gruel CR, Herndon WA, et al: Osteomyelitis of the calcaneus in children. J Pediatr Orthop 1996;16:224-230.

Page 136: 2009 oite review

Question #: 162 Which of the following tendon ruptures is associated with rheumatoid degeneration of the distal radioulnar joint? 1- Flexor carpi ulnaris 2- Flexor digitorum profundus to the little finger 3- Flexor digitorum superficialis to the little finger 4- Extensor digiti minimi 5- Extensor pollicis longus Preferred Response: 4 Recommended Reading(s): Vaughn-Jackson OJ: Rupture of extensor tendons by attrition at the inferior radioulnar joint: Report of two cases. J Bone Joint Surg Am 1948;30:528-530. Williamson SC, Feldon P: Extensor tendon ruptures in rheumatoid arthritis. Hand Clin 1995;11:449-459. Question #: 163 Which of the following best describes an associated both column fracture of the acetabulum? 1- A fracture through the acetabular dome involving both columns 2- A fracture of the articular surface involving both columns with no articular surface remaining attached to the intact posterior ilium 3- A separation of the posterior articular surface and attached ilium from the anterior articular surface and column 4- A central fracture of the articular surface that separates the articular surfaces of the posterior and anterior acetabulum 5- An articular fracture across both columns with extension into the obturator fossa and through the ischium Preferred Response: 2 Recommended Reading(s): Letournel E, Judet R: Fractures of the Acetabulum. New York, NY, Springer-Verlag, 1993, pp 253-254. Vrahas MS, Tile M: Fractures of the acetabulum, in Bucholz RW, Heckman JD (eds): Rockwood and Green’s Fractures in Adults, ed 5. Philadelphia, PA, Lippincott Williams & Wilkins, 2001, pp 1513-1545.

Page 137: 2009 oite review

Question #: 164 A 32-year-old man sustains an incomplete spinal cord injury characterized by ipsilateral impaired motor function and proprioception, and decreased contralateral pain and temperature sensation. What is the most likely clinical syndrome? 1- Anterior cord 2- Central cord 3- Lateral cord 4- Posterior cord 5- Brown-Sequard Preferred Response: 5 Recommended Reading(s): Fischgrund JS (ed): Orthopaedic Knowledge Update 9. Rosemont, IL, American Academy of Orthopaedic Surgeons, 2008, pp 49-65. Spivak JM, Connolly PJ (eds): Orthopaedic Knowledge Update: Spine 3. Rosemont, IL, American Academy of Orthopaedic Surgeons, 2006, pp 179-187. Question #: 165 Polymorphisms in the genes for the calcitonin receptor, estrogen receptor-1, type I collagen alpha-1 chain, or the vitamin D receptor have been shown to be associated with which of the following bone diseases? 1- Osteopetrosis 2- Osteoporosis 3- Osteomalacia 4- Vitamin D-deficient rickets 5- X-linked hypophosphatemic rickets Preferred Response: 2 Recommended Reading(s): Einhorn TA, O’Keefe RJ, Buckwalter JA (eds): Orthopaedic Basic Science: Foundations of Clinical Practice, ed 3. Rosemont, IL, American Academy of Orthopaedic Surgeons, 2007, pp 25-47. Einhorn TA, O’Keefe RJ, Buckwalter JA (eds): Orthopaedic Basic Science: Foundations of Clinical Practice, ed 3. Rosemont, IL, American Academy of Orthopaedic Surgeons, 2007, pp 415-426.

Page 138: 2009 oite review

Figure 166a Figure 166b

Figure 166c Figure 166d

Page 139: 2009 oite review

Question #: 166 What aspect of the fracture pattern may not be adequately addressed when using a lateral locking plate for the treatment of the fracture seen in Figures 166a through 166d? 1- Posteromedial fragment 2- Posterolateral fragment 3- Fibular head fracture 4- Anteromedial fragment depressed fragment 5- Lateral wall comminution Preferred Response: 1 Recommended Reading(s): Barei DP, O’Mara TJ, Taitsman LA, et al: Frequency and fracture morphology of the posteromedial fragment in bicondylar tibial plateau fracture patterns. J Orthop Trauma 2008;22:176-182. Higgins TF, Kemper D, Klatt J: Incidence and morphology of the posteromedial fragment in bicondylar tibial plateau fractures. J Orthop Trauma 2009;23:45-51. Higgins TF, Klatt J, Bachus KN: Biomechanical analysis of bicondylar tibial plateau fixation: How does lateral locking plate fixation compare to dual plate fixation? J Orthop Trauma 2007;21:301-306.

Page 140: 2009 oite review

Question #: 167 Inhibition of what family of proteins has become the basis for a new approach to targeted chemotherapy for a wide variety of human cancers? 1- Tyrosine kinases 2- Tumor necrosis factors (TNFs) 3- Bone morphogenetic proteins (BMPs) 4- Fibroblast growth factor (FGF) family 5- Transforming growth factor (TGF)-beta family Preferred Response: 1 Recommended Reading(s): Shawver LK, Slamon D, Ullrich A: Smart drugs: Tyrosine kinase inhibitors in cancer therapy. Cancer Cell 2002;1:117-123. Einhorn TA, O’Keefe RJ, Buckwalter JA (eds): Orthopaedic Basic Science: Foundations of Clinical Practice, ed 3. Rosemont, IL, American Academy of Orthopaedic Surgeons, 2007, pp 379-393. Question #: 168 Fractures of the anteromedial process of the coronoid can lead to medial elbow instability and early arthritis because of injury at the attachment site of the 1- anterior capsule. 2- annular ligament. 3- anterior bundle of the medial collateral ligament. 4- posterior bundle of the medial collateral ligament. 5- transverse bundle of the medial collateral ligament. Preferred Response: 3 Recommended Reading(s): Steinmann SP: Coronoid process fracture. J Am Acad Orthop Surg 2008;16:519-529. Ring D: Fractures of the coronoid process of the ulna. J Hand Surg Am 2006;31:1679-1689.

Page 141: 2009 oite review

Question #: 169 The greatest difference in strength between unicortical locked screws and bicortical locked screws is seen in what loading conditions? 1- Torsion 2- Axial compression 3- 3-point bending 4- 4-point bending 5- Cantilever bending Preferred Response: 1 Recommended Reading(s): Roberts JW, Grindel SI, Rebholz B, et al: Biomechanical evaluation of locking plate radial shaft fixation: Unicortical locking fixation versus mixed bicortical and unicortical fixation in a sawbone model. J Hand Surg Am 2007;32:971-975. Fulkerson E, Egol KA, Kubiak EN, et al: Fixation of diaphyseal fractures with a segmental defect: A biomechanical comparison of locked and conventional plating techniques. J Trauma 2006;60:830-835. Question #: 170 A complete spinal cord injury after a cervical fracture-dislocation is best characterized by which of the following clinical scenarios? 1- Inability to elevate both arms to perform feeding and self care 2- Absent bowel and bladder control or perirectal sensation 3- Absent voluntary motor function but retained proprioceptive and vibratory sensation in the lower extremities 4- Absent distal motor function with flaccid rectal tone and no bulbocavernosus reflex 5- Absent distal motor function and sensation with absent rectal tone and an intact bulbocavernosus reflex Preferred Response: 5 Recommended Reading(s): Kirshblum S, Campagnolo D, DeLisa J: Spinal Cord Medicine. Philadelphia, PA, Lippincott Williams & Wilkins, 2002. American Spinal Injury Association/International Medical Society of Paraplegia. International standards for neurological and functional classification of spinal cord injury patients. Chicago, IL, 2000. Neurological assessment: The motor examination in Ditunno JF, Donovan WH, Maynared FM (eds): Reference Manual for the International Standards for Neurological and Functional Classification of Spinal Cord Injury. Chicago, IL, American Spinal Injury Association, 1994.

Page 142: 2009 oite review

Question #: 171 Which of the following preoperative findings in a patient with cerebral palsy best predicts improvement in function after wrist and hand tendon transfer surgery? 1- Good sensation 2- Dystonic features 3- Improvement in resting tone after botulinum injections 4- Full passive range of motion 5- Voluntary control of motion Preferred Response: 5 Recommended Reading(s): Van Heest AE, House JH, Cariello C: Upper extremity surgical treatment of cerebral palsy. J Hand Surg Am 1999;24:323-330. Van Heest AE. Surgical management of wrist and finger deformity. Hand Clin 2003;19:657-665. Question #: 172 The lowest hip joint reactive forces are present when the acetabular component is placed in what position? 1- Superior and medial 2- Superior and lateral 3- Inferior and medial 4- Inferior and lateral 5- Acetabular location does not have an effect Preferred Response: 3 Recommended Reading(s): Johnston RC, Brand RA, Crowninshield RD: Reconstruction of the hip: A mathematical approach to determine optimum geometric relationships. J Bone Joint Surg Am 1979;61:639-652.

Page 143: 2009 oite review

Figure 173a Figure 173b

Figure 173c

Page 144: 2009 oite review

Question #: 173 The mechanism of injury of the fracture seen in Figures 173a through 173c is best described as 1- pronation external rotation. 2- pronation abduction. 3- supination external rotation. 4- supination adduction. 5- supination internal rotation. Preferred Response: 2 Recommended Reading(s): Marsh JL, Saltzman CL: Ankle fractures, in Rockwood and Green’s Fractures in Adults, ed 6. Philadelphia, PA, Lippincott Williams & Wilkins, 2006, pp 2147-2247. Siegel J, Tornetta P III: Extraperiosteal plating of pronation-abduction ankle fractures. J Bone Joint Surg Am 2007;89:276-281.

Page 145: 2009 oite review

Question #: 174 At long-term follow-up (more than 25 years after the initial procedure), infants with clubfoot who were treated with extensive soft-tissue release demonstrate 1- joint stiffness only at the subtalar joint. 2- good long-term function if treated before age 1 year. 3- outcomes unrelated to the extent of the soft-tissue release. 4- muscle weakness primarily in the peroneal muscles. 5- poor long-term foot function, with the degree of impairment related to the extent of the soft-tissue release. Preferred Response: 5 Recommended Reading(s): Dobbs MB, Nunley R, Schoenecker PL: Long-term follow-up of patients with clubfeet treated with extensive soft-tissue release. J Bone Joint Surg Am 2006;88:986-996. Fischgrund JS (ed): Orthopaedic Knowledge Update 9. Rosemont, IL, American Academy of Orthopaedic Surgeons, 2008, pp 741-756. Question #: 175 Of the available osteoconductive bone graft substitutes, which is most rapidly resorbed? 1- Calcium sulfate 2- Calcium phosphate 3- Tricalcium phosphate 4- Coralline hydroxyapatite 5- Collagen-based matrices Preferred Response: 1 Recommended Reading(s): Walsh WR, Morberg P, Yu Y, et al: Response of a calcium sulfate bone graft substitute in a confined cancellous defect. Clin Orthop Relat Res 2003;406:228-236. Watson JT: The use of an injectable bone graft substitute in tibial metaphyseal fractures. Orthopedics 2004;27:S103-S107. Bucholz RW: Nonallograft osteoconductive bone graft substitutes. Clin Orthop Relat Res 2002;395:44-52. Kelly CM, Wilkins RM, Gitelis S, et al: The use of a surgical grade calcium sulfate as a bone graft substitute: Results of a multicenter trial. Clin Orthop Relat Res 2001;382:42-50. Tay BK, Patel VV, Bradford DS: Calcium sulfate- and calcium phosphate-based bone substitutes: Mimicry of the mineral phase of bone. Orthop Clin North Am 1999;30:615-623.

Page 146: 2009 oite review

Figure 176 Question #: 176 When treating the fracture shown in Figure 176 with intramedullary nailing, in what position would a blocking screw help prevent an apex anterior deformity? 1- Anterior to the nail in the distal fragment 2- Anterior to the nail in the proximal fragment 3- Anterior to the nail in the proximal fragment and posterior to the nail in the distal fragment 4- Posterior to the nail in the proximal fragment 5- Posterior to the nail in the distal fragment Preferred Response: 4 Recommended Reading(s): Krettek C, Stephan C, Schandelmaier P, et al: The use of Poller screws as blocking screws in stabilizing tibial fractures treated with small diameter intramedullary nails. J Bone Joint Surg Br 1999;81:963-968. Ricci WM, O’Boyle M, Borrelli J, et al: Fractures of the proximal third of the tibial shaft treated with intramedullary nails and blocking screws. J Orthop Trauma 2001;15:264-270.

Page 147: 2009 oite review

Question #: 177 When performing open reduction and internal fixation, absolute stability at the fracture site is best achieved with a 1- locking plate in “bridging” mode. 2- single interfragmentary lag screw and neutralization plate. 3- long standard plate with widely spaced screws placed in a neutral position. 4- ring external fixator. 5- reamed intramedullary nail with good endosteal contact. Preferred Response: 2 Recommended Reading(s): Ito K, Perren SM: Biology and biomechanics in bone healing, in Rüedi TP, Buckley RE, Moran CG (eds): AO Principles of Fracture Management, ed 2. Boston, MA, AO Publishing, 2007, pp 9-31. Claes LE, Heigele CA: Magnitudes of local stress and strain along bone surfaces predict the course and type of fracture healing. J Biomech 1999;32:255-266.

Page 148: 2009 oite review

Figure 178a Figure 178b Question #: 178 A 15-year-old boy injured his right elbow in a fall. Radiographs are shown in Figures 178a and 178b. Treatment should consist of 1- open reduction and internal fixation of the fracture. 2- open reduction of the elbow joint and removal of the loose body, followed by early motion of the elbow. 3- closed reduction and percutaneous pin fixation of the fracture. 4- immobilization for 3 weeks, followed by gradual mobilization. 5- arthroscopic excision of the loose body, followed by early elbow motion. Preferred Response: 1 Recommended Reading(s): Lee HH, Shen HC, Chang JH, et al: Operative treatment of displaced medial epicondyle fractures in children and adolescents. J Shoulder Elbow Surg 2005;14:178-185. Farsetti P, Potenza V, Caterini R, et al: Long-term results of treatment of fractures of the medial humeral epicondyle in children. J Bone Joint Surg Am 2001;83:1299-1305.

Page 149: 2009 oite review

Question #: 179 Rheumatoid factors are antibodies directed against 1- multiple immunoglobulins. 2- IgA. 3- IgE. 4- IgM. 5- IgG. Preferred Response: 5 Recommended Reading(s): Vaccaro AR (ed): Orthopaedic Knowledge Update 8. Rosemont, IL, American Academy of Orthopaedic Surgeons, 2005, pp 229-244. James D, Young A, Kulinskaya E, et al: Early Rheumatoid Arthritis Study Group (ERAS). Orthopaedic intervention in early rheumatoid arthritis: Occurrence and predictive factors in an inception cohort of 1064 patients followed for 5 years. United Kingdom, Rheumatology (Oxford) 2004;43:369-376. Question #: 180 Which of the following is an inhibitor of particle-induced osteolysis? 1- Interleukin-1 alpha 2- Interleukin-6 3- Osteoprotegerin 4- Tumor necrosis factor (TNF)-alpha 5- Receptor-activator of nuclear factor KB(RANK) ligand Preferred Response: 3 Recommended Reading(s): Einhorn TA, O’Keefe RJ, Buckwalter JA (eds): Orthopaedic Basic Science: Foundations of Clinical Practice, ed 3. Rosemont, IL, American Academy of Orthopaedic Surgeons, 2007, pp 365-377. Jacobs JJ, Roebuck KA, Archibeck M, et al: Osteolysis: Basic science. Clin Orthop Relat Res 2001;393:71-77.

Page 150: 2009 oite review

Question #: 181 Which of the following nonsurgical treatment techniques has the strongest supporting evidence as an effective treatment for osteoarthritis of the knee? 1- Acupuncture 2- Intra-articular injection of hyaluronan 3- Unloading knee brace 4- Weight loss 5- Range of motion and flexibility exercises Preferred Response: 4 Recommended Reading(s): AAOS Clinical Guideline on the Treatment of Osteoarthritis of the Knee (Non-Arthroplasty). Rosemont, IL, American Academy of Orthopaedic Surgeons, 2008. http://www.aaos.org/research/guidelines/GuidelineOAKnee.asp. Accessed on July 24, 2009. Question #: 182 Which of the following best describes the contour and vertical position of the lateral tibial plateau relative to the medial? 1- Symmetric in contour and vertical position 2- More concave-shaped and situated more proximal 3- More concave-shaped and situated more distal 4- More convex-shaped and situated more proximal 5- More convex-shaped and situated more distal Preferred Response: 4 Recommended Reading(s): Watson T, Schatzker J: Tibial plateau fractures, in Browner BD, Jupiter JB, Levine AM (eds): Skeletal Trauma, ed 3. Philadelphia, PA, WB Saunders, 2003, pp 2074-2130. Egol KA, Koval KJ: Fractures of the proximal tibia in, Bucholz RW, Heckman JD, Court-Brown C (eds): Fractures in Adults, ed 6. Philadelphia, PA, Lippincott Williams & Wilkins, 2001, pp 1999-2029. Hashemi J, Chandrashekar N, Gill B, et al: The geometry of the tibial plateau and its influence on the biomechanics of the tibiofemoral joint. J Bone Joint Surg Am 2008;90:2724-2734.

Page 151: 2009 oite review

Question #: 183 During the posterior approach to the humerus, where is the radial nerve found relative to the triceps muscle? 1- Superficial to all heads 2- Medial to the long, lateral, and deep heads 3- Medial to the long and lateral heads and proximal to the deep head 4- Medial to the long head and superficial to the lateral and deep heads 5- Medial to the lateral head, superficial to the long head, and proximal to the deep head Preferred Response: 3 Recommended Reading(s): Hoppenfeld S, deBoer P: Surgical Exposures in Orthopaedics. Philadelphia, PA, Lippincott Williams & Wilkins, 1984, pp 58-63. Martini FH, Timmons MJ, Tallitsch RB: Human Anatomy, ed 5. San Francisco, CA, Pearson/Benjamin Cummings, 2006, pp 300-302. Question #: 184 A 56-year-old woman with end-stage arthritis undergoes a primary total knee arthroplasty with a standard posterior stabilized knee implant. Preoperatively she had a 10° flexion contracture with 120° of flexion. A postoperative radiograph demonstrates well-aligned implants and full extension on the lateral radiograph. At 4 weeks postoperatively she has a 10° flexion contracture. What is the most likely cause of her residual contracture? 1- Hamstring tightness and spasm 2- Oversized femoral implant 3- Overstuffing the patellofemoral space 4- Inability to reach terminal extension preoperatively 5- Use of standard knee implants instead of gender-specific implants Preferred Response: 1 Recommended Reading(s): Bellemans J, Vandenneucker H, Victor J, et al: Flexion contracture in total knee arthroplasty. Clin Orthop Relat Res 2006;452:78-82.

Page 152: 2009 oite review

Figure 185 Question #: 185 Given the data in Figure 185, the positive predictive value of the new meniscus test is expressed as which of the following times 100? 1- A+B/A+B+C+D 2- A/A+B 3- D/C+D 4- A/A+C 5- D/B+D Preferred Response: 4 Recommended Reading(s): Wojtys EM, Greenfield ML, Kuhn JE: A statistics primer: Statistical thermology-part 2. Am J Sports Med 1996;24:564-565. Kuhn JE, Greenfield ML, Wojtys EM: A statistics primer: Prevalence, incidence, relative risks, and odds ratios: Some epidemiologic concepts in the sports medicine literature. Am J Sports Med 1997;25:414-416.

Page 153: 2009 oite review

Figure 186 Question #: 186 In the fracture shown in Figure 186, which of the following radiographic views of the elbow is most likely to accurately show the maximum degree of displacement of the fracture fragment? 1- Lateral 2- Anteroposterior 3- Internal oblique 4- External oblique 5- Posteroanterior Preferred Response: 3 Recommended Reading(s): Song KS, Kang CH, Min BW, et al: Internal oblique radiographs for diagnosis of nondisplaced or minimally displaced lateral humeral condylar fractures of the humerus in children. J Bone Joint Surg Am 2007;89:58-63. Bast SC, Hoffer MM, Aval S: Nonoperative treatment for minimally and non-displaced lateral humeral condyle fractures in children. J Pediatr Orthop 1998;18:448-450.

Page 154: 2009 oite review

Question #: 187 A 24-year-old man sustained an open tibia fracture in a motorcycle accident. What patient or injury factor has the highest relative odds for amputation of the limb? 1- Fracture classification/pattern 2- Associated open foot fracture 3- Severity of the muscle injury 4- History of alcohol abuse 5- Type of medical insurance Preferred Response: 3 Recommended Reading(s): MacKenzie EJ, Bosse MJ, Kellam JF, et al: Factors influencing the decision to amputate or reconstruct after high-energy lower extremity trauma. J Trauma 2002;52:641-649. Question #: 188 Which of the following nerves serves as an anatomic landmark leading to the radial nerve during a paratricipital approach for humeral shaft fracture fixation? 1- Musculocutaneous 2- Lateral antebrachial cutaneous 3- Medial antebrachial cutaneous 4- Medial brachial cutaneous 5- Posterior antebrachial cutaneous Preferred Response: 5 Recommended Reading(s): Gerwin M, Hotchkiss RN, Weiland AJ: Alternative operative exposures of the posterior aspect of the humeral diaphysis, with reference to the radial nerve. J Bone Joint Surg Am 1996;78:1690-1695. Zlotolow DA, Catalano LW III, Barron OA, et al: Surgical exposures of the humerus. J Am Acad Orthop Surg 2006;14:754-765.

Page 155: 2009 oite review

Question #: 189 What is the most common deformity seen after intramedullary nailing of a proximal tibia fracture? 1- Varus and apex anterior angulation 2- Varus and apex posterior angulation 3- Valgus and apex anterior angulation 4- Valgus and apex posterior angulation 5- Valgus and external rotation Preferred Response: 3 Recommended Reading(s): Freedman EL, Johnson EE: Radiographic analysis of tibial fracture malalignment following intramedullary nailing. Clin Orthop Relat Res 1995;315:25-33. Lang GJ, Cohen BE, Bosse MJ, et al: Proximal third tibial shaft fractures: Should they be nailed? Clin Orthop Relat Res 1995;315:64-74.

Page 156: 2009 oite review

Figure 190a Figure 190b Question #: 190 The patient whose MRI scans are seen in Figures 190a and 190b has a 4-week history of lower extremity weakness and imbalance with ambulation. Examination shows hyperreflexia and clonus in the lower extremities. What is the best treatment option? 1- Laminectomy 2- Physical therapy 3- Epidural injection 4- Anterior diskectomy 5- Observation and pain medications Preferred Response: 4 Recommended Reading(s): Bohlman HH, Zdeblick TA: Anterior excision of herniated thoracic discs. J Bone Joint Surg Am 1988;70:1038-1047. Currier BL, Eismont FJ, Green BA: Transthoracic disc excision and fusion for herniated thoracic discs. Spine 1994;19:323-328.

Page 157: 2009 oite review

Figure 191a Figure 191b Question #: 191 Figures 191a and 191b show the radiographs of a 55-year-old man who elects to undergo surgery for chronically dislocated second and third metatarsophalangeal joints. Following extensor tenotomies and complete capsular releases, the toes continue to dislocate. What is the next most appropriate step in treatment? 1- Flexor tenotomies 2- Amputation of the second toe 3- Metatarsal head resections 4- Metatarsal shortening osteotomies 5- Osteotomies of the proximal phalanx Preferred Response: 4 Recommended Reading(s): Coughlin MJ: Lesser-toe abnormalities. J Bone Joint Surg Am 2002;84:1446-1469. Trinka HJ, Muhlbauer M, Zettl R, et al: Comparison of the results of the Weil and Helal osteotomies for the treatment of metatarsalgia secondary to dislocation of the lesser metatarsophalangeal joints. Foot Ankle Int 1999;20:72-79.

Page 158: 2009 oite review

Question #: 192 When using a tension band technique for fixation of a simple transverse olecranon fracture, gentle active motion of the elbow produces what type of forces across the fracture at the level of the articular surface? 1- Tension 2- Shear 3- Compression 4- Bending 5- No load - the implants and fixation neutralize the fracture from load Preferred Response: 3 Recommended Reading(s): Jupiter JB: AO Manual of Fracture Management: Hand and Wrist. Boston, MA, AO Publishing, 2006. Chandler RW: Principles of internal fixation, in Bucholz RW, Heckman JD (eds): Rockwood and Green’s Fractures in Adults, ed 5. Philadelphia, PA, Lippincott Williams & Wilkins, 2001, pp 181-229. Question #: 193 Osteosarcoma most commonly develops in which of the following locations? 1- Pelvis 2- Distal humerus 3- Proximal tibia 4- Proximal femur 5- Proximal humerus Preferred Response: 3 Recommended Reading(s): Menendez LR (ed): Orthopaedic Knowledge Update: Musculoskeletal Tumors. Rosemont, IL, American Academy of Orthopaedic Surgeons, 2002, pp 175-186. Mankin HJ, Hornicek FJ, Rosenberg AE, et al: Survival data for 648 patients with osteosarcoma treated at one institution. Clin Orthop Relat Res 2004;429:286-291.

Page 159: 2009 oite review

Figure 194a Figure 194b Question #: 194 Figures 194a and 194b show the radiographs of a 25-year-old woman who reports that a softball impacted the tip of her middle finger. Which of the following factors is the most important determinant of final functional outcome? 1- Stable internal fixation 2- Continuous passive motion in the postoperative period 3- Anatomic reduction of the middle phalanx articular surface 4- Reattachment of the volar plate to the base of the middle phalanx 5- Reduction of the middle phalanx on the condyles of the proximal phalanx Preferred Response: 5 Recommended Reading(s): Kiefhaber TR, Stern PJ: Fracture dislocations of the proximal interphalangeal joint. J Hand Surg Am 1998;23:368-380. Dias JJ: Intra-articular injuries of the distal and proximal interphalangeal joints, in Berger RA, Weiss APC (eds): Hand Surgery. Philadelphia, PA, Lippincott Williams & Wilkins, 2004, pp 153-174.

Page 160: 2009 oite review

Question #: 195 When comparing a retrograde insertion technique to an antegrade insertion technique for treatment of femoral shaft fractures, the retrograde technique has a greater incidence of 1- union. 2- nonunion. 3- malunion. 4- hip pain. 5- knee pain. Preferred Response: 5 Recommended Reading(s): Ricci WM, Bellobarba C, Evanoff B, et al: Retrograde versus antegrade nailing of femoral shaft fractures. J Orthop Trauma 2008;22:S31-S38. Winquist RA, Hansen ST Jr, Clawson DK: Closed intramedullary nailing of femoral fractures: A report of five hundred and twenty cases. J Bone Joint Surg Am 1984;66:529-539. Moed BR, Watson JT, Cramer KE, et al: Unreamed retrograde intramedullary nailing of fractures of the femoral shaft. J Orthop Trauma 1998;12:334-342. Question #: 196 A 16-year-old male soccer player collapses on the field. What is the most likely cause of sudden cardiac death in this case? 1- Aortic stenosis 2- Mitral valve prolapse 3- Dilated cardiomyopathy 4- Hypertrophic cardiomyopathy 5- Arrhythmogenic right ventricular dysplasia Preferred Response: 4 Recommended Reading(s): Garrick JG (ed): Orthopaedic Knowledge Update: Sports Medicine 3. Rosemont, IL, American Academy of Orthopaedic Surgeons, 2004, pp 311-321. Basillico FC: Cardiovascular disease in athletes. Am J Sports Med 1999;27:108-121.

Page 161: 2009 oite review

Figure 197a Figure 197b Question #: 197 A 19-year-old man has had a 6-month history of progressive left hip pain. He has taken ibuprofen continuously for the past 3 months, stating that it provides complete but transient relief. An AP radiograph and CT scan are shown in Figures 197a and 197b. What is the next most appropriate step in management? 1- Biopsy 2- Crutches 3- Chest imaging 4- En bloc resection 5- Percutaneous ablation Preferred Response: 5 Recommended Reading(s): Lee EH, Shafi M, Hui JH: Osteoid osteoma: A current review. J Pediatr Orthop 2006;26:695-700. Moser T, Giacomelli MC, Clavert JM, et al: Image-guided laser ablation of osteoid osteoma in pediatric patients. J Pediatr Orthop 2008;28:265-270.

Page 162: 2009 oite review

Question #: 198 Early (less than 5 days) rather than late (10-14 days) open reduction and internal fixation of complex acetabular fractures is beneficial because it is most likely to result in which of the following? 1- Less blood loss 2- Reduced surgical time 3- Reduced incidence of deep venous thrombosis 4- Reduced risk of neurologic injury 5- Improved fracture reduction Preferred Response: 5 Recommended Reading(s): Baumgaertner MR, Tornetta P III (eds): Orthopaedic Knowledge Update: Trauma 3. Rosemont, IL, American Academy of Orthopaedic Surgeons, 2005, pp 271-280. Plaisier BR, Meldon SW, Super DM, et al: Improved outcome after early fixation of acetabular fractures. Injury 2000;31:81-84.

Page 163: 2009 oite review

Figure 199a Figure 199b Question #: 199 Figures 199a and 199b show the MRI scans of a man who reports cervical pain with right-sided upper extremity radiating pain. His neurologic examination is most likely to show which of the following deficits? 1- Weakness in the deltoid with no reflex abnormality and diminished sensation over the upper shoulder 2- Weakness in the elbow flexors with a diminished biceps reflex and loss of sensation over the lateral arm 3- Weakness in the triceps with decreased triceps reflex and diminished sensation involving the middle finger 4- Weakness in the elbow flexors with a decreased brachioradialis reflex and diminished sensation over the thumb and index finger 5- Weakness in the interossei with no reflex change and diminished sensation over the medial border of the hand and forearm Preferred Response: 3 Recommended Reading(s): Heller JG: The syndromes of degenerative cervical disease. Orthop Clin North Am 1992;23:381-394. Zeidman S: Evaluation of patients with cervical spine lesions, in Clark CR, Benzel EZ, Currier BL, et al (eds): The Cervical Spine, ed 4. Philadelphia, PA, Lippincott Williams & Wilkins, 2004, pp 149-165.

Page 164: 2009 oite review

Figure 200a Figure 200b Question #: 200 An obese 60-year-old woman with type 2 diabetes mellitus has a painless swollen foot. Radiographs are shown in Figures 200a and 200b. Which of the following is most likely associated with this foot deformity? 1- Acute trauma 2- Morbid obesity 3- Peripheral neuropathy 4- Peripheral vascular disease 5- Plantar fascial rupture Preferred Response: 3 Recommended Reading(s): Brodsky JW: The diabetic foot, in Coughlin MJ, Mann RA (eds): Surgery of the Foot and Ankle, ed 7. St Louis, MO, Mosby, 1999, pp 895-969. Guyton GP, Saltzman CL: The diabetic foot: Basic mechanism of disease. J Bone Joint Surg Am 2001;83:1083-1096.

Page 165: 2009 oite review

Question #: 201 When performing a transfemoral amputation, what surgical technique has the greatest impact on functional outcome? 1- Iliotibial band release 2- Adductor myodesis 3- Primary wound closure 4- A posterior-based skin flap 5- An anterior-based skin flap Preferred Response: 2 Recommended Reading(s): Pinzur MS, Gottschalk F, Pinto MA, et al: Controversies in lower extremity amputation. Instr Course Lect 2008;57:663-672. Pinzur MS, Bowker JH, Smith DG, et al: Amputation surgery in peripheral vascular disease. Instr Course Lect 1999;48:687-691. Question #: 202 What clinical scenario is a contraindication for use of a constrained acetabular liner? 1- Patient with a cup in 40° of abduction and 25° of anteversion 2- Patient with a cup in 60° of abduction and 5° of retroversion 3- Neurologic decline 4- Absent abductor mechanism 5- When used in combination with an acetabular reconstruction cage Preferred Response: 2 Recommended Reading(s): Callaghan JJ, O’Rourke MR, Goetz DD, et al: Use of a constrained tripolar acetabular liner to treat intraoperative instability and postoperative dislocation after total hip arthroplasty: A review of our experience. Clin Orthop Relat Res 2004;429:117-123. Soong M, Rubash HE, Macaulay W: Dislocation after total hip arthroplasty. J Am Acad Orthop Surg 2004;12:314-321.

Page 166: 2009 oite review

Figure 203a Figure 203b

Figure 203c

Page 167: 2009 oite review

Question #: 203 A 12-year-old girl reports a history of right foot pain and recurrent right ankle sprains. AP, lateral, and oblique foot radiographs are shown in Figures 203a through 203c. Immobilization in a short leg cast for 6 weeks has failed to provide significant pain relief. Treatment should now consist of 1- subtalar fusion with internal fixation. 2- extra-articular subtalar fusion with allograft. 3- triple arthrodesis. 4- combined cuboid-cuneiform osteotomy. 5- excision of the calcaneal navicular coalition with interposition of extensor digitorum brevis muscle. Preferred Response: 5 Recommended Reading(s): Vincent KA: Tarsal coalition and painful flatfoot. J Am Acad Orthop Surg 1998;6:274-281. Cowell HR, Elener V: Rigid painful flatfoot secondary to tarsal coalition. Clin Orthop Relat Res 1983;177:54-60. Swiontkowski MF, Scranton PE, Hansen S: Tarsal coalitions: Long-term results of surgical treatment. J Pediatr Orthop 1983;3:287-292.

Page 168: 2009 oite review

Figure 204 Question #: 204 What is the best treatment option for the injury to the ankle shown in Figure 204? 1- Closed reduction and a total contact cast 2- Open reduction and internal fixation of the fibula only 3- Open reduction and internal fixation of the fibula and syndesmotic fixation 4- Open reduction and internal fixation of the fibula and repair of the deltoid ligament 5- Open reduction and internal fixation of the fibula, repair of the deltoid ligament, and syndesmotic fixation Preferred Response: 3 Recommended Reading(s): Zalavras C, Thordarson D: Ankle syndesmotic injury. J Am Acad Orthop Surg 2007;15:330-339. Marsh JL, Saltzman CL: Ankle fractures, in Bucholz RW, Heckman JD (eds): Rockwood and Green’s Fractures in Adults, ed 5. Philadelphia, PA, Lippincott Williams & Wilkins, 2001, pp 2001-2090.

Page 169: 2009 oite review

Question #: 205 When evaluating patients after severe lower extremity injuries, what subscale of the Sickness Impact Profile does not improve with time? 1- Work 2- Recreation 3- Psychosocial 4- Physical function 5- Household management Preferred Response: 3 Recommended Reading(s): Bosse MJ, MacKenzie EJ, Kellam JF, et al: An analysis of outcomes of reconstruction or amputation of leg-threatening injuries. N Engl J Med 2002;347:1924-1931. MacKenzie EJ, Morris JA Jr, Jurkovich GJ, et al: Return to work following injury: The role of economic, social, and job-related factors. Am J Public Health 1998;88:1630-1637.

Page 170: 2009 oite review

Figure 206a Figure 206b Question #: 206 Figures 206a and 206b show the MRI scans of a 39-year-old woman with a 2-month history of right-sided leg pain radiating down the posterior aspect of the thigh and onto the sole of the foot. Her pain has improved only slightly over time despite a caudal epidural steroid injection and oral anti-inflammatory drugs, as well as physical therapy. What intervention is most likely to provide the greatest pain relief at this time? 1- Spinal manipulation 2- Repeat caudal epidural steroid injection 3- Laminotomy and diskectomy of the L5/S1 level 4- Lumbar diskectomy and interbody fusion at L5-S1 5- Posterior instrumented lumbar spine fusion across the L5/S1 disk with a right-sided laminectomy of L5 Preferred Response: 3 Recommended Reading(s): Weinstein JN, Lurie JD, Tosteson TD, et al: Surgical vs nonoperative treatment for lumbar disk herniation: The Spine Patient Outcomes Research Trial (SPORT) observational cohort. JAMA 2006;296:2451-2459. Weinstein JN, Tosteson TD, Lurie JD, et al: Surgical vs nonoperative treatment for lumbar disk herniation: The Spine Patient Outcomes Research Trial (SPORT): A randomized trial. JAMA 2006;296:2441-2450. Weber H: Lumbar disc herniation: A controlled, prospective study with ten years of observation. Spine 1983;8:131-140.

Page 171: 2009 oite review

Figure 207a Figure 207b

Figure 207c

Page 172: 2009 oite review

Question #: 207 Figures 207a through 207c show the radiographs and CT scan of an otherwise healthy 42-year-old man who sustained a displaced right talar neck fracture in a motor vehicle accident. Reduction and fixation are performed through a medial approach with anteromedial compression screws. What motion is most significantly compromised? 1- Ankle dorsiflexion 2- Ankle plantar flexion 3- Ankle axial rotation 4- Foot eversion 5- Foot inversion Preferred Response: 4 Recommended Reading(s): Daniels TR, Smith JW, Ross TI: Varus malalignment of the talar neck: Its effect on the position of the foot and on subtalar motion. J Bone Joint Surg Am 1996;78:1559-1567. Herscovici D Jr, Anglen JO, Archdeacon M, et al: Avoiding complications in the treatment of pronation-external rotation ankle fractures, syndesmotic injuries, and talar neck fractures. J Bone Joint Surg Am 2008;90:898-908. Sanders DW, Busam M, Hattwick E, et al: Functional outcomes following displaced talar neck fractures. J Orthop Trauma 2004;18:265-270.

Page 173: 2009 oite review

Figure 208 Question #: 208 A nonambulatory 7-year-old girl with spastic quadriplegia has hip abduction of 30° on the right and 10° on the left. An AP pelvis radiograph is shown in Figure 208. Treatment should consist of 1- full-time hip abduction bracing. 2- observation, with follow-up AP pelvis radiographs in 1 year. 3- administration of botulinum toxin to the hip adductors bilaterally, followed by physical therapy. 4- bilateral hip adductor releases without osteotomy, followed by abduction bracing. 5- bilateral varus derotation femoral osteotomies in conjunction with bilateral hip adductor releases. Preferred Response: 5 Recommended Reading(s): Flynn JM, Miller F: Management of hip disorders in patients with cerebral palsy. J Am Acad Orthop Surg 2002;10:198-209. Spiegel DA, Flynn JM: Evaluation and treatment of hip dysplasia in cerebral palsy. Orthop Clin North Am 2006;37:185-196.

Page 174: 2009 oite review

Question #: 209 A patient with an open mid-diaphyseal tibial shaft fracture undergoes debridement, unreamed tibial nailing, and soleus flap coverage. Radiographs obtained 6 months following the injury reveal a hypertrophic nonunion. The most appropriate treatment is 1- open reduction and plate fixation using iliac crest bone graft. 2- open reduction and plate fixation using bone morphogenetic protein. 3- exchanged reamed tibial nailing. 4- exchanged reamed tibial nailing with iliac crest bone graft. 5- exchanged reamed tibial nailing with bone morphogenetic protein. Preferred Response: 3 Recommended Reading(s): Brinker MR, O’Connor DP: Exchange nailing of ununited fractures. J Bone Joint Surg Am 2007;89:177-188. Zelle BA, Gruen GS, Klatt B, et al: Exchange reamed nailing for aseptic nonunion of the tibia. J Trauma 2004;57:1053-1059. Question #: 210 A 45-year-old man sustained a humeral shaft fracture. Examination reveals no wrist extension, no thumb extension, and absent sensation in the first dorsal webspace. Interphalangeal finger extension is noted to be present. What extensor function is expected to be regained first with neurologic recovery? 1- Elbow 2- Thumb 3- Metacarpophalangeal joint 4- Wrist in neutral 5- Wrist in radial deviation Preferred Response: 5 Recommended Reading(s): Abrams RA, Ziets RJ, Lieber RL, et al: Anatomy of the radial nerve motor branches in the forearm. J Hand Surg Am 1997;22:232-237. Branovacki G, Hanson M, Cash R, et al: The innervation pattern of the radial nerve at the elbow and in the forearm. J Hand Surg Br 1998;23:167-169.

Page 175: 2009 oite review

Figure 211 Question #: 211 Which of the following is considered a risk factor for recurrence following surgical correction of the hallux valgus deformity shown in Figure 211? 1- Resection of the lateral sesamoid 2- Correction using a proximal osteotomy 3- Undercorrection of the widened 1-2 intermetatarsal angle 4- Overcorrection of the widened 1-2 intermetatarsal angle 5- Overtightening of the medial metatarsophalangeal joint capsule Preferred Response: 3 Recommended Reading(s): Richardson EG (ed): Orthopaedic Knowledge Update: Foot and Ankle 3. Rosemont, IL, American Academy of Orthopaedic Surgeons, 2004, pp 3-15. Mann RA, Coughlin MJ: Adult hallux valgus, in Coughlin MJ, Mann RA (eds): Surgery of the Foot and Ankle, ed 7. St Louis, MO, Mosby, 1999, pp 150-269.

Page 176: 2009 oite review

Figure 212 Question #: 212 A 42-year-old man sustained a shoulder injury after falling from a ladder. The MRI scan shown in Figure 212 reveals an injury to the structure marked with a white asterisk (*). Which of the following physical examination findings will most likely reveal weakness in this patient? 1- Shoulder external rotation with the arm at the side 2- Shoulder abduction 3- Belly-press maneuver 4- Forward elevation in the scapular plane 5- Elbow flexion Preferred Response: 3 Recommended Reading(s): Sanders TG, Miller MD: A systematic approach to magnetic resonance imaging interpretation of sports medicine injuries of the shoulder. Am J Sports Med 2005;33:1088-1105. Tennent TD, Beach WR, Meyers JF: A review of the special tests associated with shoulder examination: Part I: The rotator cuff tests. Am J Sports Med 2003;31:154-160.

Page 177: 2009 oite review

Question #: 213 Eight weeks after undergoing open reduction and internal fixation of a C3 distal humerus fracture, a 55-year-old man has persistent limited range of motion (40° to 90°) despite participating in an outpatient physical therapy program. Radiographs demonstrate partial healing without other abnormalities. What is the next most appropriate step in management? 1- Open capsular release 2- Arthroscopic scar releases 3- Static turnbuckle splinting 4- More active-assisted therapy sessions 5- A continuous passive motion device Preferred Response: 3 Recommended Reading(s): Gelinas JJ, Faber KJ, Patterson SD, et al: The effectiveness of turnbuckle splinting for elbow contractures. J Bone Joint Surg Br 2000;82:74-78. Doornberg JN, Ring D, Jupiter JB: Static progressive splinting for posttraumatic elbow stiffness. J Orthop Trauma 2006;20:400-404. Question #: 214 Intramedullary pressure and marrow embolization is greatest during what procedure? 1- Reaming for a femoral nail 2- Placement of an unreamed femoral nail 3- Pressurization of cement for a total hip arthroplasty 4- Impaction of a cementless total hip arthroplasty 5- Broaching for a cementless total hip arthroplasty Preferred Response: 3 Recommended Reading(s): Buckwalter JA, Einhorn TA, Simon SR (eds): Orthopaedic Basic Science: Biology and Biomechanics of the Musculoskeletal System, ed 2. Rosemont, IL, American Academy of Orthopaedic Surgeons, 2000, pp 307-316. Dobrjanski D, Saghir Z, Behdinan K, et al: Intramedullary canal pressure distribution: An experimental parametric study. J Arthroplasty 2007;22:417-427.

Page 178: 2009 oite review

Question #: 215 The Heuter-Volkmann Law is summarized best by which of the following statements? 1- Bone remodels in response to mechanical stimuli. 2- Bone formation is induced in an electronegative zone and resorbed in an electropositive zone. 3- Compression across the growth plate slows longitudinal growth. 4- Tensile load across the growth plate increases longitudinal growth. 5- The bending strength of diaphyseal bone is determined by its diameter raised to the third power. Preferred Response: 3 Recommended Reading(s): Einhorn TA, O’Keefe RJ, Buckwalter JA (eds): Orthopaedic Basic Science: Foundations of Clinical Practice, ed 3. Rosemont, IL, American Academy of Orthopaedic Surgeons, 2007, pp 115-127. Stokes IA: Mechanical effects on skeletal growth. J Musculoskelet Neuronal Interact 2002;2:277-280. Question #: 216 Treatment of infectious mononucleosis in young athletes is guided by attempts to prevent what potential complication of infection? 1- Splenic rupture 2- Pneumothorax 3- Hemorrhagic cystitis 4- Dilated cardiomyopathy 5- Acute tubular necrosis of the kidney Preferred Response: 1 Recommended Reading(s): Committee on Sports Medicine and Fitness: American Academy of Pediatrics: Medical conditions affecting sports participation. Pediatrics 2001;107:1205-1209. Farley DR, Zietlow SP, Bannon MP, et al: Spontaneous rupture of the spleen due to infectious mononucleosis. Mayo Clin Proc 1992;67:846-853.

Page 179: 2009 oite review

Figure 217a Figure 217b

Figure 217c Figure 217d

Figure 217e

Page 180: 2009 oite review

Question #: 217 Figure 217a shows an AP pelvis radiograph of a patient with an acetabular fracture. Figures 217b through 217e are successive axial CT scan cuts through this fractured acetabulum. The fracture fragment labeled A in Figure 217d represents the 1- posterior wall fragment of a transverse/posterior wall acetabular fracture. 2- posterior wall fragment of a posterior wall acetabular fracture. 3- posterior column fragment of a posterior column acetabular fracture. 4- proximal (posterior/superior) fragment of a transverse acetabular fracture. 5- distal fragment of a both column acetabular fracture. Preferred Response: 4 Recommended Reading(s): Letournel E, Judet R: Associated transverse and posterior wall fractures, in Letournel F, Judet R (eds): Fractures of the Acetabulum, ed 2. Berlin, Heidelberg, Springer-Verlag, 1993, pp 201-221. Reilly MC: Fractures of the acetabulum, in Rockwood and Green’s Fractures in Adults, ed 6. Philadelphia, PA, Lippincott Williams & Wilkins, 2006, pp 1665-1714.

Page 181: 2009 oite review

Question #: 218 When performing a posterior approach to the hip, what structure protects the anterior retractor from causing damage to the iliac vessels? 1- Psoas 2- Piriformis 3- Rectus femoris 4- Obturator externus 5- Obturator internus Preferred Response: 1 Recommended Reading(s): Skaggs DL, Kaminsky CK, Eskander-Rickards E, et al: Psoas over the brim lengthenings: Anatomic investigation and surgical technique. Clin Orthop Relat Res 1997;339:174-179. Hoppenfeld S, deBoer P: Surgical Exposures in Orthopaedics: The Anatomic Approach, ed 2. Philadelphia, PA, Lippincott Williams & Wilkins, 1994, pp 385-387. Question #: 219 After a total shoulder arthroplasty, which of the following activities should be avoided in the first 6 weeks to prevent failure of the subscapularis repair? 1- Reaching at waist level 2- Picking up a cup of coffee 3- Passive elevation to shoulder level 4- Pushing oneself up from a chair 5- Performing pendulum exercises Preferred Response: 4 Recommended Reading(s): Arroyo JS: Surgical technique and results, in Crosby LA (ed): Total Shoulder Arthroplasty. Rosemont, IL, American Academy of Orthopaedic Surgeons, 2000, pp 27-37. Wirth MA, Rockwood CA Jr: Complications of total shoulder-replacement arthroplasty. J Bone Joint Surg Am 1996;78:603-616.

Page 182: 2009 oite review

Figure 220a Figure 220b Question #: 220 What factor most closely correlates with failure of fixation after reduction and cannulated screw fixation of the fracture seen in Figures 220a and 220b? 1- Surgery after 24 hours 2- Use of three screws 3- 7-mm spread of screws on the lateral view 4- Varus malreduction 5- Early postoperative weight bearing Preferred Response: 4 Recommended Reading(s): Swiontkowski MF: Intracapsular fractures of the hip. J Bone Joint Surg Am 1994;76:129-138. Haidukewych GJ, Rothwell WS, Jacofsky DJ, et al: Operative treatment of femoral neck fractures in patients between the ages of fifteen and fifty years. J Bone Joint Surg Am 2004;86:1711-1716.

Page 183: 2009 oite review

Figure 221a Figure 221b Figure 221c

Figure 221d Figure 221e

Figure 221f

Page 184: 2009 oite review

Question #: 221 A 16-year-old girl has a large, painful anterior ankle mass. Radiographs, MRI scans, and biopsy specimens are shown in Figures 221a through 221f. What is the most likely diagnosis? 1- Blastomycosis 2- Synovial sarcoma 3- Myositis ossificans 4- Parosteal osteosarcoma 5- Pigmented villonodular synovitis Preferred Response: 5 Recommended Reading(s): Bickels J, Isaakov J, Kollender Y, et al: Unacceptable complications following intra-articular injection of yttrium-90 in the ankle joint for diffuse pigmented villonodular synovitis. J Bone Joint Surg Am 2008;90:326-328. Sharma H, Jane MJ, Reid R: Pigmented villonodular synovitis of the foot and ankle: Forty years of experience from the Scottish bone tumor registry. J Foot Ankle Surg 2006;45:329-336.

Page 185: 2009 oite review

Question #: 222 Which of the following factors has the greatest effect on the rate of implant cutout following fixation of an intertrochanteric hip fracture with a sliding hip screw implant? 1- Length of the sliding hip screw barrel 2- Angle of the sliding hip screw device 3- Tip-apex distance 4- Accuracy of reduction 5- Whether the sliding hip screw device is constrained or nonconstrained Preferred Response: 3 Recommended Reading(s): Baumgaertner MR, Solberg BD: Awareness of tip-apex distance reduces failure of fixation of trochanteric fractures of the hip. J Bone Joint Surg Br 1997;79:969-971. Templeman D, Baumgaertner MR, Leighton RK, et al: Reducing complications in the surgical treatment of intertrochanteric fractures. Instr Course Lect 2005;54:409-415. Question #: 223 In what position should the lower limb be placed during open reduction and internal fixation of the acetabulum to minimize tension on the sciatic nerve? 1- Hip flexion and knee flexion 2- Hip flexion and internal rotation 3- Hip extension and knee extension 4- Hip extension and knee flexion 5- Hip external rotation and knee flexion Preferred Response: 4 Recommended Reading(s): Borrelli J Jr, Kantor J, Ungacta F, et al: Intraneural sciatic nerve pressures relative to the position of the hip and knee: A human cadaveric study. J Orthop Trauma 2000;14:255-258.

Page 186: 2009 oite review

Figure 224a Figure 224b Question #: 224 Figures 224a and 224b show the clinical photographs of a 15-month-old boy who has an outtoeing gait. Examination reveals an external foot-progression angle of 20° and a neutral thigh-hindfoot angle. What is the most likely cause of his outtoeing? 1- Excessive femoral anteversion 2- Excessive external tibial torsion 3- Slipped capital femoral epiphyses 4- External rotation contracture of the hips 5- Bilateral developmentally dislocated hips Preferred Response: 4 Recommended Reading(s): Lincoln TL, Suen PW: Common rotational variations in children. J Am Acad Orthop Surg 2003;11:312-320. Abel MF: Orthopaedic Knowledge Update: Pediatrics 3. Rosemont, IL, American Academy of Orthopaedic Surgeons, 2006, pp 3-12.

Page 187: 2009 oite review

Figure 225a Figure 225b

Figure 225c

Page 188: 2009 oite review

Question #: 225 A 42-year-old woman has an enlarging mass in her foot and she reports that recently it has become painful to stand on. T1- and T2-weighted MRI scans are shown in Figures 225a and 225b. Figure 225c shows a biopsy specimen. What immunohistochemical marker is most likely positive? 1- S100 2- CD99 3- Cytokeratin 4- PTH 5- Estrogen receptor beta Preferred Response: 5 Recommended Reading(s): Deyrup AT, Tretiakova M, Montag AG: Estrogen receptor-beta expression in extraabdominal fibromatoses: An analysis of 40 cases. Cancer 2006;106:208-213. Weiss SW, Goldblum JR: Enzinger and Weiss’ Soft Tissue Tumors, ed 5. Philadelphia, PA, Mosby Elsevier, 2008, pp 237-246.

Page 189: 2009 oite review

Figure 226 Question #: 226 Figure 226 shows the clinical photograph of a 5-month-old girl who has a flexible deformity of her toe. What is the most appropriate management? 1- Observation 2- Genetics consultation 3- Syndactylization to the adjacent toe 4- Daily taping and stretching of the toe 5- Epiphyseolysis of the bracket epiphysis Preferred Response: 1 Recommended Reading(s): Sweetnam R: Congenital curly toes: An investigation into the value of treatment. Lancet 1958;2:398-400. Kasser JR: The foot, in Morrissy RT, Weinstein SL (eds): Lovell and Winter’s Pediatric Orthopaedics, ed 6. Philadelphia, PA, Lippincott Williams & Wilkins, 2006, pp 1257-1328.

Page 190: 2009 oite review

Question #: 227 A 45-year-old patient reports wrist pain and swelling following a fall from a ladder. Radiographs show a perilunate dislocation. Examination reveals an intact motor exam but slightly diminished two-point discrimination in the median nerve distribution. What is the next most appropriate step in management? 1- Closed reduction 2- Open reduction because the lunate is usually irreducible due to a button-hole tear in the capsule 3- CT to rule out an occult scaphoid fracture prior to reduction 4- MRI to identify specific ligament tears and rule out an occult scaphoid fracture 5- Observation Preferred Response: 1 Recommended Reading(s): Melone CP Jr, Murphy MS, Raskin KB: Perilunate injuries: Repair by dual dorsal and volar approaches. Hand Clin 2000;16:439-448. Kozin SH: Perilunate injuries: Diagnosis and treatment. J Am Acad Orthop Surg 1998;6:114-120. Question #: 228 The use of BMP-2 has been shown to lead to fewer reoperations in what clinical scenario? 1- Acute open tibia fractures treated with intramedullary nails 2- Acute open tibia fractures treated with external fixators 3- Metaphyseal proximal tibia fractures treated with locking plates 4- Tibial nonunions treated with plates 5- Tibial nonunions treated with intramedullary nails Preferred Response: 1 Recommended Reading(s): Swiontkowski MF, Aro HT, Donell S, et al: Recombinant human bone morphogenetic protein-2 in open tibial fractures: A subgroup analysis of data combined from two prospective randomized studies. J Bone Joint Surg Am 2006;88:1258-1265. Govender S, Csimma C, Genant HK, et al: Recombinant human bone morphogenetic protein-2 for treatment of open tibial fractures: A prospective, controlled, randomized study of four hundred and fifty patients. J Bone Joint Surg Am 2002;84:2123-2134.

Page 191: 2009 oite review

Figure 229a Figure 229b

Figure 229c

Page 192: 2009 oite review

Question #: 229 A 28-year-old man reports a 6-month history of right wrist pain. The symptoms came on insidiously and he denies any history of trauma. A radiograph and MRI scan are shown in Figures 229a and 229b. Wrist arthroscopy findings in the ulnocarpal joint are shown in Figure 229c. What is the most appropriate treatment at this time? 1- Reconstruction of the lunotriquetral ligament tear 2- Ulnar shortening osteotomy 3- Arthroscopic debridement of the triangular fibrocartilage complex (TFCC) 4- Arthroscopic repair of the TFCC 5- Matched hemiresection arthroplasty of the distal ulna Preferred Response: 2 Recommended Reading(s): Baek GH, Chung MS, Lee YH, et al: Ulnar shortening osteotomy in idiopathic ulnar impaction syndrome. J Bone Joint Surg Am 2005;87:2649-2654. Chun S, Palmer AK: The ulnar impaction syndrome: Follow-up of ulnar shortening osteotomy. J Hand Surg Am 1993;18:46-53.

Page 193: 2009 oite review

Figure 230 Question #: 230 The patient shown in Figure 230 is having difficulty managing stairs after cruciate-retaining total knee arthroplasty. What is the most likely explanation? 1- Elevation of the joint line 2- Patellar tendon rupture 3- Posterior cruciate insufficiency 4- Anterior cruciate insufficiency 5- External rotation of the femoral component Preferred Response: 3 Recommended Reading(s): Pagnano MW, Hanssen AD, Lewallen DG, el al: Flexion instability after primary posterior cruciate retaining total knee arthroplasty. Clin Orthop Relat Res 1998;356:39-46. Waslewski GL, Marson BM, Benjamin JB: Early incapacitating instability of posterior cruciate ligament retaining total knee arthroplasty. J Arthroplasty 1998;13:763-767.

Page 194: 2009 oite review

Figure 231 Question #: 231 A 5-year-old girl has shoulder asymmetry as well as restricted range of motion of the left shoulder. A three-dimensional CT reconstruction is shown in Figure 231. What is the most likely cause of the shoulder condition? 1- Upper thoracic myelodysplasia 2- Intrauterine exposure to cytomegalovirus 3- Birth palsy involving the upper cervical nerve roots 4- Hypoplasia of the pectoralis major and trapezius muscles 5- Failure of the scapula to migrate caudally during fetal development Preferred Response: 5 Recommended Reading(s): Bellemans M, Lamoureux J: Results of surgical treatment of Sprengel deformity by a modified Green’s procedure. J Pediatr Orthop B 1999;8:194-196. Leibovic SJ, Ehrlich MG, Zaleske DJ: Sprengel deformity. J Bone Joint Surg Am 1990;72:192-197. Borges JL, Shah A, Torres BC, et al: Modified Woodward procedure for Sprengel deformity of the shoulder: Long-term results. J Pediatr Orthop 1996;16:508-513.

Page 195: 2009 oite review

Figure 232 Question #: 232 The tumor shown in Figure 232 is located in which of the following muscles? 1- Sartorius 2- Vastus medialis 3- Biceps femoris 4- Rectus femoris 5- Semimembranosus Preferred Response: 4 Recommended Reading(s): Miller M, Gomez B: Anatomy, in Miller MD (ed): Review of Orthopaedics, ed 3. Philadelphia, PA, WB Saunders, 2000, pp 519-585. Hoppenfeld S, deBoer P: Surgical Exposures in Orthpaedics, ed 2. Philadelphia, PA, 1994, pp 330-331.

Page 196: 2009 oite review

Question #: 233 The changes that have occurred in recent years to practice patterns for treating geriatric intertrochanteric femoral fractures with plates or intramedullary nails are best characterized by which of the following statements? 1- There is little geographic variation in device use. 2- There is increased use of intramedullary nails, which have a higher Medicare reimbursement. 3- Uncommon use of intramedullary nails continues despite evidence of superiority to plates. 4- Intramedullary nails are the most commonly used method based on clear scientific evidence of superior patient outcomes. 5- Despite wide geographic variation, overall the two devices are equally used. Preferred Response: 2 Recommended Reading(s): Anglen JO, Weinstein JN: American Board of Orthopaedic Surgery Research Committee. Nail or plate fixation of intertrochanteric hip fractures: Changing pattern of practice. A review of the American Board of Orthopaedic Surgery Database. J Bone Joint Surg Am 2008;90:700-707. Forte ML, Virnig BA, Kane RL, et al: Geographic variation in device use for intertrochanteric hip fractures. J Bone Joint Surg Am 2008;90:691-699. Question #: 234 Wrist injuries often occur after a fall on the outstretched hand. When the wrist is extended, force transmission is shifted to which articulation? 1- Radiolunate 2- Radioscaphoid 3- Distal radioulnar 4- Triangular fibrocartilage-lunate 5- Triangular fibrocartilage-triquetrum Preferred Response: 2 Recommended Reading(s): Majima M, Horii E, Matsuki H, et al: Load transmission through the wrist in the extended position. J Hand Surg Am 2008;33:182-188. Weber ER, Chao EY: An experimental approach to the mechanism of scaphoid waist fractures. J Hand Surg Am 1978;3:142-148.

Page 197: 2009 oite review

Question #: 235 A disk herniation between the vertebral bodies of L3 and L4 that is located lateral to the neuroforamen will most likely cause clinical signs and symptoms involving what nerve root? 1- L2 2- L3 3- L4 4- L5 5- S1 Preferred Response: 2 Recommended Reading(s): Tamir E, Anekshtein Y, Melamed E, et al: Clinical presentation and anatomic position of L3-L4 disc herniation: A prospective and comparative study. J Spinal Disord Tech 2004;17:467-469. Rhee JM, Schaufele M, Abdu WA: Radiculopathy and the herniated lumbar disc: Controversies regarding pathophysiology and management. J Bone Joint Surg Am 2006;88:2070-2080.

Page 198: 2009 oite review

Figure 236a Figure 236b

Figure 236c Figure 236d

Page 199: 2009 oite review

Question #: 236 A 35-year-old woman has increasing shoulder pain. A radiograph, sagittal T2-weighted MRI scan, and CT scan are shown in Figures 236a through 236c. A biopsy specimen is shown in Figure 236d. What is the most likely diagnosis? 1- Infection 2- Tuberculosis 3- Giant cell tumor 4- Aneurysmal bone cyst 5- Telangiectatic osteosarcoma Preferred Response: 4 Recommended Reading(s): Kaila R, Ropars M, Briggs TW, et al: Aneurysmal bone cyst of the pediatric shoulder girdle: A case series and literature review. J Pediatr Orthop B 2007;16:429-436. Schwartz HS (ed): Orthopaedic Knowledge Update: Musculoskeletal Tumors 2. Rosemont, IL, American Academy of Orthopaedic Surgeons, 2007, pp 87-102.

Page 200: 2009 oite review

Question #: 237 A 22-year-old polytrauma patient sustained bilateral femoral fractures, an open grade IIIB tibial fracture, and a pneumothorax that was treated with a thoracostomy. In the operating room, immediately after appropriate debridement and irrigation of the open fracture, the patient becomes acutely unstable and hypoxic. In addition to resuscitative measures, what is the next most appropriate step in management? 1- Retrograde nailing of both femurs and nailing of the tibia 2- Retrograde nailing of both femurs and external fixation of the tibia 3- Two-team plating of both femurs and external fixation of the tibia 4- External fixation of both femurs and external fixation of the tibia 5- External fixation of both femurs and percutaneous tibial plating Preferred Response: 4 Recommended Reading(s): Roberts CS, Pape HC, Jones AL, et al: Damage control orthopaedics: Evolving concepts in the treatment of patients who have sustained orthopaedic trauma. Instr Course Lect 2005;54:447-462. Turen CH, Dube MA, LeCroy MC: Approach to the polytraumatized patient with musculoskeletal injuries. J Am Acad Orthop Surg 1999;7:154-165. Question #: 238 A 12-year-old boy is referred for scoliosis treatment. He has moderate pectus excavatum. He is tall and slender, with a ratio of 1.2 of wingspan to height, a high arched palate, and a positive Steinberg thumb sign. On echocardiogram, he has a dilated aortic arch. He has a 32° curve measured from T-2 to T-11. What is the most likely long-term outcome of brace treatment? 1- Aggravation of his pectus excavatum 2- Stabilization of the curve magnitude 3- Decrease in the curve magnitude 4- Progression of the curve magnitude 5- Exacerbation of thoracic kyphosis Preferred Response: 4 Recommended Reading(s): Jones KB, Sponseller PD, Erkula G, et al: Symposium on the musculoskeletal aspects of Marfan syndrome: Meeting report and state of the science. J Orthop Res 2007;25:413-422. Jones KB, Erkula G, Sponseller PD, et al: Spine deformity correction in Marfan syndrome. Spine 2002;15:2003-2012.

Page 201: 2009 oite review

Question #: 239 An otherwise healthy 17-year-old wrestler suspects that he has been bitten on the neck by a spider. He has no recent history of hospitalization. Other members of his wrestling team have also recently reported skin lesions. Examination reveals superficial abscess formation on the right side of his neck. No other skin lesions are noted. The patient is afebrile. What is the most likely cause of the abscess? 1- Highly drug-resistant Pseudomonas aeruginosa 2- Extremely drug-resistant Mycobacterium tuberculosis 3- Vancomycin-resistant Enterococcus faecium 4- Penicillin-resistant Clostridium perfringens 5- Methicillin-resistant Staphylococcus aureus Preferred Response: 5 Recommended Reading(s): Marcotte AL, Trzeciak MA: Community-acquired methicillin-resistant staphylococcus aureus: An emerging pathogen in orthopaedics. J Am Acad Orthop Surg 2008;16:98-106. Bach HG, Steffin B, Chhadia AM, et al: Community-associated methicillin-resistant staphylococcus aureus hand infections in an urban setting. J Hand Surg Am 2007;32:380-383. Question #: 240 The female athlete triad refers to which of the following? 1- Anterior cruciate ligament tears, overuse tendinopathy, and osteoporosis 2- Eating disorders, amenorrhea, and osteoporosis 3- Knee injuries, amenorrhea, and carpal tunnel syndrome 4- Tendinitis, stress fractures, and scoliosis 5- Ligament laxity, anemia, and urinary tract infections Preferred Response: 2 Recommended Reading(s): Garrick JG (ed): Orthopaedic Knowledge Update: Sports Medicine 3. Rosemont, IL, American Academy of Orthopaedic Surgeons, 2004, pp 345-352. Nattiv A, Agostini R, Drinkwater B, et al: The female athlete triad: The inter-relatedness of disordered eating, amenorrhea, and osteoporosis. Clin Sports Med 1994;13:405-418.

Page 202: 2009 oite review

Figure 241a Figure 241b

Figure 241c

Page 203: 2009 oite review

Question #: 241 A 52-year-old man is injured in a rollover ATV accident. He describes an immediate loss of strength in the arms following the accident, which is worse on the right side than on the left. He currently reports severe burning dysesthesias in both arms. His lower extremity strength is normal to manual examination; however, he demonstrates very brisk reflexes in the patellar tendons and five to six beats of clonus bilaterally. He has an up-going toe on the right and has a frontal scalp laceration just above his forehead but is otherwise uninjured. CT and MRI scans are shown in Figures 241a through 241c. Which of the following most accurately describes this patient’s injury? 1- Conversion disorder 2- Bilateral brachial plexus injury 3- Unstable ligamentous injury 4- Complete spinal cord injury 5- Central spinal cord injury Preferred Response: 5 Recommended Reading(s): Harrop JS, Sharan A, Ratliff J: Central cord injury: Pathophysiology, management, and outcomes. Spine J 2006,6:198S-206S. Epstein N, Epstein JA, Benjamin V, et al: Traumatic myelopathy in patients with cervical spinal stenosis without fracture or dislocation: Methods of diagnosis, management, and prognosis. Spine 1980;5:489-496.

Page 204: 2009 oite review

Figure 242 Question #: 242 Figure 242 shows the preoperative radiograph of a 72-year-old woman who underwent primary total knee arthroplasty and now describes shifting of the patella during flexion and extension. What is the most likely explanation for these symptoms? 1- Medialization of the patella 2- Damage to the iliotibial band 3- Medial collateral ligament laxity 4- Excessive external rotation of the femoral component 5- Excessive internal rotation of the femoral component Preferred Response: 5 Recommended Reading(s): Lee GC, Cushner FD, Scuderi GR, et al: Optimizing patellofemoral tracking during total knee arthroplasty. J Knee Surg 2004;17:144-149. Briard JL, Hungerford DS: Patellofemoral instability in total knee arthroplasty. J Arthroplasty 1989;4:S87-S97.

Page 205: 2009 oite review

Question #: 243 Management of an olecranon fracture through triceps advancement is best indicated in 1- a transverse fracture in a young adult. 2- a transverse fracture in an elderly patient with osteoporosis. 3- an oblique fracture in both young and elderly patients. 4- a comminuted fracture involving proximal 40% of the olecranon in a young adult. 5- a comminuted fracture involving proximal 40% of the olecranon in an elderly low demand patient. Preferred Response: 5 Recommended Reading(s): Hak DJ, Golladay G Jr: Olecranon fractures: Treatment options. J Am Acad Orthop Surg 2000;8:266-275. Veillette CJ, Steinmann SP: Olecranon fractures. Orthop Clin North Am 2008;39:229-236. Question #: 244 Transverse instability of the Lisfranc joint is the result of injury to the interosseous first cuneiform-second metatarsal ligament (Lisfranc’s ligament) and which of the following ligaments? 1- No other ligament injury is necessary 2- Spring ligament 3- Plantar ligament between the first cuneiform and the second and third metatarsals 4- Long plantar ligament 5- Bifurcate ligament Preferred Response: 3 Recommended Reading(s): Kaar S, Femino J, Morag Y: Lisfranc joint displacement following sequential ligament sectioning. J Bone Joint Surg Am 2007;89:2225-2232. Fischgrund JS (ed): Orthopaedic Knowledge Update 9. Rosemont, IL, American Academy of Orthopaedic Surgeons, 2008, pp 485-492.

Page 206: 2009 oite review

Figure 245 Question #: 245 What type of lateral plating technique is most appropriate for the fracture shown in Figure 245? 1- Locked 2- Buttress 3- Bridge 4- Neutralization 5- Dynamic compression Preferred Response: 2 Recommended Reading(s): Baumgaertner MR, Tornetta P III (eds): Orthopaedic Knowledge Update: Trauma 3. Rosemont, IL, American Academy of Orthopaedic Surgeons, 2005, pp 419-429. Bucholz RW, Heckman JD, Court-Brown C: Fractures of the proximal tibia, in Bucholz RW, Heckman JD, Court-Brown C (eds): Rockwood and Green’s Fractures in Adults, ed 6. Philadelphia, PA, Lippincott Williams & Wilkins, 2006, pp 1999-2036.

Page 207: 2009 oite review

Figure 246 Question #: 246 What structure is most at risk for injury during percutaneous placement of the plate shown in Figure 246? 1- Superficial peroneal nerve 2- Deep peroneal nerve 3- Posterior tibial artery or nerve 4- Posterior tibial tendon 5- Flexor digitorum communis Preferred Response: 1 Recommended Reading(s): Deangelis JP, Deangelis NA, Anderson R: Anatomy of the superficial peroneal nerve in relation to fixation of tibia fractures with the less invasive stabilization system. J Orthop Trauma 2004;18:536-539. Roberts CS, King D, Wang M, et al: Should distal interlocking of tibial nails be performed from a medial or a lateral direction? Anatomical and biomechanical considerations. J Orthop Trauma 1999;13:27-32. Wolinsky P, Lee M: The distal approach for anterolateral plate fixation of the tibia: An anatomic study. J Orthop Trauma 2008;22:404-407.

Page 208: 2009 oite review

Question #: 247 A football player is tackled and sustains a direct blow to his anterolateral thigh. Examination reveals a significant loss of motion and an extensor lag of 30°, and he is unable to return to play. Immediate management should include 1- hematoma evacuation. 2- thigh compartment releases. 3- immediate range of motion and stretching. 4- splinting the knee in extension. 5- splinting the knee in hyperflexion (120°) overnight. Preferred Response: 5 Recommended Reading(s): Ryan JB, Wheeler JH, Hopkinson WJ, et al: Quadriceps contusions: West Point update. Am J Sports Med 1991;19:299-304. Aronen JG, Garrick JG, Chronister RD, et al: Quadriceps contusions: Clinical results of immediate immobilization in 120 degrees of knee flexion. Clin J Sport Med 2006;16:383-387.

Page 209: 2009 oite review

Figure 248a Figure 248b

Figure 248c

Page 210: 2009 oite review

Question #: 248 A 49-year-old woman has medial foot pain, swelling, and progressive loss of arch height. Nonsurgical management has failed to provide symptomatic relief. Examination reveals full active and passive range of motion of her ankle and hindfoot and pain with resisted inversion. Radiographs are shown in Figures 248a through 248c. Which of the following surgical treatments is most appropriate for this patient? 1- Triple arthrodesis 2- Isolated flexor hallucis longus transfer to the navicular 3- Isolated flexor digitorum longus transfer to the navicular 4- Flexor digitorum longus transfer to the navicular with lateral calcaneal displacement osteotomy 5- Flexor digitorum longus transfer to the navicular with calcaneal osteotomy for lateral column lengthening Preferred Response: 5 Recommended Reading(s): Myerson MS: Adult acquired flatfoot deformity: Treatment of dysfunction of the posterior tibial tendon. Instr Course Lect 1997;46:393-405. Pinney SJ, Lin SS: Current concepts review: Acquired adult flatfoot deformity. Foot Ankle Int 2006;27:66-75.

Page 211: 2009 oite review

Question #: 249 A 7-year-old boy is brought to the emergency department by his grandmother, who only speaks Spanish. You speak no Spanish, but the child speaks fluent English. The child indicates that he was born in the United States, but his mother is still in Mexico. He needs surgical reduction and internal fixation for a type III supracondylar humerus fracture. How should informed consent for the surgery be obtained? 1- Talk to the grandmother using the child as a translator. 2- Talk to the grandmother using a Spanish-speaking nurse as a translator. 3- Call the mother in Mexico using the child as a translator over the phone. 4- Call the mother in Mexico using a Spanish-speaking nurse as a translator. 5- Do the surgery and document that this is an emergency and informed consent for the patient was not obtainable. Preferred Response: 4 Recommended Reading(s): Bhatacharyya T, Yeon H, Harris MB: The medical-legal aspects of informed consent in orthopaedic surgery. J Bone Joint Surg Am 2005;87:2395-2400. Wenger NS, Lieberman JR: Achieving informed consent when patients appear to lack capacity and surrogates. Clin Orthop Relat Res 2000;378:78-82. Lindseth RE: Ethical issues in pediatric orthopaedics. Clin Orthop Relat Res 2000;378:61-65.

Page 212: 2009 oite review

Figure 250a Figure 250b

Figure 250c Figure 250d

Figure 250e

Page 213: 2009 oite review

Question #: 250 An 8-year-old girl has had right elbow pain for the past 2 months. Radiographs, MRI scans, and a biopsy specimen are seen in Figures 250a through 250e. What is the most likely diagnosis? 1- Osteomyelitis 2- Chondroblastoma 3- Simple bone cyst 4- Eosinophilic granuloma 5- Aneurysmal bone cyst Preferred Response: 5 Recommended Reading(s): Basarir K, Piskin A, Guclu B, et al: Aneurysmal bone cyst recurrence in children: A review of 56 patients. J Pediatr Orthop 2007;27:938-943. Gibbs CP Jr, Hefele MC, Peabody TD, et al: Aneurysmal bone cyst of the extremities: Factors related to local recurrence after curettage with a high-speed burr. J Bone Joint Surg Am 1999;81:1671-1678.

Page 214: 2009 oite review

Question #: 251 When applying casts, which of the following actions results in a decreased incidence of thermal injuries? 1- Dip water temperatures are set to 68°F to 75.2°F (20°C to 24°C). 2- The cast rests on a pillow while setting. 3- The number of layers of the cast is increased. 4- The curing cast is overwrapped in fiberglass. 5- Isopropyl alcohol is applied to the exterior of the cast. Preferred Response: 1 Recommended Reading(s): Halanski MA, Halanski AD, Oza A, et al: Thermal injury with contemporary cast-application techniques and methods to circumvent morbidity. J Bone Joint Surg Am 2007;89:2369-2377. Lavalette R, Pope MH, Dickstein H: Setting temperatures of plaster casts: The influence of technical variables. J Bone Joint Surg Am 1982;64:907-911. Question #: 252 Which of the following best describes the clinical presentation of a patient who has sustained an inferior dislocation of the glenohumeral joint (luxatio erecta)? 1- Arm at side, unable to externally rotate past neutral 2- Arm at side, internally rotated with hand behind lower back 3- Arm at side, externally rotated with limited internal rotation 4- Arm abducted between 110° and 160° 5- Arm abducted to side about 45° Preferred Response: 4 Recommended Reading(s): Sewecke JJ, Varitimidis SE: Bilateral luxatio erecta: A case report and review of the literature. Am J Orthop 2006;35:578-580. Wirth MA, Rockwood CA Jr: Subluxations and dislocations about the glenohumeral joint, in Bucholz RW, Heckman JD (eds): Rockwood and Green’s Fractures in Adults, ed 5. Philadelphia, PA, Lippincott Williams & Wilkins, 2001, pp 1109-1207.

Page 215: 2009 oite review

Question #: 253 Which of the following clinical signs characterize an upper motor neuron disorder? 1- Spasticity and fasciculations 2- Spasticity and exaggerated deep tendon reflexes 3- Fasciculations and decreased deep tendon reflexes 4- Flaccid weakness, muscle atrophy, and increased deep tendon reflexes 5- Flaccid weakness, muscle atrophy, and decreased deep tendon reflexes Preferred Response: 2 Recommended Reading(s): Einhorn TA, O’Keefe RJ, Buckwalter JA (eds): Orthopaedic Basic Science: Foundations of Clinical Practice, ed 3. Rosemont, IL, American Academy of Orthopaedic Surgeons, 2007, pp 427-443. Renshaw TS, Deluca PA: Cerebral palsy, in Morrissy RT, Weinstein SL (eds): Lovell and Winter’s Pediatric Orthopaedics. Philadelphia, PA, Lippincott Williams & Wilkins, 2006, pp 551-603. Question #: 254 Sudden inability to extend the thumb 3 months after a distal radius fracture is most commonly associated with what type of injury? 1- Open volar Barton’s fracture 2- Open dorsally displaced extra-articular fracture 3- Closed nondisplaced metaphyseal fracture 4- Closed dorsally comminuted metaphyseal fracture 5- Closed displaced intra-articular fracture of the distal radius Preferred Response: 3 Recommended Reading(s): Jupiter JB, Fernandez DL: Complications following distal radial fractures. J Bone Joint Surg Am 2001;83:1244-1265. Hove LM: Delayed rupture of the thumb extensor tendon: A 5-year study of 18 consecutive cases. Acta Orthop Scand 1994;65:199-203.

Page 216: 2009 oite review

Figure 255a Figure 255b

Figure 255c

Figure 255d

Page 217: 2009 oite review

Question #: 255 A 44-year-old woman has had right hip fullness and severe aching for the past 6 months. She denies any history of trauma. A radiograph, coronal T1- and axial T2-weighted MRI scans, and a biopsy specimen are shown in Figures 255a through 255d. Treatment should include 1- surgery alone. 2- surgery and radiation therapy. 3- radiation therapy alone. 4- physical therapy. 5- chemotherapy and surgery. Preferred Response: 1 Recommended Reading(s): Prado MA, Miró RL, Leal IM, et al: Intramuscular myxoma: Differential diagnosis and review of the literature. Orthopedics 2002;25:1297-1299. Schwartz HS (ed): Orthopaedic Knowledge Update: Musculoskeletal Tumors 2. Rosemont, IL, American Academy of Orthopaedic Surgeons, 2007, pp 257-263.

Page 218: 2009 oite review

Question #: 256 The superior gluteal nerve is located between what two structures at what level above the trochanter? 1- Capsule and gluteus minimus 4 to 5 cm above the tip 2- Capsule and gluteus minimus 8 to 10 cm above the tip 3- Gluteus minimus and gluteus medius 4 to 5 cm above the tip 4- Gluteus minimus and gluteus medius 8 to 10 cm above the tip 5- Gluteus medius and gluteus maximus 8 to 10 cm above the tip Preferred Response: 3 Recommended Reading(s): Hoppenfeld S, deBoer P (eds): Surgical Exposures in Orthopaedics: The Hip and Acetabulum, ed 2. Philadelphia, PA, Lippincott Williams & Wilkins, 1994, pp 323-399. Question #: 257 A patient is undergoing a cervicothoracic osteotomy for a severe kyphotic deformity from ankylosing spondylitis. Performing this surgery in the seated position compared to the prone position is associated with a greater risk of 1- an air embolus. 2- a neurologic deficit. 3- blood loss. 4- pressure necrosis of the skin. 5- inadequate reduction of the deformity. Preferred Response: 1 Recommended Reading(s): Matjasko J, Petrozza P, Cohen M, et al: Anesthesia and surgery in the seated position: Analysis of 554 cases. Neurosurgery 1985;17:695-702. Engelhardt M, Folkers W, Brenke C, et al: Neurosurgical operations with the patient in sitting position: Analysis of risk factors using transcranial Doppler sonography. Br J Anaesth 2006;96:467-472.

Page 219: 2009 oite review

Question #: 258 Patients with familial adenomatous polyposis have a 10,000 times increased risk of developing what kind of mesenchymal tumor? 1- Neurofibroma 2- Enchondroma 3- Osteochondroma 4- Pleomorphic soft-tissue sarcoma 5- Extra-abdominal desmoid tumor Preferred Response: 5 Recommended Reading(s): Donthineni R: Orthopaedic clinics: Oncology. Orthop Clin North Am 2006;37:53-63. Menendez LR (ed): Orthopaedic Knowledge Update: Musculoskeletal Tumors. Rosemont, IL, American Academy of Orthopaedic Surgeons, 2002, pp 243-253. Question #: 259 Which of the following factors is associated with the highest rate of complications from treatment of a patient with diabetes mellitus and an ankle fracture? 1- Osteopenia 2- Renal dialysis 3- Presence of peripheral neuropathy 4- Open treatment of the ankle fracture 5- Closed treatment of the ankle fracture Preferred Response: 3 Recommended Reading(s): Chaudhary SB, Liporace FA, Gandhi A, et al: Complications of ankle fracture in patients with diabetes. J Am Acad Orthop Surg 2008;16:159-170. Costigan W, Thordarson DB, Debnath UK: Operative management of ankle fractures in patients with diabetes mellitus. Foot Ankle Int 2007;28:32-37. Jones KB, Maiers-Yelden KA, Marsh JL, et al: Ankle fractures in patients with diabetes mellitus. J Bone Joint Surg Br 2005;87:489-495.

Page 220: 2009 oite review

Question #: 260 A 25-year-old man injured his knee in a motor vehicle collision. Abnormal examination findings include 10° increased external tibial rotation at 30° and 90° knee flexion. What additional examination finding is expected? 1- Increased opening to valgus stress at 30° of knee flexion 2- Increased varus opening at 0° of knee flexion 3- Positive apprehension sign with lateral patellar translation 4- Positive pivot shift test 5- Medial tibial plateau rests 10 mm anterior to the medial femoral condyle Preferred Response: 2 Recommended Reading(s): LaPrade RF, Terry GC: Injuries to the posterolateral aspect of the knee: Association of anatomic injury patterns with clinical instability. Am J Sports Med 1997;25:433-438. Gollehon DL, Torzilli PA, Warren RF: The role of posterolateral and cruciate ligaments in the stability of the human knee: A biomechanical study. J Bone Joint Surg Am 1987;69:233-242. Question #: 261 With the diagnosis of valgus extension overload of the elbow, radiographs often show osteophyte formation on the 1- radial head. 2- posteromedial olecranon fossa. 3- posterolateral olecranon fossa. 4- tip of the olecranon process. 5- medial humeral epicondyle. Preferred Response: 2 Recommended Reading(s): Garrick JG (ed): Orthopaedic Knowledge Update: Sports Medicine 3. Rosemont, IL, American Academy of Orthopaedic Surgeons, 2002, pp 101-111. Wilson FD, Andrews JR, Blackburn TA, et al: Valgus extension overload in the pitching elbow. Am J Sports Med 1983;11:83-88.

Page 221: 2009 oite review

Question #: 262 Which of the following conditions is a contraindication to the use of most bisphosphonates? 1- Paget’s disease 2- Osteomyelitis 3- Bone infarct 4- Myasthenia gravis 5- Severe renal insufficiency/failure Preferred Response: 5 Recommended Reading(s): Lin JT, Lane JM: Bisphosphonates. J Am Acad Orthop Surg 2003;11:1-4. Rosier RN: Expanding the role of the orthopaedic surgeon in the treatment of osteoporosis. Clin Orthop Relat Res 2001;385:57-67.

Page 222: 2009 oite review

Figure 263a Figure 263b

Figure 263c Figure 263d

Page 223: 2009 oite review

Question #: 263 A 38-year-old woman underwent curettage of the lesion shown in Figures 263a through 263c at the time of anterior cruciate ligament reconstruction. A biopsy specimen is shown in Figure 263d. What is the next most appropriate step in management? 1- CT of the chest 2- Wide resection 3- Chemotherapy 4- Radiation therapy 5- Observation Preferred Response: 5 Recommended Reading(s): Weiner SD: Enchondroma and chondrosarcoma of bone: Clinical, radiologic, and histologic differentiation. Instr Course Lect 2004;53:645-649. Schwartz HS (ed): Orthopaedic Knowledge Update: Musculoskeletal Tumors 2. Rosemont, IL, American Academy of Orthopaedic Surgeons, 2007, pp 103-120. Question #: 264 In a Monteggia fracture-dislocation with an anteriorly dislocated radial head, where is the apex of the ulnar fracture? 1- Medial 2- Lateral 3- Anterior 4- Posterior 5- Posterolateral Preferred Response: 3 Recommended Reading(s): Bado JL: The Monteggia lesion. Clin Orthop Relat Res 1967;50:71-86. Fowles JV, Sliman N, Kassab MT: The Monteggia lesion in children: Fracture of the ulna and dislocation of the radial head. J Bone Joint Surg Am 1983;65:1276-1282.

Page 224: 2009 oite review

Question #: 265 What modality recommended in the AAOS Guideline on the Prevention of Symptomatic Pulmonary Embolism is recommended across all risk (low to high risk of either bleeding or pulmonary embolism) groups undergoing total hip or total knee arthroplasty? 1- Mechanical prophylaxis 2- Vena cava filter 3- Spinal anesthesia 4- Low-molecular-weight heparin 5- Synthetic pentasaccharides Preferred Response: 1 Recommended Reading(s): AAOS Clinical Guideline on Prevention of Pulmonary Embolism in Patients Undergoing Total Hip or Knee Arthroplasty. Rosemont, IL, American Academy of Orthopaedic Surgeons, 2007. http://www.aaos.org/research/guidelines/PE_guideline.pdf. Accessed on July 24, 2009. Question #: 266 Botulinum toxin A exerts its therapeutic effect via what mechanism of action? 1- Blocks presynaptic release of acetylcholine 2- Blocks reuptake of acetylcholine 3- Blocks acetylcholinesterase release 4- Increases production of acetylcholine at the nerve terminus 5- Increases acetylcholinesterase release Preferred Response: 1 Recommended Reading(s): Abbruzzese G, Berardelli A: Neurophysiological effects of botulinum toxin type A. Neurotox Res 2006;9:109-14. Das TK, Park DM. Effect of treatment with botulinum toxin on spasticity. Postgrad Med J 1989;65:208-210.

Page 225: 2009 oite review

Question #: 267 Which of the following prosthetic limb knee joints is most commonly used in childhood, is not recommended for older or weaker patients, allows only a single speed of walking, and relies solely on alignment for stance phase stability? 1- Polycentric 2- Stance-phase control 3- Fluid control 4- Constant friction 5- Variable friction Preferred Response: 4 Recommended Reading(s): Tang PC, Ravji K, Key JJ, et al: Let them walk! Current prosthesis options for leg and foot amputees. J Am Coll Surg 2008;206:548-560. Rossbuch PG, Joyce MA, Schaffer E, et al: Principles of amputee prosthetics, in Chapman MW (ed): Chapman’s Operative Orthopaedics, ed 3. Philadelphia, PA, Lippincott Williams & Wilkins, 2001, pp 3181-3199. Question #: 268 For an 8-mm nerve gap in the hand, which of the following has been shown to be as reliable and effective as the gold standard of autogenous nerve grafting? 1- Silicone sleeve repair 2- Mobilization and primary repair 3- Collagen conduit 4- Poly-L-lactic acid conduit 5- Vein graft interposition Preferred Response: 3 Recommended Reading(s): Bertleff MJ, Meek MF, Nicolai JP: A prospective clinical evaluation of biodegradable neurolac nerve guides for sensory nerve repair in the hand. J Hand Surg Am 2005;30:513-518. Waitayawinyu T, Parisi DM, Miller B, et al: A comparison of polyglycolic acid versus type I collagen bioabsorbable nerve conduits in a rat model: An alternative to autografting. J Hand Surg Am 2007;32:1521-1529. Li ST, Archibald SJ, Krarup C, et al: Peripheral nerve repair with collagen conduits. Clin Mater 1992;9:195-200.

Page 226: 2009 oite review

Question #: 269 A 68-year-old woman underwent total hip arthroplasty 2 years ago for degenerative arthritis secondary to acetabular dysplasia. She now reports an unstable hip that has dislocated six times. She walks with a severe Trendelenburg limp. Examination reveals no active abduction of the hip. The components appear well positioned. What management offers the best chance of success? 1- Re-education on hip precautions 2- Revision to a large femoral head 3- Revision to a constrained liner 4- Use of a hip abduction brace 5- Trochanteric advancement Preferred Response: 3 Recommended Reading(s): Shrader MW, Parvizi J, Lewallen DG: The use of a constrained acetabular component to treat instability after total hip arthroplasty. J Bone Joint Surg Am 2003;85:2179-2183. Lieberman JR, Berry DJ (eds): Advanced Reconstruction: Hip. Rosemont, IL, American Academy of Orthopaedic Surgeons, 2005, pp 223-231. Question #: 270 A healthy 64-year-old woman sustains a compression fracture of L3 in a fall. She is neurologically intact. On radiographic examination, the anterior column is compressed 25%, and there is 20° of localized kyphosis. Based on these findings, what is the most appropriate management? 1- Bed rest for 2 weeks followed by use of a lumbosacral orthosis 2- Application of a thoracolumbosacral orthosis and progressive increasing ambulation 3- Anterior column reconstruction and placement of structural autograft and a plate 4- Posterior short segment pedicular fixation and placement of autogenous bone graft 5- Combined anterior and posterior reconstruction Preferred Response: 2 Recommended Reading(s): Wood K, Buttermann G, Mehbod A, et al: Operative compared with nonoperative treatment of a thoracolumbar burst fracture without neurological deficit: A prospective, randomized study. J Bone Joint Surg Am 2003;85:773-781. Gertzbein SD: Scoliosis Research Society: Multicenter spine fracture study. Spine 1992;17:528-540.

Page 227: 2009 oite review

Question #: 271 What is the most effective shoe modification for reducing plantar pressures in the forefoot? 1- SACH heel 2- Rocker sole 3- Custom indepth shoe 4- Extended steel shank 5- Closed cell polyethylene foam insole Preferred Response: 2 Recommended Reading(s): Janisse DJ, Janisse E: Shoe modification and the use of orthoses in the treatment of foot and ankle pathology. J Am Acad Orthop Surg 2008;16:152-158. Brown D, Wertsch JJ, Harris GF, et al: Effects of rocker soles on plantar pressures. Arch Phys Med Rehabil 2004;85:81-86. Question #: 272 A 53-year-old woman is seen in the emergency department after sustaining an injury to her left nondominant shoulder in a fall last week. Radiographs reveal 100% displacement of the distal clavicle above the acromion. What is the best treatment option at this time? 1- Distal clavicle resection 2- Sling and early range-of-motion exercises 3- Weaver-Dunn procedure including coracoacromial ligament transfer 4- Anatomic coracoclavicular ligament reconstruction with allograft 5- Anatomic coracoclavicular ligament reconstruction with autograft semitendinosus tendon Preferred Response: 2 Recommended Reading(s): Garrick JG (ed): Orthopaedic Knowledge Update: Sports Medicine 3. Rosemont, IL, American Academy of Orthopaedic Surgeons, 2004, pp 53-77. Schlegel TF, Burks RT, Marcus RL, et al: A prospective evaluation of untreated acute grade III acromioclavicular separation. Am J Sports Med 2001;29:699-703.

Page 228: 2009 oite review

Question #: 273 The rotator cable, seen arthroscopically at the margin of the avascular zone, is an extension of the 1- biceps tendon sheath. 2- teres minor tendon. 3- subscapularis tendon. 4- coracohumeral ligament. 5- superior labrum. Preferred Response: 4 Recommended Reading(s): Clark JM, Harryman DT II: Tendons, ligaments, and capsule of the rotator cuff: Gross and microscopic anatomy. J Bone Joint Surg Am 1992;74:713-725. Burkhart SS, Lo IK: Arthroscopic rotator cuff repair. J Am Acad Orthop Surg 2006;14:333-346. Question #: 274 Disease modifying antirheumatic drug therapy such as infliximab is primarily targeted against 1- leukocytes. 2- rheumatoid factor. 3- antinuclear antibodies. 4- C-reactive protein. 5- tumor necrosis factor-alpha. Preferred Response: 5 Recommended Reading(s): Vaccaro AR (ed): Orthopaedic Knowledge Update 8. Rosemont, IL, American Academy of Orthopaedic Surgeons, 2005, pp 229-244. Elliot MJ, Maini RN, Feldmann M: Treatment of rheumatoid arthritis with chimeric monoclonal antibodies to tumor necrosis factor alpha. Arthritis Rheum 2008;58:S92-S101.

Page 229: 2009 oite review

Question #: 275 Osteoclasts are the primary cells involved in bone resorption. What is one of the most critical factors for osteoclast differentiation and activation? 1- PTH 2- BMP2 3- RANKL 4- Calcitonin 5- TNF-alpha Preferred Response: 3 Recommended Reading(s): Fischgrund JS (ed): Orthopaedic Knowledge Update 9. Rosemont, IL, American Academy of Orthopaedic Surgeons, 2008, pp 115-164. Alman BA, Howard AW (eds): Metabolic and endocrine abnormalities, in Morrissy RT, Weinstein SL (eds): Philadelphia, PA, Lippincott Williams & Wilkins, 2006, pp 167-200.